Property Final Outline

160
Exam Keys 1. Read problem a. Highlight/jot down important facts*****EVERY FACT MATTERS b. Find out what the question is asking 2. Think 3. Organize your thoughts a. Draw diagrams b. Make timeline c. Outline Key points when making arguments 1. Always consider the scope of review on appeal (ask, did the court honor that review or did they exceed their authority) o When appealing question of fact: scope of review is no reasonable fact finder can find for the other side As finding of fact, trial court established land cultivation was substantial and you can only overturn this if no reasonable fact finder would find otherwise o When appealing question of law: scope of review is de novo; the appellate court looks at it likes it’s the 1 st time 2. When making an argument, consider the effect of human behavior a. As lawyer, I can make the argument that if you hold for the other guy, it will negatively affect human behavior, but if you hold for me, it will positively affect human behavior 3. If case law goes against you, distinguish the shit out of it a. If case law is for you, draw similarities and fit case into holding 4. If your dealing with case that sets bad precedant a. 1) factually distinguish case b. 2) make policy arguments: expain why court should adopt a different rule 5. Always argue: purpose of statute 6. Always argue public policy: why the rule should be different or what behavior we want to encourage 7. ******make case law arguments, textual arguments (interpretations), statutory arguments (adverse possession, recording act), policy arguments (if you don’t find for my party: here are ramifications) 1

description

Great Property Outline

Transcript of Property Final Outline

Page 1: Property Final Outline

Exam Keys1. Read problem

a. Highlight/jot down important facts*****EVERY FACT MATTERSb. Find out what the question is asking

2. Think3. Organize your thoughts

a. Draw diagramsb. Make timelinec. Outline

Key points when making arguments1. Always consider the scope of review on appeal (ask, did the court honor that review or did

they exceed their authority)o When appealing question of fact: scope of review is no reasonable fact finder can find

for the other side As finding of fact, trial court established land cultivation was substantial and

you can only overturn this if no reasonable fact finder would find otherwiseo When appealing question of law: scope of review is de novo; the appellate court looks

at it likes it’s the 1st time2. When making an argument, consider the effect of human behavior

a. As lawyer, I can make the argument that if you hold for the other guy, it will negatively affect human behavior, but if you hold for me, it will positively affect human behavior

3. If case law goes against you, distinguish the shit out of it a. If case law is for you, draw similarities and fit case into holding

4. If your dealing with case that sets bad precedanta. 1) factually distinguish caseb. 2) make policy arguments: expain why court should adopt a different rule

5. Always argue: purpose of statute6. Always argue public policy: why the rule should be different or what behavior we want to

encourage 7. ******make case law arguments, textual arguments (interpretations),

statutory arguments (adverse possession, recording act), policy arguments (if you don’t find for my party: here are ramifications)

8. When asked to advise, sometimes the best advice is to bargain for right9. ****when city is bringing claim, it will be zoning, eminent domain or public nuissance

If asked to write brief: (only make arguments for 1 side: anticipate other side’s arguments)***whats the scope of review on appeal 1) start by saying, “trial court erred by granting sj to p. 1st error was…2 error was…2) Issue 1: the trial court erred because…

Deal with issue 1 entirely, then move to issue 23) issue 2: trial court erred because…****for each issue, 1) state the general rule 2) distinguish/draw similarities btwn cases that apply rule or distinguish and say rule does not apply4) explain case, discuss why its misapplied to facts in this case (distinguish and explain why its misapplied: can even say this case should be overruled or it was wrongly decided

1

Page 2: Property Final Outline

5) if rule goes against you, argue why court should adopt minority rule and make public policy arguments (what can of behavior do we want to encourage, goals of property law, ramifications if you find for other side)6) sum up brief: even if ll breached the most the tenant is entitled to is…

If asked to write memo: (make arguments for each side): pure issue spotting1) take issue 1, articulate the rule, then make arguments for each side2) if rule goes against you, argue why court should adopt minority rule and make public

policy arguments (what can of behavior do we want to encourage, goals of property law, ramifications if you find for the other side)

The importance of policy ends to the nature of property rights: Generally, property doctrine tries to serve four important values

o 1) reward productivity and foster efficiency, and protect people’s subjective value in their land

o 2) create simple, easily enforceable ruleso 3) create property rules that are consistent w/ societal habits and customso 4) produce fairness in terms of prevailing cultural expectations of fairness.

← Remember to think about practical consequences: cost of litigation, cost of negotiation, leverage, hold-outs, fairness, framing precedent, assumptions, etc.

When can a property owner attach his or her own value to the land? The law tends to come down on the side of property owners to choose how to value their land. But of course, in certain circumstances, the law gives way to the state.

*The power to litigate is the power to destroy.

2

Page 3: Property Final Outline

INTRODUCTION TO SOME FUNDAMENTALS

Acquisition of Property by Discovery, Capture and Creation

Acquisition by DiscoveryThe idea that property can be acquired by discovery

Johnson v M’Intosh (1823)rival claims for ownership of land in Ill. Johnson’s claim was the last link in a chain of title going back to the aboriginal inhabitants of the land. M’Intosh said his title was better bc it came from the US, although after the Indian conveyance to Johnson’s predecessor.Court held for M’Intosh bc Indians only held aboriginal title, a right of occupancy that could be cut off at any moment by the US, as successor to the Euro discovers of the land. US derived its ownership by conquest.

Acquisition by CapturePursuit is insufficient to establish a property right. You actually need to capture. Pierson.

Pierson v Post (1805) p. 17While post was hunting and chasing and pursuing a fox with his dogs, Pierson, knowing the fox was pursued, caught the fox, killed it and carried it away. Post sued in trespass on the case arguing he acquired a property right in the fox by pursing it.Mere pursuit gave Post no legal right to the fox. Pierson, however, acquired such rights by capturing it.

Capture is not required to acquire a right: wounding is also a viable way. Reason for the rule that pursuit is not occupancy is for sake of preserving peace, order and

certainty in society Dissent (Livingston): custom among sportsman was for pursuit to grant a property right to the pursuer. This rule encourages fox killing bc chickens are important.

You hunt foxes either for sport, to protect chickens or for gain (mear). If you are trying to protect chickens you would be happy to have Pierson come along, otherwise, Piersons would discourage you from hunting. Depends on how you conceptualize the sport.

Acquisition of Property by Adverse possessionRational: facilitates marketability and transfer of land even though there are clouds in title search, rewards person who uses, works and improves property for long time

Elements of Adverse Possession: each element with the case behind it*****FIRST LOOK AT THE STATUTE*****In general (common law), occupant acquires title to land by adverse possession if his possession is:

1. Element 1: Actual Entry Giving Exclusive Possession: the possessor must actually physically take possession of the owners land making neighbors and observers regard the occupant as a person exercising exclusive dominion. Exclusive possession is one which must not be shared with either the true owner or the public.

3

Page 4: Property Final Outline

a. Actual Possession: Minority approach (NY): conduct that constitutes actual possession when adverse possession is premised on claim of right, claimant must cultivate, improve or substantially enclose the property. Van Valkenburgh

Van Valkenburgh v Lutz (1952); NY COALutz owned piece of land, but found if easier to get to the street by crossing a piece of land he didn’t own and cleared a traveled way to do so. The Van Valkenburghs bought the property at foreclosure sale and constructed a fence across the traveled way. Suit 1: Lutz sued to enjoin the VV from interfering with his right of way. Trail court held for Lutz awarding him a right of way over the traveled way (prescriptive right)Suit 2: VV sued Lutz to compel removal of their encroachments on the VV’s land. Lutz argued for adverse possession.Court held no adverse possession bc proof fails to establish actual occupation under the NY statute: protected by substantial enclosure or usually cultivated or improved

1) lutz’s cultivation of the garden did not utilize the whole premises claimed and the area cultivated did not have defined boundaries (needed to cultivate a plot for the statutory period)

2) Lutz’s claim of title was not hostile; he thought he owned it but was mistaken in that belief that e owned it and he admitted VV was true owner

2. Element 2: Open and Notorious : acts of possession must be so visible and obvious that a reasonable owner who inspects the land will receive notice of the adverse claim. Occupation is open if use of the property is in the manner that an reasonable owner would use it under the circumstances.

a. the owner must be on notice of the possession. Kunto. b. If the possession is not objectively notorious but the landowner had actual knowledge,

that is sufficient. Manillo.c. Not necessary to show owner obtained actual knowledge of the claim. Manillod. As long as adverse possessor is open and notorious, NY requirements for cultivation

and improvement or substantial enclosure is not big deal

Manillo v Gorski (1969)D, adverse possessor built a set of steps that encroached on p’s land by 15 inches.Court held if true owner had knowledge of the encroachment, it was open and notorious and therefore was adversely possessed. If not, court must determine whether true owner should convey the disputed tract, and if so, what consideration should be paid.

3. Element 3: Adverse or hostile Under Claim of Right/title : state of mind of adverse possessor

a. Objective test : adverse possessor’s state of mind is irrelevanti. Key question: did true owner authorize possession? If no permission from

true owner, adverse possessor has satisfied this element under objective test.1. True owner argues it was permissive2. Adverse possessor argues he is using as true owner would without

consent3. In othens, there was a gate and that was considered permission.

court says the your character and actions can show permissionb. Good faith test : adverse possessor must believe in good faith he owns the land

i. Was it permissive

4

Page 5: Property Final Outline

1. True owner argues adverse boy didn’t have good faith or bad faith bc he though he was permissively occupying the premises

a. True owner argues it’s a lease and lease is permissive2. Adverse boy argues I believed I had claim of title3. In othens, there was a gate and that was considered permission.

court says the your character and actions can show permissionc. Bad faith (intentional trespass test): adverse possessor must know he does not own

the land and must intend to take title from true owneri. Was it permissive?

1. True owner argues its not good or bad faith bc it was lease and lease is permissive

2. Adverse boy argues he read lease, knew it expired and he was therefore on land in bad faith

3. In othens, there was a gate and that was considered permission. court says the your character and actions can show permission

d. *****IF TRUE OWNER AUTHORIZES OR CONSENTS TO POSSESSION, IT IS NOT CONSIDERED ADVERSE OR HOSTILE IN ALL 3 JURISDICTIONS

1. a lease is always permissive occupation and not ownership2. In othens, there was a gate and that was considered permission.

court says the your character and actions can show permissione. ****make public policy argument: if the court doesn’t adopy any test, it should not

be bad faith: people in good faith should be protected. They have more justifiable claim bc objective of doctrine is to allow people to be settled and to have strong reliance interests so they can investent in their property. People in good faith have deeper reliance interest, they prob invested more

West v Tilley (1970); NY COAAdverse possessors owned land on a shore. The built a wall along the shore lien, but extended it along the frontage owned by true owner. Later, they built an additional wall, effectively usurping all of the true owner’s lake side property.Court hold although the adverse possessors took possession by mistake, there claim was still hostile. The possession was visible and claim of right was held with an intention to possess the land occupied under the belief that he owned the land. Even a casual inspection of the boundary lines of the property would have revealed the adverse possessor’s occupation.

Van Valkenburgh’s dicta: VV stated that the garage encroachment by mistake was insufficient to establish hostility. However, lutz also insufficiently cultivated the land, therefore, this is DICTA and is OVERRULED in WEST.

Walling v Przybylo (2006); NY COAAdverse possessors owned a lot next to the true owner. They bulldozed and deposited topsoil on the northerly side of yard; they constructed an underground dog wire fence to enclosure their dog; they continuously mowed, graded, raked, planted and watered the grassy area; they installed PVC pipe and bird house. When learning of survey by true owners, adverse possessors sued to quite title.Court held that an adverse possessor’s claim of right or ownership will not be defeated by mere knowledge than another holds legal title (no regard to good or bad faith). REVERSED BY NY STATUTE

NY Statute for Adverse Possession: 501(2) Same as common law in that it Requires open and notorious, exclusive and actual

5

Page 6: Property Final Outline

Different in regards to claim of right: o IN NY, NEED AFFIRMATIVE EVIDENCE THAT A REASONABLE PERSON

WOULD BELIEVE THEY OWNED THE LAND Now in NY, cannot gain title unless adverse possessor can introduce evidence that would

justify a reasonable belief of ownership: statements by the seller, survey that owner would take up

4. Element 4: Continuous possession for statutory period: adverse possessor continous possession must be of the character of a reasonable owner,

a. Kunto: summer months only for 10 years was continuous possession bc average owner of similar property would use it in this way

b. If true owner reenters the land and retakes possession in open and notorious manner, required continuity and exclusivity ends

c. EXCEPTION: TACKING: where several successive purchasers received record of title in a continuous manner, their privity of estate permits tacking their time of possession together. Kunto

i. In order to tack, the adverse possessors must each satisfy the same claim of title (if good faith juris, all tackers must be in good faith) and elements of adverse possession AND must be in privity (continuously possessing the land and believing they have connection or privity of estate to past person on land)

Howard v Kunto (1970)Kunto occupied a summer house under a defective deed. Howard, record owner, sought ejectment, and kunto countered with adverse possession. Court held it was adverse possession. 1) continuous possession: occupancy during summer months for more than 10 year period by d and his predecessors together with the continued existence of the improvements on the land and beach area constitutes uninterrupted possession 2) tacking: tacking on successive possessions of property where successive purchasers received record title in a continuous manner is permitted for purposes of established adverse possessors if all the successive owners are in privity (by operation of the deed running btwn them)

In order to tack, adverse possessors must each satisfy all elements of AP and must satisfy the same claim of title (if good faith jurisdiction, must all be in good faith to tack)

Distinguish howard: the land in howard was seasonal community

Other ElementsGovt Property: under common law, adverse possession doesn’t run against govt but a few states has permitted adverse possession against govt land on same terms as private land. Others permit it only if possession continues for a period much longer than applies in private lands. Cotenant as owner: possession by cotenant is not considered adverse or hostile to other cotenants (rebuttable presumption of permissive use); in order to begin adverse possession, cotenant in possession must clearly claim sole ownership by either 1) physically ousting cotenants 2) taking steps that clearly notify cotenants of the claimLandlord/tenant: tenant who holds possession of leased premises with permission of ll cannot assert adverse possessionFuture Interest: future interest is immune from adverse possession until the interest holder is entitled to immediate possession of the land. You cant adversely possess a future interest holder bc they don’t have power to eject.

6

Page 7: Property Final Outline

THE SYSTEM OF ESTATES

Terminology: decedent: someone who dies w/o a will testator: someone who dies w/ a will issue: means ALL descendants alienability: right to dispose of land during a lifetime

“O conveys”—o is making a transfer during o’s life using a deed“o devises”— means its in a will and his will conveying the land (o bequeaths is same as will)

POSSESSORY ESTATES

1. FEE SIMPLE: Fee simple may endure forever and is as close to absolute ownership as our law recognizes. A fee simple absolute is alienable (transferrable or assignable), devisable and descendible.

To create fee simple absolue: “to a and his heirs” Inheritance of a fee simple

Heirs: if person dies intestate, the decedants real property descends to his heirs (intestacy statute).

o There is NO SUCH THINGS HAS HAVING HEIRS WHILE YOU ARE LIVING.

Issue: descendants / ancestors: parents / Collaterals: all person related by blood to descendant who are neither descendants nor ancestors

Escheat: if person dies intestate w/o heirs, person’s real and personal property escheats to the state.

2. THE FEE TAIL: freehold estate whose duration was measured by the lives of the lineal descendants of a designated person

i.e. to “a and heirs of his body”. Estate would endure as long as a’s bloodline continued virtually extinct in the US and YOU SHOULD NEVER USE majority of states interpret fee tail language as creating fee simple absolute in first taker

3. THE LIFE ESTATE: Life estate can endure for the life of a person, which means the grantor can control who takes the property at the life tenants death. A can transfer the life estate to b, but b only has life estate per autre vie- an estate that is measured by A’s life span.

Life estate owner may collect all rents and profits generated from the land and use property as would an owner except he may not destroy the value of successive future interests.

Ambiguous language in conveyance by grantor holding fee simple absolute is judicially interpreted as transferring fee simple absolute

Trust provides tax advantages and avoids probate

White v Brown (1977)Decedent leaves a will which reads, “I wish Evelyn white to have my home to live in and not to be sold.” Decedant also leaves personal property to Sandra. Decedent’s sisters quitclaimed any interest in the residence, but her nieces and nephews did not. White filed an action to obtain construction of the will. Evelyn white argues for fee simple. Nieces want to construe will as only leaving Evyln life estate

7

Page 8: Property Final Outline

(under the intestacy statute, they hare decedants closest heirs and have right to property if the will does not provide for remainder after life estate)Court held the will passed a fee simple absolute in the home and her attempted restraint on alienation must be declared as void as inconsistent with nature of the estate devised and contrary to public policy.

At common law, there was a presumption of life estate, but Tenn statute reversed the presumption (now a rebuttable presumption of fee simple)

Problem with restraints on alienationo Makes property unmarketableo Discourages improvements on land (no value added)o If the life tenant cannot afford upkeep or doesn’t value the land and another person

would buy it for fmv, this creates inefficiency and we want free market This devalues land and other lands soar in price creating inefficiency in market

bc people who want to buy land cannot afford ito When bank sees restriction on alienation, cant get mortgage or line of credit bc bank

cannot foreclose on house (this constitutes a sale)

Argument for life Estate when conveyance ambiguouso GRANTOR”S INTENTo Given that it expressly says my home is not to be sold is NOT consistent with fee

simple absolute—that’s consistent with giving someone a limited righto “I wish for eveyln to live in it”—seems to imply life estate

decedant knew how to make gift in fee simple w/o restriction by leaving her personal property—if she wants to do it, she knows how to and she didn’t do that in regards to evelyn

Argument for fee simple when conveyance ambiguouso There is presumption against life estate.

In order to sell house, or use it as collateral, life teant must agree with remaidnerman and transaction costs are high (holdout problem)

o Conflict: Life tenants want to max value now—remainderman want to max value later (different time horizons creates great tension)

Waste doctrine: Waste of property by life tenant is an actionable injury for a remainderman. Purpose of doctrine is to use land efficiently

Waste occurs when the life tenant permanently impairs the property’s condition or value to the future interest holder’s etriment

o Waste doctrine restraints present estate owner from acting in manner that unreasonably injuries the affected land thus reducing the value of the future interest

o Law presumes grantor intended estate holder to pass possession of land to future interest holder in approx the same condition as it was received

affirmative waste: voluntary acts that are more than trivially injurious (substantially reduces the value of the property

o demolition of worthless improvements in order to permit productive use of land will not constitute waste

Permissive waste: negligent failure to act and take reasonable care of the propertyo Failure to pay prop taxes, mortgage payments

8

Page 9: Property Final Outline

Remainderman argument for waste doctrineo grantor’s wishes not to sell property:

1) grantor wanted remainderman to get something of value and wanted the remainderman to get the property and not just money from a sale

2) allowing a to cut down all the tress will promote A acting inefficiently—cutting trees down before they mature bc he doesn’t have time to wait

o to sell property life tenants is materially devaluing land compared to the potential worth if life tenant fails to sell prop now, I forcloses the possibility of future profit

(opportunity cost) Counter: time element is too speculative and gives future interest holders

too much power Life tenant

grantor wishes to provide life tenant sall property: 1) remainderman might try to extort $ from a for his consent: if

remainderman could prevent the life tenant, the remainderman might assert a claim of sentimental value even though there isn’t one in an effort to get money from the life tenant.

2) it was grantor’s wish that A get use of the land and not put a at b’s mercy 3) if life estate wants to sell and remainderman attachs subjective value to

proerty and does not want to sell, buy life tenant’s interest don’t sell property

this goes counter to intention of the grantor; he wanted us both to have the house

life tenant has right to put property to his own use in absence of restriction by grantor

Baker v Weedon (1972)Decedents owns farm and creates life estate in Anna with a condition remainder; Anna’s issue, but if none, Florence’s children. Language in the instrument prevents creation of fee simple. Anna wants to sell property based on economic waste (farm is making $ to support her), but the most productive use, until the highway is built, is to use the property, as it is being used, for agricultural use. Ann is actually arguing she will get a better distribution of the land: sell it for 168k, invest in a trust estate and live off the estate’s interest (16k per year). Problem is that land is worth more for speculation due the highway being built rather than farming. Grandchildren object to sale saying the future worth of house is 336k. Court found no economic waste and remands back to court bc best interests of all parties would NOT be served by judicial sale. This would cause great financial lose to remainderman. Courts sets up the parties for settlement negotiations.

Grantor’s Intento Court looks at John Weedon’s intention and have respect for what the would have

wanted for herTrust better arrangement than life estate; resolves the tension of self interests of life tenant and remainderman if they diverge

a trustee holds legal fee simple as the manager of the property and may be directed to pay all income to the life tenant. Trustee has power to sell, lease, mortgage and do whatever a prudent person would do with respect to property.

9

Page 10: Property Final Outline

trustee has duty of impartiality; must treat all beneficiaries equally and trust document can stipulate to treat someone as principal beneficiary (ann wheedon)

If grant is unclear, trust resolves it; trustee has legal title to make all decisions and DOES NOT NEED CONSENT from remainderman to make decisions

Creditor protection: john could create spendthrift clause in trust and no beneficiary interest is attachable by creditors. But If use spendthrift clause, beneficiary cant assign property interest of trust to others

DEFEASIBLE ESTATES******what was grantor’s intent

Defeasible fee: fee that may last forever or come to an end if a future event happens 1. Fee simple determinable: a fee simple so limited that it will end automatically when a

stated event happenso Possibility of reverter: future interests in the fee. AUTOMATICALLY DIVESTSo Classic language (DURATION): “to the school district for so long as the land is used

for school purposes, and then to revert to grantors So long as, until, during, or while or if grantor expressly retains possibility of

reverter or uses other words indicating an intention to create an automatic return of possession in fee simple absolute

2. Fee simple subject to condition subsequent: fee simple that does not automatically terminate but may be cut short or divested at the grantor’s election when a stated condition happens.

o Right of entry: future interest in the fee. DIVESTS AT RE-ENTRY.o Classic language (CONDITION): “to the school district, but if the land is used for

non-school purposes, the grantor shall have right of re-entry” Provided, however, but if, and on condition that

Transfer of future interest (possibility of reverter/right of entry): in most states, the possibility of reverter and right of entry are transferrable inter vivos (transfer of property made during transferor’s lifetime) (not in Mahrenholz). However, some states follow common law rule that future interests are not transferrable except to the owner of the possessory fee (release)

o Common law allows release and not transfer bc it prefers fee simple absolute. We want land to be used to the best advantage and release is more likely to get it back to fee simple absolute.

o Restrictions: a grantor can restrict the use but not the alienation of his conveyed property. Mountain brow lodge

Adverse possession: o Determinable: SOL starts running on the possibility of reverter as soon as the

determinable fee ends if the reversionary interest holder doesn’t claim his right to ownership of

determinable when breached w/in SOL period, he is barred from claiming the property and adverse possessor owns prop in fee simple absolute.

o Condition Subsequent: SOL does not begin to run until holder of right of entry exercises his right

the SOL of fee simple condition subsequent isn’t tolled until the reversionary interest holder brings action for right of entry

10

Page 11: Property Final Outline

Mahrenholz v County Board of School Trustee (1981)Huttons conveyed defeasible fee to school: “land so be used for school purposes only; otherwise to revert to grantors herein. Mahrenholz sought to quite title, but it was unclear what type of defeasible fee was granted.1941: Huttonschool district school purposes only, otherwise to revert: 1.5 acres1941: Huttonjacqmains: 38.5 acres + reversionary interest in school’s land1959: Jacqmainsmahrenholz: 38.5 acres + reversionary interest in school’s landIL statute renders possibility of reverter and right of re-entry inalienable by will or inter-vivos conveyance.1973: school changes use for storgage purposes1977: Hutton son, Harry, coveys his interest in property to mahrenholz (huttons dead and harry is ony living heir)1977: harry disclaims his interest to the school board

Mahren wants determinable: Harry Mahrenholz: future interest.

Fee Simple Determinable: If breach, H had a fee simple absolute to confer.Fee Simple Subject to Condition Subsequent. Even if breach, H can’t convey the property without first re-entering, and cannot convey his right of entry.

School wants condition sub: Harry School District: release.

Fee Simple Determinable: H already sold the property so can’t release.Fee Simple Subject to Condition Subsequent. H could still release the school district. The statute only prevents alienation, not release.

Court finds a fee simple determinable due to inclusion of the word “only” and clause “otherwise to revert to grantors” triggers mandatory return rather than permissive

so when hutton dies, harry has reversionary interest—so when he conveys his reversionary interest to mahrenholz, there is nothing left to give to the school board.

IL statute is bad bc its inefficient—as soon as person with reversionary interest dies, it descends to heirs. Therefore, if school board wants to buy reversionary interest, there are many problems

o Must find all heris that own reversionary interest, must get all heirs to agree (power of holdout)

o Transaction costs too high for reversionary interest which is prob worthless

Arguments for fee simple subject to condition subsequent w/ right of re-entry ****grantor’s intent Where ambiguous language has been used, courts have expressed constructional preference for

condition subsequent bc If school board breaches condition, holders of reversionary interest are NOT aware they own property and WE WANT CERTAINTY in property

o Possessor continues to own until hold of right of re-entry acts to retake property The word revert doesn’t overcome presumption in favor of condition subsequent The word only is not a word of duration

Arguments for fee simple determinable Grantor’s intent Words of duration (only, revert) held fee simple determinable and set a presumption of clear

grantor’s intent for fee simple determinable

11

Page 12: Property Final Outline

Restraints on alienation

Mountain Brow Lodge…Odd Fellows v Toscano (1967)Decedants gave gift deed to the lodge with both use and alienation restriction (use and benefit of property to the Lodge only). The lodge sought to quite title to the property.Court enforced the use restriction (condition subsequent), but not the alienation condition. ***courts automatically strike alienation restriction.

Conditions restraining alienation are void, but use restrictions are okay even though they hamper and often, completely impede, alienation

o Alienation restrictions creates unmarketable properties; this is not efficient for a market bc land will be so expensive to buy

o High transaction costs (locating holders of reversionary interest is expensive and time consuming)

Unfortunately, the use restriction is that lodge has to use the land, therefore, lodge cant sell ito They might be able to lease the property, but the grantors might object bc such a lease

would not benefit the lodge only Arguments against restriction

o Land restrictions create unmarketable properties and this is not efficient for market bc land will be too expensive for those buying

o Distinguish mountain brow: use restriction there was for public interest organization versus a private individual

This distinguishable fact is outcome determinativeo When bank sees restriction on alienation, cant get mortgage or line of credit bc bank

cannot foreclose on house (this constitutes a sale) Arguments for restriction

o Argue the land use clause upheld in mountain browo Non profits benefit community and voiding use restrictions would deter people from

granting to public organizationso If we allow restrictions to non-profits (which does not happen often), this will not create

damaging effect on grantors (most land is granted to private people)o If alienation restrictions were allowed to be enforced, the lodge could reach an

agreement with the grantor to sell and release the property to a third-party. Furthermore, there are market incentives on both the lodge and the grantor to sell their interest to the third-party. Bilateral monopoly.

Counter: the number of heirs might be too large to locate or to reach agreement with (hold out problem). This would be costly and inefficient

FUTURE INTERESTS

A. FUTURE INTERESTS IN THE TRANSFEROR (RETAINED BY GRANTOR) 1. Reversion: reversion is future interest created when the grantor conveys a lesser estate that

he originally owned. Unlike poss of reveter or right of entry, Its Freely alienable during life and may be devised or inherited

o i.e. if o has fee simple absolute and transfers life etate to a, then o retains reversion

2. Possibility of reverter: future interest remaining in grantor or his heirs when fee simple determinable is created.

12

Page 13: Property Final Outline

o Common law did not permit transfer of possibility of reverter inter vivos or by will, but only by inheritance

o Today, most states permit possibility of reverter to be alienated inter vivos, devised or inherited

3. Right of entry: future interest remaining in grantor or his heirs when an estate is created subject to condition subsequent

o Common law did not permit transfer of possibility of reverter inter vivos or by will, but only by inheritance

Jurisdictions today are split over whether to follow common law rule or to permit free alienation

B. FUTURE INTERESTS IN TRANSFEREES/3rd PARTIES (CREATED IN GRANTEES)-

1. is the future interest retained by the grantor? No. move to 2.2. is future interest contingent or vested?

a. Its contingent if the condition is on the same side of the comma or period (to a for life then if b lives to age 21) OR

b. There is no current identifiable taker (no person you can point to who is remainderman)

3. If no condition precedent or no current identifiable taker, its vested a. Its vested if the condition is after the comma or period)

4. If vested, What type of vested remainder?a. Indeafsibly vested: nothing can happen to take property right awayb. Vested subject to open:

i. Remainder given to class of people: at least one of who is presently existing and entitled to possession as soon as preceding estate expire, but which is capable of expansion to include unknown people

1. Class is left open for entry of new membersc. Vested subject to compelte divestment:

i. Where is the condition placed?1. If condition is after the comma or period, its vested subject to

complete divestment.a. To a for life, remainder to b, if b not 21 at a’s death, then to c

2. If on the same side of comma or period, its contingent to condition precedant

a. To a for life, then b if b is 21 at a’s deathd. RULE: IF YOU HAVE 2 ALTERNATE REMAINDERS AND THE FIRST

ONE IS CONTINGENT, THE ALTERNATE REMAINDER IS ALSO CONTINGENT (CALLED ALTERNATE CONTINGENT REMAINDER)

e. RULE: IF YOU HAVE 2 ALTERNATE REMAINDERS AND THE FIRST IS VESTED SUBJECT TO DIVESTMENT, THE 2ND IS EXECUTORY

f. Executory interest occurs in 2 situationsi. 1) if 2 alternative remainders and 1st is vested subject to divestment, 2nd

is executoryii. 2) grantor creates a defeasible fee (determinable or condition

subsequent), instead of retaining reversionary interest, they give the

13

Page 14: Property Final Outline

right to re-enter or possibility of reverter to 3rd party, 3rd part’s interest is executory

Remainders v. executory interestDifference btwn remainders and executory interests is taking possession as soon as the prior estate ends or divesting the prior estate. A remainder is future interests that waits politely until the termination of the preceding possessory estate, at which time the remainder moves into possession if it is then vested.

An executory interest is a future interest in a transferee that can take effect only by divesting another interest

Remainders

Vested remainder: remainder is vested if it is:1) given to an ascertained person AND 2) not subject to a condition precedent.

Where an instrument is ambiguous, courts construe in favor or vested remainder. See Kroos

1. Indefeasibly vested: the remainder is certain to become possessory in the future. Nothing will prevent possession from happening and once possession occurs, it will last forever.

a. To a for life, remainder to b: b. to a for life, then to b for life, then to c if c survives a

i. a has life estate; b has indefeasible vested remainder in fee simpleii. c has contingent remainder

iii. o retains reversionary interest if c dies before ac. to a for life and in the event of a’s death, to b and heirs

i. b has indefeasibly vested remainder and at a’s death, b has fee simple absoluted. to a for life, then to b for life, then to c and her heirs

i. c have indefeasibly vested remaindersii. b’s interest is only a indeafsibly vested remainder in life estate.

2. Vested subject to open: remainder created in a class of persons is vested if one member of the class is ascertained and there is no condition precedent

a. To a for life, remainder at a’s death to a’s childreni. Right now a has 5 year old child. A’s child is entitled to take, but if a has more

children, she wont get entire propertyb. to a for life, then to a’s children who shall reach 21.

i. If at time of creation, a has no children over 21, the remainder is contingentii. If A has one child 25 years old. The condition is satisfied as to a, but there could

be other children. If there are other children who might qualify, a takes possession and may have to share with other children.

c. To a for life, then to a’s children, but if at a’s death he has no children, then to bi. A is alive and has no children: a’s children have contingent remainder and b has

execturoy interestii. If a has child and a is still alive: vested subject to open and vested subject to

complete divestment3. Vested subject to complete divestment: if the first future interest created is a vested

remainder in fee simple, the 2nd future interest in a transferee will be a divesting executory interest.

a. To a for life, then b and her heirs, but if b does not survive a to c and heirs

14

Page 15: Property Final Outline

i. Grant to b is in separate clause and the condition follows so its vested subject to complete divestment

ii. C has executory interestb. To a for life, then to b. if a is survived by any children, then to surviving children

i. A is alive and has 2 childrenii. B is vested subject to complete divestment

iii. A’s children has executory interestRULE: IF YOU HAVE 2 ALTERNATE REMAINDERS AND THE FIRST IS VESTED SUBJECT TO DIVESTMENT, THE 2ND IS EXECUTORYRULE: IF YOU HAVE 2 ALTERNATE REMAINDERS AND THE FIRST ONE IS CONTINGENT, THE ALTERNATE REMAINDER IS ALSO CONTINGENT (CALLED ALTERNATE CONTINGENT REMAINDER)

Contingent remainder: 1) given to an unascertained person OR 2) made contingent upon some event occurring other than the natural termination of the preceding estate.

Rule: If the 1st future interest created is a contingent remainder in fee simple, the 2nd future interest in a transferee will be an alternate contingent remainder:

o To a for life, then b and heirs if b survives a To a for life, then to b for life, then to c and her heirs if c survives a and b

o C has contingent remainder bc of the condition To a an b for their joint lives, then to the survivor in fee simple

o No current identifiable taker, thus its contingent. o We don’t know who the survivor is, thus, while both are alive, the remainder is

contingent. To a for life, then to a’s children who shall reach 21

o A has 1 child 17 years old Its contingent remainder. Once child turns 21, turns into vested subject open.

To a for life, then to a’s children who survive a, but if none survive a, to b.o A is alive and has 2 children

A’s children have contingent remainder B has an alternate contingent remainder

RULE: IF YOU HAVE 2 ALTERNATE REMAINDERS AND THE FIRST ONE IS CONTINGENT, THE ALTERNATE REMAINDER IS ALSO CONTINGENT (CALLED ALTERNATE CONTINGENT REMAINDER)

Ambiguity in Grant; presumption for vested remainder over contigentIn Re KroosHusband (herman) life estate to his wife with remainder in fee simple absolute to his kids, but if either of the children die before the wife, to the grandkids. The wife, elsie, has 2 children, John and Florence. Florence dies before elise leaving NO CHILDREN. Florence’s husband arguing that Florence had indeafisbly vested remainder that can be divested only if she predecease her mom or has issue. John, the bro, argues its a contingent remainder on surviving elise—she didn’t survive, she gets nothing.Court finds vested remainder and grantor was willing to have the estate divested upon 2 conditions: florence or john had to die before her mother and child dying must leave kids.

Court favors a presumption for vested interest bc it enables property to be freely transferred at the earliest possible date

15

Page 16: Property Final Outline

Grantor’s intent: OMISSION OF ANY WORDS OF SURVIVORSHIP TO INDICATE AN INTENT THAT FLORENCE’S BRO WAS TO TAKE IF FLORENCE DIES W/O CHILDREN SHOWS GRANTOR’S INTENT TO GIVE HIS DAUGHTER VESTED REMAINDER AND NOT CONTINGENT REMAINDER

HOW TO DRAFTER BETTER if grantor wants her to have property if she survives her mom and if she doesn’t survive her mom, then to her children:

Vested subject to complete divestment and executory interest in children if any, and if no children, then to her brother OR

Make remainder contingent on surviving mom, if she doesn’t, to children, if no children, to john

Court goes against presumption for vested remainder and decides for contigentBrown v Sacrison (1974)To “ada sacrison, a life estate, with remainder over at her death to my grandsons, if either of them be dead, then all the other. Frank dies before life tenant. Frank browning’s wife (p) sued Robert Browning (d) claiming her husband had an indefeasibly vested interest bc the language might be read as satisfying the condition at the time of Kate Webb’s death (grantor). Since frank was alive when Katie Webb died, condition was satisfied and frank had indefeasibly vested interest. Robert argues that the condition meant Ada Sacrison’s death (the life tenant).Court holds it was contingent remainder at Ada’s death (life tenant)

Court doesn’t adopt Presumption of vesting when unclear; court wants contingent remainder. Why?

o Vested remainder created bad tax consequences; construing as a contingent saves taxes for families

i.e. frank dies with child. If he has vested remainder and wants to pass property to his child at his death, there is an estate tax and value of vested remainder is included in his taxable estate

if contingent, there is no taxes due on contingent remainder. When life tenant dies, estate would down to child and bypass level of estate taxes

Grantor’s intent o She didn’t want life tenants husband (clyde) to have any interest in property

If frank dies before his father and estate is vested, his father as heir would get property

Argue for contigent remainder over vested remaindero “surviving” means to survive life tenant, which creates a contigent remainder contigent

on surviving the life estateo use brown v sacrison, holding unclear cvneyance was a contigent remainder (grantor’s

intent)o presumption of bested remainder when unclear has been substantially weakened: courts

don’t automatically apply presumption anymore for reasons including favorable tax treatment of contigent remainder

argue for vested remainder over contigent?o X recently lost child, thur surviving meant to survive grantor and not life estateo In brown, grantor didn’t want son in law to get land. Here, grantor’s intent is to benefit

his child and her grandchildreno *****when somebody creates contigent remainder, they usually create an alternative

contigent remainder*****What she could have done to draft better grant?

16

Page 17: Property Final Outline

To ada in life estate, with remainder to my grandsons, if either fails to survive me, then all to other

Even better, if both grandsons dies before grantor: to francis and Robert provided they survive ada. If one fails to survive ada, then all to the other. If both predecease ada, then property shall pass to their decedents shared equally.

RULE AGAINST PERPETUITIES 1) A contingent interest or vested interest subject to open 2) must vest or terminate3) no later than 21 years after the death of validating life (person alive at the creation of the interest)

Reasoning: control for too long inhibits productive use and alienability and clouds titles. However, the grantor may, knowing his children, not trust the children with the estate

STEPS1) Classify the interest

a. If remainder is retained by grantor (reversion, possibility of reverter and right of entry,) NOT subject to rule

2) Is future in interest vested subject to open, contingent or executory?a. IF SO, RULE APPLIES

3) Determine when perpetuities period begins (when the grant becomes effective)a. Inter vivos grant: perpetuities period begins when grant is madeb. Devised in will: period begins when testator who makes grant dies

4) If the interest is vested subject to open, the only question is when does the class close and all internal preconditions imposed on class members are satisfied by all class members-

a. “ALL OR NOTHING RULE”: class gift is not vested under RAP until ALL class members are identified and all conditions precedant have been satsiefeid by every member

b. Common law rule: open class cannot be validating life5) List all the individuals who ARE ALIVE when grant is made and whose actions during life or

by death can impact future interest createda. Lives in being expressly named in or described by grantb. Lives in being implied from grant language (who are not identified in grant but who are

relevant lives in being (parents of a class of grandchildren)6) Give birth to any after born persons7) Kill all the lives in being (at creation) at some future point and add 21 years from this date

a. THIS IS THE RAP LINE8) Ask is there a possibility that the interest will vest beyond the RAP line

a. If it possible that interest may vest after this date, its invalidb. If interest must vest, if at all, within rap line, its valid

Leslie RAP problemsT’s will creates a trust and directs trustee to pay income to my wife, w, for life then distribute the principal among my children that survive me. T is is survived by w and 3 children, a, b and c.

W has life estate A, b, c have indefeasibly vested remainder bc they all survived t

o There interests are vested No rap issue

17

Page 18: Property Final Outline

T’s will says to my wife, w, for life, then distribute the principal to my children that survive w. T is is survived by w and 3 children, a, b and c.

1) A,b,c have contingent remainders bc there is contingent precedent to vesting; they must survive w to be entitled to take.

2) Interest will vest or fail to vest at w’s death or if a, b, c predecease w 3) the contingency will be resolved by w’s life or by a,b,c as a class 4) were any of the those listed alive at the creation of the interest (if class is listed: asks what

the entire class alive at creation of interest)o yes. W was alive and a,b,c were all alive.

5) Interest is valid

t’s will creates a trust and directs trustee to pay income to my children for their lives, remainder to my grandchildren in equal parts. T is survived by his children a,b,c and b’s child, q.

a,b,c have life estate and q has remainder that is vested subject to open (bc a and c may have children, and b may have more children)

this remainder is valid bc class of t’s children: a,b,c are validating lives even though a,b,c could have more children, the class of grandchildren will close when t’s last

child dieso thus the grandchildren’s remainder will be completely vested at time

t creates trsut executing deed during his life (inter vivos) instead of thru his will. t directs trustee to pay income to my children for their lives, remainder to my grandchildren in equal parts. T is survived by his children a,b,c and b’s child, q.

remainder is invalid t’s children cannot be validating lives bc t could have child after trust is created interest in grandchildren might vest at the after born child’s death, which could be more than 21

years after the deaths of all those who were alive at creation of interest

T’s will creates a trust, and directs the trustee to pay the income to my nieces and nephews for their lives, remainder to my grand-nieces and grand-nephews in equal parts. T is survived only by his siblings, A and B, and B=s child, Q.

remainder to t’s grand nieces and nephews is contingent bc none of them have been born at t’s death

the contingent remainder is invalido last moment the interest could vest or fail is when t’s siblings and all of t’s niece and

nephews dieo bc a, b and q are all alive, we presume any of them could have another childo thus, its possible that the longest lived niece or nephew might be person who was not

alive at t’s death (which was when interest was created)

T’s will creates a trust that provides: To A for life, then to A’s children, but if none of A’s children survive A, then to B. T is survived by A, and A=s children, Q & R, and B.

q and r haved remainder vested subject to open (bc a could have more children) and subject complete divestment (bc none of a’s children survive a, then a’s children’s estate divested by b

o b has executory interest all interests are valid

o a’s life estate is possessory

18

Page 19: Property Final Outline

o a’s children’s remainder vests immediately on a’s death so a is the validating life (a was alive at creation)

o b’s executory interest will vest if at all, at a’s death (a is validating life)

[In his will] T bequeaths a fund in trust to pay the income to my son for life; then to my son’s widow, if any, for life; then to pay the principal to my son’s children, but if no child of my son is alive at the death of the survivor of my son and his widow, then pay the principal to the American Red Cross.

Son has life estate: o Interest is valid; vests immediately

Widow has contingent life estate (bc drafter did not identify the widow by name, therefore we wont know the widow until the son dies- his current wife could die and he could remarry)

o Interest is valid bc it will vest, if at all, at sons death (son is validating life) son’s children: remainder is vested subject to open (bc the son could have more children) and

complete divestment (bc all children, if any, could predecease the son and his widow)o remainder is valid bc class will close at son’s death (although remainder will no

longer be vested subject to open, it will continue to be subject to compete divestment, but subject to divestment ARE NOT SUBJECT TO RAP)

American red cross has executory interesto Interest is invalid bc it will vest if at all, at death of widow, who could be an after

born person who outlives all lives in being by more than 21 years

To a for life, and on a’s death to a’s children for their lives and upon the death of a and a’s children, to the a’s grandchildren. A and b survive t.

o a’s kids cant be validating life because its an open class we will know when the last of a’s kids dies- then we will know when it vests

o not valid: there is no validating lifeo ****it does not worko how to make it work: ***name a’s children- create validating life by naming them

All hypos for RAPto a for life, remainder to b if b graduates from law school

Interest is contingent Contingency resolves at last moment of time, when B will dies plus 21 years B validates his own remainder; thus, b is validating life

o B was alive at time of grant

To a for life then a’s first child who reaches 21 Interest is contingent Contingency resolved when a’s first child turns 21 or no child reaches 21

o It will happen by time a dies A is validating life and a was alive at time of grant

A for life, then to a’s first child to reach 25. A has 1 child, q, 10 Contingent remainder After interest is created, a can have another child, r. then a and q dies and r, who is now 1, will

turn 25 in 24 years A and q were alive at creation of interest therefore its possible the future interest might not vest

during their lives

19

Page 20: Property Final Outline

to a for life, then to a’s children for life, upon the death of last surviving children of a, to a’s grandchildren. A is 80 yr old with children b and c

B and C CANNOT BE VALIDATING LIVES bc the class is still open Remainder to a’s grandchildren is void

To a for life, then to b if b attains the age of 30. B is now 2 years old B has contingent remainder B is the validating life bc b is alive at conveyance and either interest will fail to vest (if b dies

before 30) or will vest when b turns 30

To a for life, then to a’s children for their lives, then b if b is then alive, and if b is not alive, to b’s heirs. A has no children at time of conveyance.

B’s interest is contingent B is validating life bc b is alive when interest is created and b can predeace a’s children and gift

would fail to vest or he will live to time of possession and it will vest B’s children have alternate contingent remainder bc we don’t know who b’s heirs are until he

dieso B is their validating life bc contingent remainder will vest at b’s death or fail to vest

Prof declaes she holds in trust 1k for “all members of my present prop class who are admitted to the bar.

This is valid Interest is vested subject to open Class is alive at creation of interest; class is validating life Last student passes bar or last student dies w/o passing the bar is when interest will vest of fail

to vestProf declares, 1k to the first child of A admitted to the bar?

Invalid. A’s children cannot be validating life bc entire class is possibly not alive A cannot be validating life bc a could have more than 1 child and more than 21 years after a’s

death, 1 of a’s children could pass the bar

To a for life, then to a’s widow, if any for life, then to a’s issue the living. At a’s death, we will know if there is a widow or no A’s issue: contigent remainder NO validating life:

o if we assume the widow is alive the creation of interest its valid BUTo shortly after conveyance, a’s gets divorced. 18 years later, marries woman not alive at

creation of interest a has child. A then dies. At this point, we only have a’s widow and after born

child. This would be valid if A’s widow was named in the grant

Modern approaches to RAP NY uses common law RAP with few saving rules

20

Page 21: Property Final Outline

o Ny rule: If poor drafting cause something to invalidate rule, court will assume that it referred to a life in being at creation of interest

We will assume any widow he will marry is alive after conveyanceo Ny rule: they will pencil in 21 years old rather than 25o NY has statute that provides that woman 55 or over is presumed infertile

Wait and see approach: future interest that is valid under common law version of RAP remains valid

o Future interest that is void is not struck down immediately. Instead, one waits to see if the future interest does in fact RAP vest within perpetuities period

Uniform Statutory Rule Against Perpetuities (USRAP) Approach: future interest that is valid under common law version of RAP remains valid

o If future is void under common law version of RAP, one must wait 90 years instead of 21 years(measured from when grant is made) to determine if future interest does in fact RAP vest within the 90 year period

Cy pres: when conveyance violates common law RAP, some jurisdictions allow courts to reform conveyance so it doesn’t violate rule

o Judicial tool for reforming conveyance in manner consistent with grantor’s intent

21

Page 22: Property Final Outline

CO-OWNERSHIP AND MARITAL INTEREST (COMMON LAW CONCURRENT INTERESTS)- to a for life, remainder to b and c

TENANCY IN COMMON: separate but undivided interest which is descendible and may be conveyed by deed or will.

No survivorship rights in tenants in common Each tenant in common owns an undivided share of whole Tenancy in common occurs when severance ends joint tenancy or divorce ends tenancy by

entirety Absent clear language expressing intent to create joint tenancy, presumption for TIC Creditors: tenant’s creditor’s may touch his interest Does not avoid probate: half of share goes to probate

JOINT TENANCY: undivided whole interest with survivorship rights (property passed to surviving tenants).

Joint tenants conveyance termintes joint tenancy and changes interest to TIC. NOW, joint tenant can convey to herself and make the conversion. Riddle

Creditors: sometimes creditors may touch tenant’s survivorship right Sawada Common law favored joint tenancies over tenancies in common, but today the situation is

reversedo Presumption in favor of TIC unless intent to create joint tenancy is expressly

declared Avoids probate Hypo: o conveys blackacre to a, b, c as joint tenants. Subsequently, a conveys his interest to d.

b then dies intestate leaving h as heir. What is state of title?o When a conveys interest to d, a has severed his right of survivorship but it does not

sever right of survivorship of b and co Blackacre is now owned by d and c

d owns 1/3 and c owns 2/3 (c is survivor btwn b and c) hypo: t devises blackacre to a and b as joint tenants for their joint lives, remainder to survivor.

A conveys her interest to q. a then dies.o B argues: a transferred his life estate to q but life estate is measured by a’s life (per

autrie view). When a dies, b gets the contingent remaindero Q argues: title was right of survivorship. When a conveyed his interest, it severed

survivorship and turned estate in TIC Q owns a’s interest.

Unilateral Severance of Joint Tenancy Converts to TIC

22

Page 23: Property Final Outline

Riddle v Harmon (1980)Mr and Mrs. Riddle Purchased the land as joint tenants. Mrs. Riddle didn’t want prop to pass to her husband upon her death. Mrs. Riddle granted to herself by deed an undivided ½ interest in property severing joitn tenancy and making it tenancy in common.Court held it was a tenancy in common and that she could validy sever the joint tenancy by conveyace to herself as a joint to tenant to herself as tenant in common w/o use of strawman.

Established law at the time said you couldn’t convey to yourself but there was no purpose for this rule (so many ways of getting around this indirectly—strawman or conveying her interest to her daughter to destroy survivorship right)

o Strawman: a to b back to a, created TIC Argument against conveying interest to yourself and severing joint tenancy:

o Rule that joint tenancy can be severed by conveying an interest to themselves encourages fraud and deprives other joint tenant of prop interest w/o notice

o With strawman, there is a level of legitimacy Counterargument:

o joint tenant has notice bc when they enter into joint tenancy, they know either party can unilaterally sever survivorship

Harms v. SpragueBrothers took property as joint tenancy. One of brothers executed mortgage on his interest of the property in favor of 3rd party w/ o his brother’s notice. Mortgage brother died and in his will, he devised his interest in property to 3rd party.Court held 1) there was no severance and 2) brother who is alive owns the property entirely free of the mortgage. Mortgage burdened only the dead bro and dead bro’s interest died with him leaving an unencumbered interest for other bro with the mortgage dying too.

Arguments for mortgage severs joint tenancy:o Mortgage effects transfer of title, makes it subject to equitable right of mortgagor to

reclaim title by paying off the loan secured by mortgageo When mortgage was given, 4 unities were destroyed and survivorship converted to TIC

Argument for mortgage doesn’t sever joint tenancyo Mortgage is just a lien on property and therefore doesn’t severo Although bro gave mortgage, it does not sever survivorship bc its just an alteration of

the title and does not sever the joint tenancy. Once bro dies, I get the prop

TENANCY BY THE ENTIRETY: form of joint ownership available only to husband and wife. 4 unities like in joint tenancy must exist plus 5th: marriage. The couple may only make a conveyance together and cannot defeat the other’s survivorship right

Can be terminated only by divorce, death of 1 spouse or agreement by both spouses Intent to create tenancy by entirety must be clearly expressed Creditors: most states says creditor of individual spouses cannot reach tenancy by entirety

propertyo Some states allow creditors to execute on right of survivorship of debtor spouse onlyo Other states permit creditors to sell debtor’s spouse interest subject to non-debtor

spouse right of survivorship Hypo: a and b plan to be married. 2 weeks before ceremony, they buy house and take title as a

and b as tenants by entirety. Years after marriage, a moves out and conveys his interest to his brother, c. c brings action to partition property.

o A and b taking title as tenancy by entirery bf marriage is not valid

23

Page 24: Property Final Outline

Modern court might recognize their marriage 2 weeks beforeo We can assume its TIC or joint tenancy, although it doesn’t mattero If survivorship, conveyance destroys joint tenancy and it becomes TIC

See Sawada v Endo

Joint Tenancy Bank AccountsDuring the lifetime of the parties, there is a rebuttable presumption that the joint account belongs to the parties in proportion to their net contributions. Either party can withdraw all the money, but the depositor has a claim for the deposited sum.

Creditors: Creditors can only attach a debtor’s assets, but they may touch the entire joint account (unless the non-debtor joint tenant can prove how much he deposited) because the parties are in a better position to declare the source of the money.

Surviving Tenant: Takes the whole sum unless there is clear and convincing evidence that a convenience account was intended.

Rights and Obligations of Concurrent Owners (and remedies)

Remedy 1: PARTITION

If parties to a joint tenancy or TIC cannot agree on a division of property, they can bring an equitable action of partition (not tenants by entirety)

Partition copes with:o 1) holdouts, who may cause the division not the reflect their interest and o 2) large numbers of cotenants

A. PARTITION BY SALE: only when:1) the physical attributes of land make partition in kind impracticable or inequitable AND 2) the owner’s interests are better promoted by the sale. Delfino ***part seeking sale must prove there interests are more harmed than other party court appoints referee to hold an auction sale—they would split the proceeds proportionately to

tenant’s share

B. Parition in Kind: physical division of the property is the preferred method. Presumption for partition in kind. Delfino Court will order physical partition unless the other party can prove:

o 1) physical partition impossible, impractical (Not practical to divide single family homeOR

o 2) physical partition is not in the best interests of all the parties. Evidence here includes economic costs or gain involved and the subjective value

imposed on tenant in possession by ordering partition by sale

Delfino v Vealencis (1980)

24

Page 25: Property Final Outline

Delinfos (99/144) are TIC with Helen Vealencis (45/144). Helen lived on a portion of the property and operated a garbage business from it. Delfinos wanted to develop the property into single family homes. Delfinos seek partition by sale (sell to developers). Helen seeks partition in kind (maintain her home and garbage bus)Court holds it was not in best interest of all parties (Helen) to sell the entire property thus Court grants parition in kind: 1) the property may practicably be physically divided and 2) the interests of all owners are better promoted by partition in kind.

Delfino wants partition by sale bc he thinks helen’s use of the land devalues property—people don’t value land that’s nxt to garbage dump—this way, delfino gets more $

o Delfino must prove they were being harmed and his interests would be harmed more than helen’s interest

Helen wants partition in kind bc its her livelihood, She has an emotional attachment to the property

o attaches subjective value to the property, so the property is worth more her than its market value

o Favorable presumption for parition in kind by the court: o how does court determine if parties subjective value is really worth that much

Looks to surrounding circumstances (how long has she been operating bus, how long lived there, how long family lived there)

Johnson v Hendrickson: Court relies entirely on the conclusion that the division of the farm into parcels would materially depreciate its value.

Options when concurrent owners are feuding ****both TIC own property and have equal righs to occupy so there are no grounds to

kick other party out****this is not an optiono settlement: avoid time, cost and stress of litigation

if there is partition in kind, land is worth 500k, therefore, she can settle for btwn 500k and 750k (bro’s share after entire prop sold)

negotiate with each other (I can stay in the house or you stay)o default judgment against sis: allow property to be sold and go and bid for property

(will have to bid over 1.5mm)o argument for partition in kind

presumption for in kind. Delfino best interests of parties and dividing the land is practical (in delfino, dividing

land was practical, here is much easier) grantors intent: honor grantor intent to keep house in family

o argument against partition in kind cannot grant in kind bc its single family home and impractical to divide court should grant sale bc key factor: whole is greater than sum of its parts.

Johnson v Hendrickson distinguish sentimental value of Helen where she lived and worked on property

forever. Sister has options unlike Helen- she is rich and can outbid developer Subjective attachment only applies when mv is lower than subjective

attacher values ito Sis subjective attachment not supported by objective facts

25

Page 26: Property Final Outline

o Sis claim for subjective attachment is only to impair bro’s negotiating leverage

o Argument for parition sale If person who wants to live in house and attaches high subjective value must go

and have enough money to bid for house

WHAT ARE OTHER Available remedies FOR CO-OWNERSHIP other than parition

2) Accounting action: each cotenant is entitled to pro rata share of rents received from 3rd party using land. The cotenant who is received rent from 3rd party is obligated to account to his co-tenants for those rents.

3) Exclusive possession by co-owner creates right to rental value or ouster: each co-enant has right to possess all of the property, exclusive possession by co-owner is presumptively valid

A. Rental Value: o 1. Majority rule: co-tenant has no liability for his share of the rental value unless 1)

co-tenants were ousted or 2) co-tenant in position owes fidicuary duty to other co-tenants or 3) co-tenant in position has agreed to pay rent

Rational for majority rule: 1) grantor’s intent 2) encourages parition 3) avoids ficticious actions of co-tenant out of possession making untrue claims

o Ouster: ouster occurs when cotenant in possession either 1) actually prevents or bars physical entry by a co-tenant (can occur by changing locks) or 2) denies the co-tenants claim to title (express statements denying the co-tenant out of possession has any valid claim of ownership to such property)

Ousted cotenant entitled to recover half of reasonable rental value

no ouster: If cotenant simply demands payment or demanding cotenant leave****must actively make a demand and act on demand to use prop

Adverse possession: ouster puts cotenant out of possession on notice that co-tenant in possession is repudiating relationship

o Minority Rule: co-tenant is liable to co-tenant out of possesson for their share fo the fair rental value of the occupied premises. Cohen v. Cohen

No need to show ouster or agreement to pay rent or presence of fiduciary obligation

Rational: no need for litigation, co-tenant out of poss entitled to half rental $What rule is better: Spiller or Cohen (minority rule)

Cohen: less complicated, less litigated, just have to pay rent, don’t need to litigate ouster (more litigation in Spiller as to whether there is ouster)

Spiller: encourages partition (not always optimal), captures attention of most parties when they own prop as cotenant, can contract around rule (commercial prop)

26

Page 27: Property Final Outline

Spiller v Mackereth (1976)S and M Own building as TIC. Property was leased to 3rd party and co-tenants were sharing rent. Lessee leaves and s exercises his right to exclusive possession. Mackereth sends letter: “either vacate half the building or pay rent.” Mackreth sues spiller for value of his possession of the premises (rent.)Court holds no ouster, therefore no rent. S had neither denied that M was the owner of the building nor prevented M from actually moving in and taking possession. M’s demand that S vacate or pay rent was insufficient to trigger ouster

What should mackereth have done? Establish ouster by proving she 1) trying to physically enter or occupy building and was

prevented from doing so OR 2) trying to get a new tenant Partition or threaten partition OR Accounting

Argue for co-tenant in possession: adverse possesson argues adverse possession: open, notorious and owner was on notice. No need for ouster

o but if I did oust, then there is notice that I was hostile We want certainty of ownership he was hostile and proceeded under claim of right.

Argue against AP for co-tenant out of possession: Need to oust before adverse possession can begin Even if you did oust, denial of entry is not enough; you need to be more hostile, open and

notorious warranting notice that co-tenant in possession is possessing adversely

4) Contribution action: general rule that all cotenants obligated to pay their portion of mortgage, tax, assessments that could give rise to a lien against the property if unpaid

If 1 cotenant pays more than their share, he can recover excess in contribution action5) Liability for repair and improvement: majority rule is that the cotenant who pays for repairs or improvements is not entitled to contribution from cotenants

Repairs: was it necessary to maintain the house if yes, then it’s a repair: non-repair tenant wants repairs to be classified as improvements

o If something is classified as a repair, entitled to credit for cost of repair even if it doesn’t increase value of property

Rational: policy dtermination to give incentive to make repairs + maintain propo Cotenant in possession can be paid back by taking out repairs from rent and if cotenant

out of possession sues, in possession entitled to reasonable value of repairso If cotenant makes repairs and moves to partition of sale, repairs will be added to their

portion of sale priceo i.e. if a makes 15k repairs and moves for partition. House sells for 60k. court will chop

of 15k for repairs and divide 45k by the cotenants. A gets her 15k repairs and her share

Improvements: repair tenant wants improvements to be classified as repairso cotenant is entitled ONLY to valued added to house by improvements o House worth 60k. a makes 30k improvements and value of house increases by 15 k.

moves for partition. A only entitled to 15k off the top and not 30k she put in.

Swartzbaugh v Sampson (1936)

27

Page 28: Property Final Outline

Husband and wife own property in joint tenancy. Husband leases a piece of prop to Sampson, a boxing promoter. The property previously used for walnuts, was used to construct a boxing pavilion. Wife did not join in the lease and she objected to the boxing pavilion.Wife wants the court to find the lease void.Court finds lease valid and effective. The lease by a single joint tenant to a 3rd party is not a nullity but is a valid k. Husband can convey all of his interest to 3rd party, therefore he can assign part of his interest.

If land was held in tenants by entirety, wife has much better case cuz no spouse can unilaterally do anything w/o consent of other

What was wife’s legal remedies: o Outster: go to boxing pavilion and demad she be let into possession and if he bars

entry, outster established This would give her half the rent of reasonable value of land

o Accounting action: demand and receive ½ rents received by her husban from sampsono Parition of the leasehold: would probably result in parition by sale (bc it would be

impossible to physically divide pavilion) Proceeds of sale would be used first to reimburse Sampson for his improvements

and the balance would be dividedo Wait for husband’s death: once he dies, it would terminate the lease bc Sampson

leased john’s interest and they owned land as joint tenants, his interests would expire at death

o Seek injunction (beforehand): show court, that her right to use land will be prejudiced (someone cuts down her walnut trees)

Waste: show person is devaluing property interest

MARITAL INTERESTS- 2 times of marital property systems1) Majority: common law: separate property: tenant in common or joint tenant alone can convey his undivided share to 3rd party, can change the form of estate, has the right to parition (all of this is unavailable for community property state)

Equitable distribution: presumption that all property is divided 50/50 but court can divide property as it sees fit

2) Minority: Community property: couple is marital partnership and all property is shared equally During marriage, all income and everything bought belongs to community During life, neither party can unilaterally convey property At death each spouse has right to devise their ½ interest 1 spouse can manage property but each spouse owes fiduciary interest to other spouse to

behave in their best interests

Sawada v Endo Husband endo go into car accident with p. After the car accident but before suit was brought, the endos conveyed their home held in tenancy by entirety to their sons. p sued husband endo and p got judgment against him. When p tried to collect on their judgment, their attempt was frustrated bc endo conveyed their interest in their house to their sons. P want court to set aside conveyance of endo’s property as a fraudulent conveyance, which was held in tenancy by entirety until it was conveyed to their sons. The p cannot attach the property bc it was held by endo’s sons.Court holds property held in tenancy by entirety may not be subjected to claims of creditors against only 1 spouse. Court joins group 3 and holds the conveyance from endo to their son was not fraudulent. Court says choosing btwn family and marriage over interests of creditors is easy.

28

Page 29: Property Final Outline

*****Tenancy by entirety is still alive bc in majority of states, creditor of 1 spouse cannot reach the tenancy by entirety bc one spouse cannot assign his interest (group 3)

Group 1: estate is C/L tenancy by entirety. Husband can convey whole estate Group 2: interest of debtor spouse may be sold or levied upon for his debts, subject to the

other spouse’s right of survivorshipo If HA group 2, transfer was fraudulent. Creditor could attach.

Group 3: an attempted conveyance by either spouse is wholly void and estate may not be subjected to separate debts of one spouse only (estate not subject separate debts of 1 spouse)

Group 4: contingent right of survivorship appertaining to either spouse is separately alienable by him and attachable by creditors

o If Ha group 4, creditor could have entire propertyCourt creates way for married couples in tenancy by entirety to shield assets from creditors

Legislative intent frustrates creditor rights Family asset protection is overly broad (i.e. use homestead protection; only protect home of

family rather than all assets in tenancy by entirety like 20mm commercial prop) Vests tortfeasors with de facto immunity.

Argue for debtor using Sawada: sawada establishes that tort creditor cannot execute against 1 tenant in tenancy by entirety.

Argue for creditor using Sawada: person who owns valuable commercial property should not be able to shelter property from tort

claims by holding it as entirety o Narrow holding of Sawada: limited to cases where we want to protect single’s

spouse’s interest in family home Why are people in state which recognize tenancy by entirety immune from forfeiture and

creditors. There should be only 1 rule so states that don’t recognize tenancy by entirety are not completely disadvantaged (we need uniform law)

Equitable distribution of Marital Property what property is subject to equitable dist?

Termination of Marriage by Divorce: Professional Skills and creditials aquired during Marriage

In re Marriage of Graham (1978)Wife supported husband while he got an MBA, but there is no other potential marital property. wife seeks property interest in the MBA. Wife argues increased valued generated by MBA should be mine bc she invested most of her earnings into his MBACourt holds MBA is not marital property and just an intellectual acheivement bc it doesn’t have exchange value, terminates on death of holder, not inheritable, cannot be assigned, sold transferred or conveyed. But MBA may be considered in maintenance (alimony).

Equitable distribution: presumption that all property is divided 50/50 but court can divide property as it sees fit

Arguments for MBA NOT PROPERTY: difficulty on the valuation of the degree. To speculative that he contribution will contribute to his future earnings, not an express agreements, she assumed risk of divorce

o Dissent counter this by courts valuing loss of future earning in tortsArguments for MBA IS PROPERTY: She argues she is only entitled to marital property and the only reason they have no property is bc she invested their money into his mba; if they bought property, she would get some

29

Page 30: Property Final Outline

o Opportunity costo Courts vale future earnings in tortso Distributive justice: he should owe her something bc she invested in himo If there was marital property in marriage, she would have gotten greater share of

propertyDissent: adopt Obrien v Obrien NY standard where court held interest in professional career potential is marital property represented by direct or indirect contributions of the non-title holding spouse

Obrien court rejects that alimony is adequate bc it doesn’t take into account appreciation If Graham lived in NY she would have been way better off

o NY statute: equitable distribution law Legislature broadly defined marital property intending to give effect to

economic partnership concept of marriage relationshipo Obrien interprets NY statute that medical license is martial property

Elkus v Elkus (1991)Opera singer’s earning capacity increases during the marriage and her husband, her voice coach, sacrifices his own career to help her succeed. Husband seeks portion of wife’s earning capacity.Court holds husband entitled to the value of her status bc marital property can be intangible and to the extent the husband’s contributions and efforts led to an increase in value of wife’s career, this appreciation was product of marital partnership, therefore, marital property subject to equitable distributionNY statute: equitable distribution law: legislature broadly defines marital property intending to give effect to economic partnership concept of marriage relationship

Husband argues: using Obrien: husband should get portion of her earning capacity bc he was vocal coach (tangible contribution) and took care of kids (less tangible contribution) and these were contributions to her career

Wife argues: it was her talent and not her husband’s contributions that brought about her singing career.

o Counterargument: if she had been making the same amount when the marriage began as when it ended, there would be no claim, but that’s not the case here

Plenty of people with talent who don’t become famous as wife bc they spend time doing things husband did instead of focusing on their career

Talent required for singing is same as legal career; degrees is not the only part of career (analogize degrees to singing career)

30

Page 31: Property Final Outline

LEASEHOLDS; LAW OF LANDLORD AND TENANT***always pay attention to market forces and their role in financing the position of parties***

Term of years: estate that lasts for some fixed period of time; it must be set out clearly in the lease or by reference to a formula that will produce a fixed calender date for the beginning and end. It can be a defeasible—either deteriminable or subject to condition subsequent. (almost all commercial leases and some residential leases use term of years)- i.e. july 1, 2005 to july 1, 2006

No notice of termination necessary; lease ends when term of years expires On October 1, L leases Whiteacre “to T for one year, beginning October 1.” On the following

September 30, T moves out without giving L any notice. L’s rights? No rights. No notice is required for a Term of Years.

Periodic Tenancy: a leasehold for a recurring period of time such as month to month/year to year. A period tenancy continues in existence until either the ll or tenant gives advance notice to other party of termination of the lease. No notice given, period is automatically extended for another period. Created by the parties intentions or by operation of the law

Notice of termination: common law: termination of year to year requires 6 month noticeo Shorter tenancies require notice equal to period involved (30 days notice to terminate

month to month) notice on last day of month, not in the middle and equal to period length

rule: when tenant doesn’t give proper notice, period renews as same terms on prior lease i.e. 800 per month in rent, with no agreement on term of lease

o courts construe as periodic month to month 9600/year, payable 800 per month, no term specified

o some courts construe as year to yr periodic tenancy, other courts construe as month to month if residential prop

To T from year to year, beginning October 1, t moves out of sept 30th with no notice. Claim full rent. Six months notice is required for a Periodic Tenancy from year to year. Without such notice we assume renewal.

No fixed term. “At an annual rental of $24,000 payable $2,000 per month on the first of each month.” Periodic tenancy, but it isn’t clear what kind.

L: Year. Annual rental implies year to year with convenience payments.T: Month. Monthly payments imply month to month totaling $24,000.

31

Page 32: Property Final Outline

Tenancy at Will: leasehold for an unfixed period or time. It lasts as long as both parties desire. Either party can terminate it at anytime.

Notice of termination: either ll or tenant at any time. o C/L: estate ends immediately w/o advanced notice on day tenant abandons property or

LL delivers notice of termination Death: tenancy ends when either party dies i.e. t occupies land with LL consent but w/o agreement on duration or payment of rent

o court will find tenancy at will

Garner v. Gerrish (1984)Dude leased house to Gerrish for 100/mo from may 1, 19977 until Gerrish decides to terminate. The lease reas“for and during the term of quite enjoyment from the 1st day of may which term will end- Lou gerrish has privilege of termination this agreement at date of his own choice.” LL dies. LL’s executor tries to evict Gerrish.Court construes the parties intentions holds the lease expressly and unambiguously grants tenant the right to terminate and does not reserve the ll a similar right. Court holds lease was a determinable leasehold life estate (court calls it a life tenanacy terminable at will of tenant)

Tenant cannot argue term of years bc no start and end pointo Cannot argue period tenancy bc no periodo Does not want to argue tenancy at will bc either party can terminate

Tenant makes property argument that lease is a determinable life estateo Also makes k argument: read the lease, intent is clear

For rent payments of $500 a month L leases Greenacre “to T for the duration of the war.” What estate does T have? None of the traditional leases applies squarely, but Garner would say:

Determinable Fee Lease: If T is a corporation, this is more likely, because the corporation wouldn’t have a life to measure.Determinable Life Lease: If T is a person, this is more likely, because it is unlikely that L wanted the heirs to keep possession.

Tenancy at sufferance (holdovers): arises when tenant remains in possession (holds over) after termination of tenancy. Tenant has no right to be there but hes not automatically treated as a trespasser either (to avoid triggering adverse possession.

LL options when there is a holdover tenant which he must exercise w/in a reasonable timeo 1) treat tenant as trespasser and evict (and recover damages for period of wrongful

occupancy)o 2) charge tenant double rent (if he treats tenant as trespasser) or fair rental value OR

consent (rent as in original lease) and hold tenant to new tenancy Result of consent: periodic tenancy with length measured by rent received up to

year to year or a term of years. Ll MAY SAY: “you are evicted, but if you holdover, I will treat your decision as an agreement

for another year’s rento LL MAY NOT SAY: “failure to leave indicates acceptance of another year at 20k, the

previous rent bc 1) k law- no acceptance by silence and 2) desire to protect new tenant.

Crechale & Polles Inc v Smith (1976)

32

Page 33: Property Final Outline

5 year lease. tenant wants to stay few more months bc his new building is getting repaired. Before expiration of term, tenant discussed with LL a short-term extention bc his new building is being repaired. Tenant writes letter to LL confirming his understanding of the oral extension. LL responds by letter and denies an agreement he made with the tenant to allow them to remain, but LL takes no other action. Tenant stays on and tenders rent for 1 month and LL accept rent check (thus created a new period tenancy) and tenant vacates after giving sufficient notice to terminate the new periodic tenancy. LL seeks back rent, claiming tenant had entered into year to year lease.Court holds once LL elects to treat tenant as trespasser and refuses to extend lease on month to month basis, but fails to pursue his remedy of ejecting the tenant and accepts monthly checks for rent due creates a new month to month periodic tenancy.

LL’s mistake—Never elected the proper remedy—LL must decide at outset what his remedy is and be consistent (1 shot to get it right):

o 1) LL could treat tenant as trespasser and demand double rent o 2) LL could consent (express or implied) to creation of new tenancy

what LL should have done: LL should have argued there was no agreement and wrote letter to tenant that he was holdover and being evicted: should have demanded year and not evicted him

o Another big mistake was accepting check: court interprets tender of check as offer for month to month and cashing check as acceptance

THE LEASE

IS lease governed by property or K law Modern law reflects clear trend to treating residential lease as a k and not a conveyance Judicial rational for using k law to define LL/tenant rights is 2-pronged

o 1) courts emphasize k approach honors legitimate expectations of parties modern residential leases appears to be bilateral k: lease contains detailed

covenants addressing issues and reasonable LL/T would expect lease to be governed by k law like any other k

o 2) courts explain that application of traditional prop rules to residential leases produces inconsistent results with today’s value and standards

prop law fails to accommodate fundamental needs of new tenant; the low income urban tenant

Lease governed by prop law: lease or the conveyance is a 1-time event and imposes no duty on LL

Lease governed by k law: logical to treat LL/T relationship as one that imposes continuing duties on both parties

o Modern k law incorporates equitable principles 1) ll obligated to maintain premises in habitable condition 2) T has no obligation to pay rent if crucial structures to lease r destroyed 3) tenant can terminate lease and stop paying rent if LL materially breaches any

lease obligations 4) LL may be obligated to mitigate damages if tenant abandons

SELECTION OF TENANTS (UNLAWFUL DISCRIMINATION)

33

Page 34: Property Final Outline

Fair Housing Act of 1968 (FHA); part of civil rights act of 1968 What discrimination does FHA bar? (3604)

o discrimination based on race, color, religion, sex, familial status, national origin or handicap (DOES NOT prohibit discrimination based on marital status or sexual orientation)

definition of handicap: 3602(h): includes physical or mental impairment that substantially limits persons life activities: blind, mental disability, aids, paralysis and alcoholism is considered handicap under FHA standard in connection with sale or rental of dwelling

Handicap arguments: 3604(F)(3) defines discrimination practices different AND 3604(f)(3)(b): speaks to refusal to make reasonably policy accommodations

LL argues: 1) not a handicap—must be a mental impairment, physician saying you would be happier with a dog is not a mental impairment 2) accommodations not necessary to enjoy premises 3) no reasonable way to accommodate the type of pet in question

What actions violates the FHA if undertaken bc of the covered discrimination?o 1) 3604(a): refuse to sell or rent based on the discrimination coveredo 2) 3604(a): refuse to negotiate for rent or sale based on the discrimination covered 3) 3604(b): discriminating in terms, conditions, facilities or services involved in rent or

sale based on the discrimination coveredo Terms of sale: ll advertises apt. white couple inquires and LL says rent is 1500.

Whites walk away. Next day, blacks get apt for 1700. Violates 3604(b): terms of sale

o 4) 3604(d): falsely representing the property is not available for inspection, rent or sale based on the discrimination covered

o 5) 3604(c): publishing an ad, notice or statement that indicates any preference or limitation on prospective tenant and buyers based on the discrimination covered

Test: Souels: test for whether ad or statement indicates impermissible racial discrimination, ask “whether ad for housing suggests to ordinary reader or listener that particular race is preferred

o 6) 3604(e): for profit, to induce or attempt to induce any person to sell or rent any dwelling by representing the entry into the neighborhood of a person based on the discrimination covered

concern about ll making reps to scare people in neighborhood into selling or renting bc of fear that black people are coming into neighborhood

What are the FHA exemptions (3603)o 3603(b)(1)(b)***ANY discriminating advertising is NOT EXEMPTo 2 categories of property are exempted from all of the act:

1) 3603(b)(1): single family residence rented or sold by an owner WITHOUT the assistance of a REAL ESTATE BROKER OR SALES PERSON AND that owner does not own more than 3 single family houses at tone time AND the owner does not reside at the residence at the time of sale AND this is the only such sale within a 24 month period

if owner who owns less than 3 homes, sells or rents only 1 single family home w/in 2 years w/o a broker and no ads.

34

Page 35: Property Final Outline

2) 3603(b)(2): rooms in dwellings containing living quarters OCCUPIED by no more than 4 families living independently of each other, IF the owner lives in one of the living quarters

congress gives people right to discriminate against some in their house (small house in close quarters)

How to prove discrimination under FHA?o 1) P must establish prima facie case by showing discriminatory intento 2) How to establish discriminatory intent (2 methods)

1) disparate impact approach: statistical evidence used to show that the d’s policy or pattern of conduct has a disproportionate impact on persons in protected category

2) disparate treatment approach: focus on how an individual applicant is treated

***p must show he is member of statutorily protected class who applied for and was qualified to rent or purchase and was rejected although the housing was available*****

o 3) if p establishes prima facie case of discriminatory intent, burden shifts to d to prove a GOOD FAITH legitimate reason/justification for the discriminatory conduct in question (i.e. reasonable business purpose)

burden on LL to disprove discriminatory intent bc 1) LL is in better position to know why he chose 1 couple over other 2) if burden is on tenant, tenant would have to disprove every possible reason

o 4) if d provides a legitimate reason/justification for the discriminatory conduct, burden shifts back to the p to prove the legitimate reason/justification was a mere pretext for discrimination

Pretext is a purpose or motive alleged or an appearance assumed in order to cloak the real intention or state of affairs

Asbury v BroughmanAsbury was potential black tenant alleging discrimination by apt owner. Asbury called d’s office, employee of d said no vacancies, p asked for opportunity to fill out application, but d’s employee didn’t give one. P requested to see floor units, d employee refused. Next day, p’s sister in law, white woman, asked to see apt and d employee allowed her to see it.Court held p sustained prima facie case by showing she was minority, qualified to rent, apt was available and she was denied opportunity to rent or negotiation on basis of race.

D unable to prove legitimate, non-discriminatory reason for rejection Court held statistical data shown by d in regards to high % of minority in apt was

relevant but not dispositive

Souels, Home v. Downs and US Dept of HousingDowns was single mother who had child. Souels was the realtor, placed add for apt for ½ of 2 family dwelling and p saw ad and called realtor. P tried to get in touch with d realtor but no luck. The p and d finally spoke and d asked p how old her child was and p responded negatively. Downs contacted US Dept of housing who sent testors to d and d asked how old child was to testor. In meantime, d offered apt to family with children. D ended up renting to single woman w/ no children under 18.Court held substantial evidence supported conclusion that p bad attitude and d being out of town to care for her sick aunt RATHER than discriminatory conduct was reason in housing denial

35

Page 36: Property Final Outline

Court determined that d asking for child’s age was to accommodate elderly tenants who lived in house

US v Starrett City AssocStarrett decided to take over a complex on condition they would receive tax benefit. Prospect of large, low income housing scared many people- people feared white flight and ghettoization. Starrett and NY Dept of housing gave assurances that complex would be integrated community. To reach racial balance, Starrett put many minorities on waiting list permitting % of minorities in complex to be 30%, then 35% then 38% (this was what law suit settled for). Starrett’s policy for racial balance resulted in larger share of minority on wait list that whites.Court held quota violated 3604(a).

Starret argues: objective of the FHA and spirit of statute is to eliminate discrimination so there is integration—this is exactly what they are doing here—maintaining integrated community to avoid ghettoization

o Look to INTENT and PURPOSE of statuteo They were subsidized by govt and has taken govt’s objective of integrationo Uses Otera as precedent where everyone required to meet 3 limits: 1) quota must be

targeted program w/ stated goal 2) must establish program addressing past wrong to minority 3) ceiling versus floor quotas: floor (as least this many blacks) is cool but ceiling is not cool (no more than this amount of blacks)

Court distinguished Otera: Otera had short duration

***Arguments in Applying FHA: LL argues: I am exempt under 3603, I own less than 3 homes and LL can argue I didn’t use a

broker, I leased apt on my own, so I am exempt o This will not fly. Purpose of statute is for realtors not to perpetuate discrimination

and just by reading plan language of statute, using a “broker in any manner”- jus by contacting broker, she is not exempt

Tenant argues:o 1) Prima facie case: Damien makes prima facie case bc he is atheist, he was qualified

(willing to pay over MV), he applies for housing, it was available and he was rejected on the basis of his religion

LL would argue Atheist is the absence of religion thus its not discrimination based on religion

Statutory intent: enacted to stop targeted discrimination against particular groups: Atheist is not protected class

o Tenant: doesn’t matter if atheist is religion, its discrimination based on your religion. You cannot discriminate based on whether or not LL believes or likes your religion

2) burden shifts to LL to rebut presumption of discrimination:o LL: it wasn’t based on religion, didn’t want to rent to porn seller

Souels: legitimate to care about interests if other tenants. If tenant having bad attitude or carring about old couple is reason to reject tenant, then dude selling porn is good reason

In Souels: court says legitimate reason is to care for other tenants in building: cannot discriminate against children, but can against noisy children

Porn seller may use house in negative way

36

Page 37: Property Final Outline

o LL argues I am currently renting to person who is not my religion, so there is no discriminatory intent

Tenant; in Asbury: if portion of tenants being a certain race is not dispositive

My porn biz is in privacy of my home and wont affect anyone 3) burden shifts back to Damien: she told the broker she is looking for someone with

compatible religion3604(b): does not apply in this case3604(c): is there a violation of c?

tenant: unlawful to make ANY statement with respect to rental of dwelling that indicates discrimination…based on religion

o Souels: LL’s statement to white tenants that she should send her friends to see vacate apt but make sure they are white violates FHA: statement does not have to made directly to tenant bc statutory intent: trying to stop brokers from carrying out discrimination

o LL: I only sent this to my broker and it wasn’t published Souels: statement was sent to prospective tenants and that’s why it was violation

DELIVERY OF POSSESSIONo LL is obligated to deliver the legal right to possessession to a tenant whose terms begins

BUT LL’s obligation to deliver actual (physical) possession has split authority 1) American rule (minority view): LL is merely obligated to deliver the legal right of

possession to tenant when lease term begins. Obtaining actual possession is up to tenant. LL has no duty to oust trespasser or holdover tenant

o Arguments for American rule: 1) LL not warranted against wrongful act of 3rd party 2) tenant has same ability as LL, but even greater incentive, to bring suit to recover possession from holdover tenant

o Concern: economic waste: LL might refuse to enter into new lease until prior tenant has vacated

o Tenant’s Remedies: T has no claim against LL but can bring suit against holdover 1) tenant can treat holdover as trespasser and evict and recover damages 2) tenant can renew the holdover for new term, receive rent from holdover

2) English rule (majority rule): LL has an implied in law obligation to deliver actual possession to T when lease term begins in addition to the legal right of possession

o Arguments for English rule: 1) as btwn LL and T, LL has superior ability to ensure premises is vacant when lease begins 2) LL knows if holdover tenant intends to holdover 3) LL has legal right to evict holdover tenant 4) LL in better position to rebut any defenses that holdover tenant may raise 5) LL more sophisticated about eviction process and better able to bear costs (economies of scale)

Reasonable tenant presumes tenant will receive actual possessiono Tenant Remedies: t can bring suit against LL to terminate lease or recover damages

and holdover for possession and damages 1) terminate lease, find other premise and recover pertinent damages 2) adhere to the lease, withhold rent for period out of possession and recover

damages for the lost position

37

Page 38: Property Final Outline

Hannan v DuschTenant rented from LL for 15 year lease. When date of lease arrived, prior tenant was holdover and new tenant was unable to take actual physical possession. LL failed to oust previous tenant. Lease contained no explicit obligation that LL had to deliver actual possession. Tenant sued LL for damages.Court adopts American rule holding for LL: LL, absent an express provision in lease, impliedly covenanted that T should merely have legal right to possession at beginning of lease term.

Court says T should have sued holdover, not LL T could requested provision in lease that premises be delivered vacant, but he didn’t. Note on drafting leases: 1) figure out what jurisdiction your in 2) in any jurisdiction, make

sure to draft lease so tenant receives legal and physical possession on 1st day of the lease

SUBLEASES AND ASSIGNMENTS Assignment: transfer of tenants existing lease to a 3rd party (complete assignment of rights

by sublessor). Sublessor obligated to lessor in ko Hypo: a leases to tenant b for 10 years. After 5 years, b assigns his entire interests to c.

When a leased to b: privity of estate and k When b assigns to c: privity of k (assignment is k) Whats happens to a and b (lessor/lessee): privity of k continues but

assignment dissolves privity of estateDispute arises: A leases to b who assigns to c. who is liable if rent is not paid to a

A and b relationship: privity of k still exists so b is liable A and c relationship: privity of estate exists so c is liable B and c relationship: privity of k exists

Dispute arises: a leases to b who assigns to c, who assigns to d. no one pays rent. Who is liable?

A and b: privity of k. b is liable A and c: No privity of estate or k: C NOT liable A and d: privity of estate. D is liable

Sublease: whole new lease btwn original tenant and 3rd party (partial assignment of rights by sublessor). Sublessor obligated to lessor in estate

o Hypo: a leases to b for 10 years, b subleases to c for 3 years A and b: privity of estate and k A and c: no privity of k and no privity of estate bc b has

reversionary interest B and c: privity of k and estate ***only b is liable to A if A receives no rent

Exception: although there is no legal relationship btwn A and C, lessor can sue under the 3rd party beneficiary theory

K law principle: k made btwn 2 parties intended to benefit 3rd party may be enforced by that 3rd party

o i.e. as part of the B-C sublease, c agrees to perform b’s obligations under the A-B lease including the agreement to pay rent

Most jurisdictions: a would be considered 3rd party beneficiary of the b-c sublease

38

Page 39: Property Final Outline

o Specific covenant in sublease would create privity of k btwn a and c and a could sue b or c for unpaid rent

Privity of k: relationship of parties to k LL can always sue whoever they actually contracted with (original lessee) and

anyone who contracted for his benefit (3rd party beneficiary)Privity of Estate: relationship btwn 2 parties to conveyance of an estate

LL can sue whoever is current holder of the entire lease.

Test of assignment v. sublease (2 approaches) 1) Objective (mechanical test)- common law rule/majority

o 1) If tenant transfer right of possession for the ENTIRE remaining term of the lease, the transfer is an assignment

If tenant transfers only part of remaining term, it’s a sublease Parties actual intention is irrelevant

o 2) if transferring party transfers entire estate but retains right of entry, MOST jurisdictions still find an assignment

2) Intention of Parties Test (modern, minority rule)o 1) court ascertains whether transforer and transferee intended to create a new LL/tenant

relationship btwn themselves, if so it’s a sublease if they intended tenants rights in existing lease to be merely transferred, then its

an assignment o 2) possibility under this approach to transfer entire lease term w/o subjecting parties to

legal consequences of assignment

Ernst v Conditt (1964)Ernst leased to Rogers that required Ernst’s consent to assigment or sublease. Rogers took possession and then Rogers negotiated a sublease with Ernst’ permission, to Conditt upon the express condition that Rogers would remain personally liable for performance of lease. Rogers signed an agreement by which he “sublet the premises and Conditt signed where it said “accepted the subletting premises.” Conditt stopped paying rent. Ernst sued Conditt for back rent on theory lease was assigned to Conditt an thus there was privity of estate with Ernst.Court held the agreement btwn Rogers and Conditt was an assignment and not a sublease, therefore the Ernsts could sue Conditt because there was privity of estate

If court held it was sublease: Ernst could not sue conditt, he could only sue rogers, who could then sue conditt. However if rogers is insolvent or gone, conditt would have incentive to argue that Erns’s only remedy is to collect from him

Conditt argues for intent to create sublease: o Express right of re-enter thus did not completely give away rights as required in assmto 2nd tenant pays rent to the 1st tenant (and then 1st tenant pays ll)—looks to create a new

tenant/ll relationship tenant giving money directly

o tenant 2 calls tenant 1 directly instead of calling ll with problems—itnent to create new tenan/ll relationship

o intention of parties was for sublease bc sublease was written in ko in k, rogers (original lessee) agrees to remain liable: rogers didn’t assign his entire

interest, thus by implication, rogers retained right of re-entry Argue for intent to create Assignment

39

Page 40: Property Final Outline

o Sublessor did not reserve any rights including right to re-enter the apto Entire duration of lease conveyedo Assignee is paying money directly to lessoro 3rd party beneficiary: court must find clear intention in k that k included a benefit to 3rd

party conditt agreeing to assume covenants in lease, 2 of which were for the benefit of

original lessor who is 3rd party beneficiary

TENANTS RIGHTS TO ASSIGN OR SUBLET unless a lease expressly limits or prohibits assignment or sublease, a tenant is free to transfer

the lease by either method however, majority of leases expressly restrict tenants right to transfer

o 1) lease clause prohibits any transfer of tenants interest enforceable in most jurisdictions ONLY if freely negotiated (RE) courts interpret these clauses narrowly: ban on assignment does not preclude

subleaseo 2) Lease contains approval clause (clause that says lease cannot be assigned w/o prior

consent of lessor) Majority approach: if lease contains approval clause, LL may arbitrarily refuse

to approve a proposed assignee to matter how suitable assignee appears and NO MATTER how UNREASONABLE the LL objection

Rational: 1) lessor made choice and should not have to depend on another tenant w/o consent 2) tenant could have bargained for a reasonableness clause 3) precedent: people have been relying on this for construction of current leases

Minority Approach: if lease contains prior consent of lessor clause, such consent is withheld ONLY where lessor has a commercially reasonable objection to the assignment

Rational: 1) policy against restraints on alienation pertains to leases in their nature as conveyances 2) lease is k and in every k, there is implied covenant of good faith and fair dealing 3) lessor still has right to object on reasonably commercial grounds 4) lesee remains liable to lessot as surety even if lessor consents to assignment and assignee expressly assumes obligations of lease

What is commercially reasonable? o 1) financial responsibility of assigned 2) suitability of use 3)

legality of use 4) need for alteration 5) nature of occupancy 6) hurts tenant mix 7) business unlikely to succeed 8) LL would receive less rent 9) sublease would violate LL’s k w/ another tenant 10) lost volume seller: if I allow assignment, I lose possibility of a different rental

o What is not reasonable? Desire to raise rent, to secure a commercial advantage, taste, sensibility, convenience, or of transferee’s proposed use is ethically offensive to ll though reasonable (see American book v YU)

Kendall v. Ernest Pestana Inc

40

Page 41: Property Final Outline

Lease of hangar space contained the provision that allows subleasing and assignment ONLY with consent of the lessor. Tenant agreed to sell his business alone with assign his lease to another guy. New guy was financially stronger and richer and was willing to assume the lease obligations. LL refused to consent to assignment unless. New tenant sued LL claiming refusal of consent was unreasonable and an unlawful restrain on alienation.Court adopts minority rule and holds consent may be withheld ONLY where lessor has a commercially reasonable objection to the transfer, even in the absence of a provision in the lease stating that consent to assignment will not be unreasonably withheld.

Courts reads this requirement into the lease on property and k basis:o 1) Property: policy against restrictions of alienationo 2) k: good faith requirements of k law

the parties could have negotiated for clause prohibiting assignment or giving LL absolute discretion. The clause would be fine if parties freely negotiated it

American Book Co v YeshivaYU refused to consent to transfer to a birth control and abortion counseling center although the center was financially responsible and its use was otherwise suitable.Court held YU’s objective was not commercially reasonable.

TENANT WHO DEFAULTS

Landlord Remedies

1. LL’s self help remedy to Tenant in possession who defaults or breaches Common law rule: LL may rightfully use self help to take possession so long as 1) he is

legally entitled to possession and 2) uses peaceful means to re-entero Fast and cheap and if tenant sues LL, LL can argue tenant abandoned or surrenderedo If LL uses self help when tenant hasn’t breached, LL is liable for wrongful eviction

(significant damages and tenants possible restoration to possession may force LL to break later leases)

o Criticism: risk of violence, possibility of unjustified eviction, availability of alternative remedy in judicial proceedings (eviction proceedings allow LL to safely retake premises but w/ attorney costs and take a while)

o Hypo: LL hasn’t seen tenant for month. When can he change locks Tenant: you can’t use self help. Berg. LL: self help is not foreclosed when the premises are ABANDONED.

Advice to LL: collect evidence of abandonment before self help or go to summary proceeding.

Modern rule: self help is never available to disposes a tenant who is in possession and has not abandoned or voluntarily surrendered the premises

o Summary proceeding: quick (although still time consuming) and efficient means by which to recover possession from breaching tenant

Berg v Wiley (1978)Lease provides for no change in the building structure w/o prior written permission and for the restaurant to be operated in a lawful and prudent manner. Tenant remodels w/o permission and fails to

41

Page 42: Property Final Outline

meet health code regulations. LL notifies tenant that if lease violations were not cured in 2 weeks, he would re-take possession. 15 days later, violations have not been curred. LL enters premises and has locks changed and tenant sues LL for wrongful eviction.Court held 1) lockout cannot be excused on the ground that tenant abandoned or surrendered bc tenant intended to retain possession and only closed to remodel 2) self help re-entry was inappropriate bc lockout was not peaceful even under common law rule: Court adopts Modern view: only lawful means to dispose of tenant who has not abandoned nor voluntarily surrender is by resort to judicial process.

Argue for LL before Berg: o Common law rule: ll can use selfhelp as long as its done peacefully o tenant by claiming I should have used summary proceeding and had I done this, I would

win, thus tenant is conceding that I would have wono Judicial proceedings are expensive and time consumingo Tenant breached leases bc she remodeled w/o LL permissiono Tenant breached lease by not running premises in lawful manner; cited by health depto According to lease, LL can enter and re-take premises for breach

Argue for LL after Bergo People who benefit from Berg are deadbeat tenants (people who don’t breach will have

to bear cost of raising rents on behalf of breaching tenant)o Berg discourages useful use of propertyo Common law self help encourages LL to make sure tenant does breacho Berg imposes more costs on LLo Only parties benefited are breaching tenants (LL loses an option and non-breaching

tenants pay more rent for new options) After berg: LL, to avoid costs from leaving a property unused for long period,

will raise rents on all tenants. Slippery slop bc all rentals will increase in cost. Argue for tenant:

o Under berg self help not allowed. When tenant breaches, ll not entitled to use self help and must use summary proceedings thus I am entitled btwn diff of fmv and lease price

o LL did not go to court to evict me—he evicted the wrong way by using self helpo Legislative intent: Self help increases likelihood of violence, so get rid of ito Due process: right to adjudication before eviction as opposed to aftero LL may not do good job in making sure tenant breachedo Burden should be on LL bc they have economy of scaleo Would prefer litigation before eviction so it doesn’t interrupt business

TENANT WHO HAS ABANDONED POSSESSION-LL’S DUTY TO MITIGATE abandonment: occurs when tenant 1) vacates leased premises w/o justification 2) lacks the

present intent to return 3) defaults in payment of rent what can LL do if tenant abandons?

o 1) LL rejects surrender and leaves premises vacant and sue tenant later for accrued rent at end of lease term

LL MUST make it CRYSTAL CLEAR he is not accepting the forfeiture of lease (LL must write tenant and say, I am not letting you out of your lease and your still bound)

silence is deemed acceptance of forfeiture

42

Page 43: Property Final Outline

Issues here: vacated premises can lead to blight and tenant can disappear and/or become judgment proof

o 2) Anticipatory breach: permits LL to sue and recover damages when tenant has made it clear that he is abandoning. must make sure jurisdiction allows recovery under this theory

LL must establish tenant is going to breach (must be a clear abandonment of premises like a letter stating so)

Damages: difference btwn price under lease and fmv Worst option in an up market: anticipatory breac

Court will offset the rent owned under the lease from the fair market value, the fmv will be higher, LL gets 0

Best option in down market: anticipatory breach: can at least collect difference btwn lease price and mv assuming mv is lower

Why difficult? Jurisdictions may not recognize this remedy, litigation is expensive, might be hard for LL to show tenant is breaching

o 3) LL accepts surrender of lease and terminates lease. if ll elects to terminate, this becomes his exclusive remedy

silence is deemed acceptance of forfeiture; i.e. tenant writes a letter clearly stating he is abandoning and LL does not write back

Always Bad option: reject surrender and sue at end of lease Very risky bc it tenant cant pay now, they are prob insolvent, or gone Premises vacant could be damaging to premises

o 4) LL can re-take possession and re-let premises on tenants behalf either voluntarily or pursuant to a duty to mitigate damages mitigate damages and sue original tenant for unpaid balance

this can be done even in jurisdiction with no duty to mitigate LL MUST make it CLEAR by rejecting tenants offer to surrender and I will

relet on your behalf and anything I collect from the relet will offset the rent owed under your lease

Must be clear LL rejects forfeiture of lease Silence of LL is deemed acceptance of forfeiture

LL’s duty to mitigateo LL is obligated to mitigate damages. Sommer.

Note: not followed in NY and in some jurisdictions, only applies to residential leases

o Reasons for mitigate damages rule: 1) less burdensome on LL to find substitute residential tenant (economies of scale), but not necessarily for commercial tenant (LL might not be in better position) 2) old rule encourages economic and physical waste of housing resources (promotes economic efficiency to not have vacat apts and have apts being used and paid for)

o Reasons against mitigate damages rule: 1) tenant incentived to find subleasee or assignee 2) tenant should not be able to impose a duty on LL 3) LL should not be forced into a relationship with new tenant 4) LL should not be required to seek new tenants continually 5) abandonment of property is an invitation to vandalism and law should not encourage such conduct by putting a duty to mitigation on LL 6) lost sale theory when LL cannot readily fill vacant apts

How to mitigate damages

43

Page 44: Property Final Outline

o In order to fulfill duty to mitigate, LL must treat the breached apt just like any other vacant apt. Sommer

LL argues: this rule is unfair bc of lost volume apt: he will get double rent by renting other 4 apts and the vacated apt

LL pays for tenant’s breach Cost of mitigation is too high especially if there many defaulting T

o Although courts allow LL to claim costs of mitigation, tenants are prob judgment proof at that time

Tenant argues if LL was not able to relet: Failed mitigation: your rent was to high, lower your prices. Don’t go

over market value. If you do, this is not reasonableo LL has no obligation to get the rent that former tenant was

paying, only expected to get the mv of rent Advertise and keep records of showing tenants apts

o If tenant has presented a commercially viable alternative to breach and ll turns it down, tenant could argue LL failed to mitigate and avoid all damages

If lease says LL can reject for any reason: LL has right to say no but there is still a requirement of duty to mitigate

IF LL fails to mitigate damageso 2 jurisdictions

o 1) LL would get nothing if he fails to mitigate damageso 2) Law penalizes failure of duty to mitigate by reducing the amount of rent recovery

against the tenant by the amount the LL could have obtained thru mitigation Reduction is usually measured by fmv

o ***If LL fulfills duty to mitigate, tenant is on the hook for all damages

Sommer v. Kriedal (1977)2 situations where tenants abandoned their apts and LL’s didn’t re-rent right away.

1) a week after signing lease and paying 1st month and security deposit, tenant wrote letter that marriage was broken off and he could not afford rent. LL did not answer. 3rd party was willing and able to rent but LL did not re-rent until year later

2) tenant entered into 2 year lease and lease prohibited tenant from sublet or assignment w/o consent of LL. after year, tenant vacated.

Court holds LL has duty to mitigate damages when tenant has abandoned and where LL seeks to recover rents from defaulting tenants by making reasonable efforts to re-let the apt

o LL must treat apt as one of his vacant stocko LL has burden of proving he used reasonable diligence to re-let (i.e. advertising, hiring

broker, showing apt to prospective tenants)

LANDLORD DUTIES AND TENANT”S RIGHTS AND REMEDIES

1. Quite enjoyment and Constructive Eviction Modern (today): any act or omission of LL which renders the premises substantially

unsuitable for the purpose for which they are leased, or which seriously interferes with the enjoyment of the premises, is a BREACH of the covenant of quite enjoyment and CONSTITUTES constructive eviction for the tenant

44

Page 45: Property Final Outline

o Common law made tenants duty to pay rent conditional upon the LL’s performance of this obligation

o LL must not do anything to interfere with use and enjoyment of the premises

Constructive eviction: when ll’s conduct substantiall interferes with tenants use and enjoyment of premises, a constructive eviction has occurred. This provides tenant with remedy for LL’s breach of existing duty

Although duty of implied warranty of habitability has limited application in residential LL/T disputes, still utilized in commercial leases

3 keys issues arise hereo 1) what is the wrongful conduct?

any affirmative act of LL that seriously interferes with enjoyment of premises meets requirement

(i.e. LL brings home prostitute and causes loud noise, LL doing loud construction, LL trespassing repeatedly)

omission of LL’s duty***must tie LL’s omission to disturbance conduct of 3rd party

LL only responsible for conduct of 3rd party if LL has legal right to control 3rd party’s conduct (he is a tenant)***even if ll could take action, if most he could do is call cops, he has no duty to do so because he cannot control 3rd party

o 2) is there a breach (does the conduct substantially interfere with tenants use and enjoyment so that it’s a breach of the covenant of quite enjoyment?

Tenant must be so essentially deprived of the beneficial enjoyment of the leased premises that they are rendered unsuitable for purposes for which they are leased

Interference doesn’t need to be permanent nor do it need to prevent tenant from occupying premises

****however, if tenant is aware of LL’s conduct when taking possession, right to assert constructive eviction in future is waived

o 3) if there is a breach, what is the remedy 1) Constructive Eviction: If material breach so as to make premises

unsuitable for purposes they were rented, then tenant relieved of rent payments

1) Tenant can vacate premises, terminate lease, avoid liability for future rent and may recover for compensatory damages and special damages

o General damages=amount that fmv exceeds lease rento Special damages: moving expenses, damages to fixtures

2) tenant can remain in possession and sue for damages 2) regular contract damages for harm suffered due to breach (tenant

not entitled to constructive eviction in some cases)

Reste Realty Corp v Cooper (1969)Lessee took over a 5 year lease in lessor’s building and 1 year into lease, they made new 5 year lease. Lease contained provision: Tenant accepts as is and must keep in good condition. Premises kept flooding. Before new lease, LL promised to remedy water problem. The flooding continued, tenant complained and continued to take care of problem. Eventually the problem became so severe thar tenant could no conduct normal businsss at the premises. Last straw was Tenant had work meeting in

45

Page 46: Property Final Outline

office and premises flooded again resulting in 5 inches of rain. Tenant sent a notice of eviction and left premises. LL sued to recover rent for unexpired term of lease. tenant defended by saying LL breached lease (covenant of quite enjoyment) and therefore tenant not required to pay rent (constructive eviction).Court overrules old law that material and permanent breach gives rise to constructive eviction and held 1) LL breached covenant of quite enjoyment and 2) constructive eviction was proper (tenant can leave and avoid payment of rent) bc LL’s breach of covenant of quite enjoyment is rendering premises substantially unsuitable for purpose of which they are leased which gives rise to tenant remedy of constructive eviction

examples: failure to supply heat, waste pipe clogged causing awful odors, LL rents apt to tenant for lewd purposes

condition does not have to be permanent ***constructive eviction waived if tenant does not vacate w/in reasonable time

depending on the circumstances (here, manager died, tenant complain, waited and hoped for relief, tried to take care of problem, then crowning blow happened, tolerance ended, she vacated w/in days after notice)

o if I didn’t do anything for 2 months, remedy is prob waived

***if tenant is aware and on notice of LL’s conduct or breach when taking possession, right to assert constructive eviction is waived

Stewards v Child: covenant for rent is independent (not a condition) of the covenant of quite enjoyment. Remedy: money damages. REJECTED BY RESTE.

blown light fixtures in entry way of tenant’s business. 1) is lease breached?

o Duty to repair and light fixtures is clearly repair 2) remedy?

o does the breach render the premises substantially unsuitable for purposes of which they are rented? Prob not.

What if 2/3 of toilets are broken. 1) definite breach 2) Remedy: are toilets broken render premises substantially unsuitable for purposes of which

they are rented? Risky suit. ceiling in conf room is falling bc roof leaks. LL knows, plaster is coming off and there is hole.

1) There is a definite breach 2) this breach is prob material as long as you establish the conf room is sued for meetings and

the hole in ceiling in materially unsuitable for their business purposes.o Prob gives rise to constructive eviction

IMPLIED WARRANTY OF HABITABILITY Warranty: LL will deliver over and maintain, throughout the period of the tenancy, premises

that are objectively safe, clean and fit for human habitation What breaches warranty? Basic test is objective: would a reasonable person find

premises uninhabitable?o Compliance w/ housing code: some states say LL breaches warranty only if condition

violates housing codes other states says its not dispositive Hilder: Substantial violation of housing code is prima facie evidence of breach.

46

Page 47: Property Final Outline

o Fit for human habitation: majority states mandate lease must be fit for human habitation, meet bare living requirements, or comply with similar standard regardless of housing code. Hilder.

LL can breach even if complying with housing code What’s procedure to claim breach of warranty?

o 1) tenant must provide LL with notice of defect and allow reasonable time for repairs to be completed

o 2) Notice must be specific to inform LL about nature of defecto 3) length of time allowed for repairs varies with nature of defect (i.e. toilet broke, 1 to 2

days/roof: longer time needed) Notice: LL must have notice of defect and reasonable time to repair it. Is warranty waivable?

o Arguments for no waiver: majority of states find waiver invalid against public policy If waivable, LL would routinely compel tenants to waive warranty due to

superior bargaining position thus frustrating purpose of warranty If waiver is allowed, could create negative external effects which have bad effect

on society posner’s argument does not factor the negative impact that allowing

parties to freely contract with waiver will have on society if ll can waive warranty, there will be many inhabitable apts bc LL might have

very little incentive to make habitable due to costs if law requires warranty, LL will make changes and comply

if no warranty, many apts will be slumso Posner is for waiver: freedom of k

tenant has choice of pricing waiver into lease: I can get cheaper apt w/o waiver now instead of apt w/o warranty in great area, I must goo far away to bad

area for an apt I can afford warranty does not benefit poor people, benefits middle class warranty increases cost for LL and incentives LL to turn building into condo or

co-op thereby decreases number of available apts for poor tenants causing prices to increase while price of co-ops increase due to more supply

poor people get screwed Arguments for Implied warranty of habitability

o Goal of modern tenant is to secure safe habitable housing with adequate heat, light, etc and today’s tenant lacks skills to make repairs

Thus, if law allocates repair burden to tenant, defects will not be repairedo Modern tenant cannot protect interest thru negotiation due to disparate bargaining

positions—tenants would be forced to accept substandard housingo Enactment of housing codes, movement toward lease as k over conveyance and societal

belief that people have right to decent housing supports warrantyo Paternalistic protection for tenants who do not bring attorneys when signing lease

Arguments against implied warranty of habitabilityo Reduces quality of affordable housing

Warranty imposes extra costs on LL, LL passes costs to tenants by increasing rent, tenants unable to afford high rents, thus tenants force out of housing market

o LL’s ignore warrantyo Tenants living in inhabitable conditions who are aware of warranty rarely assert them

47

Page 48: Property Final Outline

Scope of warranty of habitabilityo Not universal: 1) not all jurisdiction shave warranty 2) warranty commonly does not

apply to all residential leases (i.e. single family residence or agriculture lease) 3) majority of jurisdictions have declined to extend idea of implied warranty of fitness or suitability to commercial leases

REMEDIES for breach of warrantyo 1) terminate lease and leave and recover damages (relocation osts plus excess of

replacement rental over the lease rental for the balance of the term)o 2) stay and withhold rent: tenant must notify ll of exercise of this remedy and deposit

rent to escrow accounto 3) stay, repair and deduct from rent: use reasonable amount of rent to make repairs

sufficient to bring premises into habitable conditiono 4) stay and recover damages:

Damages: lease is 500/month. Value now due to violates is 200. At time tenant brings suit, mv of apt is 1k

Tenant gets 800: current mv of premises in habitable condition minus what value of apt is in inhabitable condition

Punitive Damages: Where the breach is willful and wanton or fraudulent. Additionally, when a landlord after receiving notice fo a defect, fails to repair the facility that is essential to the health and safety of his tenant, an award of punitive damages is proper. Hilder

Hilder v St. Peter (1984)Tenant living in a apt in awful conditions: filthy, front door didn’t lock, plumbing leaked causing celing to fall onto baby crib, inoperable toilet. LL refused to do anything about the awful living conditions of tenant. Tenant paid all rent during tenancy. Conditions included: broken windows, toilet clogs, broken lights, outlets broken, leaking water pipes, odor of raw sewage.Court held: in the rental of any residential dwelling unit, an implied warranty of habitability exists in the lease that landlord will deliver over and maintain, throughout the period of the tenancy, premises that are safe, clean and fit for human habitation. Once the LL is notified of a defect, tenant may begin to take action

Tenant Remedy when warranty is breached: Punitive damages may also be appropriate (wanton, willful conduct

Doctrine of constructive eviction where tenant abandons is no longer viable in residential leases: tenant can withhold future rent and seek damages w/o abandoning

The Problem of Affordable Housing: Rent control Consists of price controls (set as below market rates), usually augmented by limitations on a

ll’s ability to evict tenants at the end of the lease term (thus curbing the ll’s ability to lease to new tenant at marke rates)

Reasons for: (1) We want poor people to find cheaper housing (However, this really just helps the middle class). (2) Promotes neighborhood stability (prevents rapid gentrification) 3) keep economically vulnerable (old and poor) tenants in place with a relatively fixed portion of their income going to housing

48

Page 49: Property Final Outline

Reasons against: economically inefficient—produces an inefficient allocation of housing resources (1) Posner: Creates less supply for those that rent control intends to help (less building and more condos). (2) squeezes LL’s profit margin and LL decides he will make more $ in co-ops (3) usually given to people there 1st (first come first serve) 4) creates black market (key charge for 5k) 5) when rent control apt becomes available, LL will choose better economically suited person over immigrant w/ no credit

Chicago Board of Realtors, Inc. v. City of Chicago(1987)

Court held Chicago Rent Controls were sufficiently specific and reasonable in light of purpose to promote public health, safety and welfare.

o Concurring opinion: Doesn’t actually help the poor, benefits middle class. Losers of rent control is LL, poor tenants and future tenants.

LAND TRANSACTIONS

Real estate contracts1. 2 days after executing the sales k, but before paying the full earnest money deposit, buyer has a

change of heart and wants out of the k. what alternatives, if any, does the buyer have. What remedies does the seller have

a. buyer: i. attorney review clause:

1. this is a protective provision; if the buyer has a change of hear, the buyer can disagree with a clause in the contract

a. i.e. inspection clause: buyer can argue that the time period in the inspection clause to get an inspection and give a report the seller is unacceptable bc its too short (5 days)

2. unless out of the attorney review period, there is no binding offer and acceptance bc people change their minds

b. seller: prob won’t sue here 2. suppose the seller, rather than the buyer, has a change of hear, what alternatives does the seller

have? a. Same as previous answer if within the attorney review period

3. Suppose the buyer pays a deposit and conducts an inspection. Inspection reveals extensive termite damage. What optios does the buyer have. Is the situation different if the inspection reveals the roof is 25 years old and will need to be replaced in 5 years?

a. Clause 11: professional inspection clause

49

Page 50: Property Final Outline

i. If the roof is functioning right now and performing the function for which it is intended, then its deemed in operation condition and you can’t get out the k

Marketable Title General rule: every k of sale of real estate contains an implied duty of the seller to deliver

marketable or merchantable title to the buyer.o this obligation can be expressly disclaimed by agreement by the buyer and seller

What is marketable title: title that a prudent buyer would accept, one that is free from reasonable doubt

o Title that is doubtful and unmarketable exposes the party holding it to the hazard of litigation

What makes title unmarketable1. Title is unmarketable if seller does not own the estate he purports to sell

a. Title by adverse possession is marketable where the seller proves there is no possibility the record owner will ever succeed in regaining title

2. Title is generally unmarketable if it subject to any lien, easement or encumbrancea. Encumbrance: right or interest in land other than the present freehold estate or

future interest that reduces the value or restricts use of the landi. Includes mortgages, easements, covenants, leases, tax liens,

encroachmentsii. i.e. if property lacks access to public road. Court will find title

unmarketable bc litigation is required to obtain an easement

what is marketable title versus non-marketable title? (look to loyhmeyer) all courts agree that the mere existence of zoning, building and land use regulations DOES

NOT MAKE TITLE UNMARKETABLEo the law is not an encumbrance and a normal buyer intends to use land as it was used in

the past most courts do HOLD THAT VIOLATION of zoning ordinances renders title unmarketable

o i.e. buyer purchases land, buyer would be subject to the risk of litigation buyer should reasonably expect that the property does not violate the law

o majority view: violating a building code DOES NOT MAKE TITLE unmarketable visible encumbrances

o most courts hold visible easements for roads, power lines, sewer pipes, do not affect marketability

o rational: buyer knows or should have reason to know that an easement exists and enters into a k that fails to mention the easement, the buyer presumably agrees to accept title subject to the easement

breach of title covenant1. buyer who learns of title defects before closing must notify the seller and give a

reasonable time for the seller to cure the defects2. if the seller fails to deliver required title at closing, buyer is excused from performing

the k and seller is liable for breach of the kremedies

50

Page 51: Property Final Outline

1. specific performance of the k with an abatemento equitable remedy which is not always availableo awarded ONLY when $ damages are inadequate

2. rescind the k and recover down payment and obtain restitution 3. sue seller for damages

o loss of bargain damages: basic measure is difference btwn k price and the fair market value of the property at time of breach

o incidental damages: out of pocket expenses incurred on reliance of ko consequential damages: special, forsseeable costs to non-breaching partyo lqd: valid only if future damages are difficult to ascertain and lqd is a reasonable

estimate of the damages

An encumberance makes title unmarketableLohmeyer v BowerLohmeyer entered into a k to buy a lot. K provided the vendor would give good merchantable title. the lot was burdened by covenant requiring all residences to be 2 stories. The lot violated a local zoning ordinance in addition to the restrictive covenanr imposed on the property which was also violated.Court held that violation of the ordinance and violation of the restriction encumbered the title to expose the party holding title to hazard of litigation; making such title doubtful and unmarketable, thus Lohmeyer entitled to rescind the k.

Rule with respect to covenants: mere existence of a covenant restricting use is an encumberance making title unmarketable

o Here, lohmeyer waived his right to rescind the contract if he found a restriction because he took title subject to encumberances, but the restriction was violated, which creates the right to rescission based on title being unmarketable

Rule with respect to zoning ordinance: zoning ordinance is not a basis for rescission because the existence of a zoning ordiance does not create unmarketable title

o Public records: ordinances are easy to find, just check public record Rule with respect to violation of zoning ordinance: violation of zoning ordinance is grounds

for rescission bc title will be held unmarketable on theory that buyer could not have possibly intended to purchase a violation of law and consequent liability

DUTY TO DISCLOSE AND CAVEAT EMPTOR : duty to disclose defectsCommon law: under caveat emptor, seller has no duty to disclose latent defects to buyer

Seller can remain silent but cannot misrepresent and no fraudulent concealmentModern trend requiring disclosure

Most states require the seller of residential property to disclose known latent defects to the buyer under certain conditions

o If the duty is breached, buyer can rescind or recover damageWhat must be disclosed

Seller must disclose only significant or material defects using an objective standard (i.e. leaky roof or termite are material defects)

Disclosure of seller created conditions: Where the condition which has been created by the seller materially impairs the value of the k and its within knowledge of seller or unlikely to be discovered by a prudent buyer exercising due care, seller’s non-disclosure allows buyer to rescind

Stambovsky v. Ackley: haunted house

51

Page 52: Property Final Outline

P contracted to buy house. After sale, p learned house had rep as being possessed by poltergeists. The seller reported the presence to national and local publications.Court held because the seller reporting alleged haunting in national/local publications, seller was estopped from denying poltergeists existence and AS A MATTER OF LAW, house was haunted and materially impraired value of k and allowed buyer can rescind the k.

Rule: where condition has been created by the seller materially impairs the value of the k and is peculiarly w/ in knowledge of the seller or unlikely to be discovered by the prudent buyer exercising due care, non disclosure constitutes a basis for rescission

Rule to rescinding the k: should be a defect that the seller had a reasonable basis to know about and that the buyer could not reasonably discover by inspection

MORTGAGES AND FORECLOSURESForeclosure: 2 methods

1. Judicial foreclosure (available in all jurisdictions and majority)a. Slow expensive, complex process

2. Foreclosure by power of sale a. Purely private procedure w/o judicial involvementb. Permitted ONLY when authorized by express terms of mortgagec. 2 approaches

i. Most states allow mortgagor to bring suit to cancel ONLY where the bid price is so grossly inadequate as to shock the conscience of the court or if fraudulent or unconsciousable conduct has occurred

1. Here, the mortgagee has reasonable assurance that the sale is unlikely to be nullified by the court due to the shock conscience standard

ii. Few states go farther in imposing duty on mortgagee to obtain a fair and reasonable price under the circumstances. Murphy

1. Good faith standard: may require mortgagee to exert diligent efforts to attract 3rd party bidders or adjourn sale if fair price not offered

a. Arguments against: uncertainty of good faith standard may actually produce lower bid prices bc bidder may bid less to compensate for the risk that the sale may be set aside

i. Potential for increase in overall interest rate charges for mortgagors bc if sale is adjourned, loan repayment is delayed and mortgagee loses interest it could have earned by making new loan

ii. Mortgagee must conduct more that 1 sale to get adequate bid which might incur higher ad and admin costs; extra cost by defaulting mortgagors passed to all mortgagors in form of higher rates

b. Arguments for: lender has no incentive to drive the price high; banks wants lowest bid possible so they can buy back the house for nothing, still have value of the house and still have judgment against the debtor

i. Buyer’s incentive is to get high bid price and get more equity but LENDER CONTROLS sale and this creates an an inherent conflict of interest

52

Page 53: Property Final Outline

Murphy v. Financial Development CorpMortgagor were 7 months in arrears on mortgage payments and lender gave notice of intent to forclose. Mortgagors paid overdue mortgage but failed to pay additional costs. Lender foreclosed on mortgagor’s house and foreclosure sale proceeded only with lender’s rep to bid. Lender bid amount owed on mortgage, 27k, and lender then sold for a large profit a few days later. Mortgagor sued to set aisde forclosure sale but trail court refused bc buyer was a bona fide purchaser for value.Court held that in a foreclosure sale, a mortgagee must exert a reasonable effort to obtain a fair and reasonable price under the circumstances, even to extent if necessary, of adjourning sale or establishing a bottom price which will not be accepted below.

Court imposes a fiduciary duty on the lender so that the bank must act in the best interest of the homeowner by exercising good faith and due diligence in obtaining fair price

o Bank did not exercise bad faith but there was evidence to support finding of failure to exercise due diligence in obtaining a fair price

What should the bank do to satisfy its duty? Advertise the foreclosure, establish appropriate upset price or minimum bid, use any other commercially reasonable methods to generate interest in the property

Damages as result of breach of fiduciary dutyo P cannot get back his house bc there was a subsequent bona fide purchaser for value

If sub purchaser is in good faith, court will protect this persono Bad faith Damages: Trial court awarded difference btwn fmv and price at foreclosure

(but this is only when bad faith is involved) Attorney fee can be collected here: wanton conduct

o Failure of due diligence Damages: difference btwn FAIR value of property and price at foreclosure (fair value might be less than fmv)

No attorney fees

THE DEED3 basic types

1. General warranty deed: provides title protections and contains 6 specific covenants of title that warrant against any defect in grantor’s title

a. covenant of seisin: warrants that grantor owns estate that he is conveyingb. covenant of right to convey: warrants the grantor has right to conveyc. covenants against encumbrances: warrants there are no encumerants on property

(mortgage, liens, easements, covenants)d. covenant of general warranty: warrants that grantor will defend against lawful claims

and pay grantee for any loss that grantee takes by assertion of superior titlee. covenant of quite enjoyment: warrants that grantee will not be disturbed in possession

and enjoyment of property by assertion of superior titlef. covenant of further assurances: grantor promises he will execute documents required

to perfect title conveyed2. Special warranty deed: contains the same 6 title covenants found in general warranty deed but

applies then only to defects caused by acts or omissions of grantora. Afford no protections of acts or omissions of 3rd parties

3. Quitclaim deed: contains no title covenants. Grantor does not warrant he owns the property of he has title that is good

53

Page 54: Property Final Outline

a. i.e. in adverse possession suit. a could perfect title by bringing quite title action but can avoid litigation by asking b to convey title. b is willing quitclaim only.

b. i.e. a wants to buy land in community property state. Property believed to be b’s separate property but he is married. To preclude later claim that land was in community, a can insist that b’s wife quitclaim deed in his favor

c. i.e. used to transfer title following involuntary sale of property (foreclosure)

THE RECORDING SYSTEM a deed is valid and good against the grantor upon delivery without recordation. However, the

recording system provides for priority of title btwn successive grantees. Common law: first in time, first in right. First grantee wins here bc he was the first

grantee and at that moment, when conveyed interest in blackackre, grantor has no interest in blackacre to convey to 2nd grantee, so 2nd grantee loses.

o Common law system controls unless person qualifies for protection under the recording act

Recording system: displaces the common law system: A deed is valid without recording but an unrecorded deed is likely to lose out to a recorded deed if both deeds are from the same grantor to the same property.

o Protects subsequent bona fide purchasers from prior unrecorded interests.o Recordable instruments: deeds, mortgages, wills, leases, options, etco FAILURE TO RECORD HAS NO IMPACT ON TRANSACTION. THE

PURPOSE OF RECORDING IS TO TELL EVERYONE ELSE IN THE WORLD THAT I CLAIM AN INTEREST IN PROPERTY

o ***Valid notarization is required before recording

MAJOR QUESTION TO KEEP IN MIND: WHO IS IN THE BEST POSITION TO AVOID THIS PROBLEM—cheapest cost avoider

Indexes Grantor-grantee: most common index. An alphabetical record of all grantors and grantees, by

grantee/grantor’s name.o How to search title: if a is selling land to b. 1) you want to find out how a got title to

land, so begin by searching the grantee index under his name from the present day backwards until you find the deed c to a. then from d to c. 2) run title down on grantee index to find a satisfactory foot of title- e 3) now switch to grantor index and begin with e upwards until you find a

Tract index: indexes documents by parcel identification number assigned to each tract. Does not exist in most states. Tract index are common where property has been platted by map into various blocks and lots within blocks.

Luthi v Evans (1978)Owens assigned her interests in her 8 leases to Tours. In the assignment, 7 of the leases were specifically described not including the Kufahl lease which was not specifically described in the assignment. Owens by written instrument assigned all her interest in all of her leases in her town to Tours (mother hubbard clause). 4 years later after Tours records the assignment, Owens then sold her

54

Page 55: Property Final Outline

interest in Kufahl lease to Burris. Burris did a title search, but his search did not reflect the prior assignment of the Kufahl lease to tours under the mother hubbard clause. Court held an insufficient description (mother hubbard clause) in the index does not give constructive notice. The assignment from Owens to tours, which did not describe with sufficient specificity, the Kufahl lease, was not sufficient to impart constructive notice to Burris. Burris had no notice, thus he wins.

Court must decide who has the burden?o Courts puts burden on cheapest cost avoider: Tours: incentive to be

specific and describe the land your conveyed properly Burris argues: recording index gave him no constructive notice of the leases.

o Even if the deed was recorded, it would be bad precedent, to hold the person must not only do a title search but locate and read each deed. This would make title search expensive and time consuming. This would be a monumental task, greatly increasing time and expense of title searching, which recording statues did not contemplate.

ACKNOWLEDGEMENT OF DEED and Ineligible Instrument bc of the DefectMessersmith v. Smith (1953): Caroline and Fred are the record title owners. Caroline quitclaims a deed to fred. Caroline then grants a deed to Smith who then transfers his interest to Seale. Seale and Smith record before Fred. In the first deed to Smith, it was signed and acknowledged but there was a mistake. Smith ripped up deed and prepared 2nd deed which signed by both prties but notary was not in person to acknowledge. Notary’s Acknowledgment came over the phone. Race-Notice jurisdictionC F. C Sm (lease); C Sm (deed) – improper acknowledgment; C Sm (deed) – signature notarized over the phone; Sm Se. Sm records lease; Sm and Se record deeds. F records. Court holds the recording was improperly notarized and therefore not properly recorded and does not get protection of the recording act. So fred wins (first tine time, first in right).

The majority rule is that an instrument with a defect on its face does not give constructive notice but an instrument with a hidden defect DOES impart constructive notice

Purpose of acknowledgment is to prevent forgery, but there is no allegation of forgery and absent that, the record should speak for itself***this ruling makes no sense

o Here, the defect could not possibly be known to anyone carefully scrutinizing the record, so its had to see court’s reasoning. Court tries to protect Fred and Carolyn.

Fred argues: Smith recording was insufficient. Needs to be in proper form to get protection from the recording actSmith argues: this was a latent defect in the deed. To figure out if the deed is defective even though you have reason to believe its effective, you must check all acknowledgments on every deed and track down notaries.

After ruling in Messersmith, argue for parties: construe Messersmith Narrowly

Fred Argument: B is protected under the recording act because Messersmith stands for the proposition that A’s recording is inefficient record notice. So if B never saw the recording, he took as a bona fide purchaser.

Smith’s Argument: Public Policy – The policy reasons for protecting B don’t exist, because he would have had notice if he’d done a title search. All C could do would be to try to find out what happened during the notarization. Precedent – Unlike in Messersmith, B purchased after A recorded, so even though it was defective, B should have had notice. Furthermore, Messersmith involved deeds

55

Page 56: Property Final Outline

received by oil speculators under suspicious circumstances, but the court was bound by the facts as found by the trial court.

o Court improperly relied on messersmith in holding restriction invalid. Messersmith holds that you cannot get protection under the recording statute as prior purchaser for value if recording is invalid

o Distinguish messersmith on grounds of purpose of notarization: notarize is for recording. In messersmith, an improperly acknowledge doc does nto give notice.

Here, there was notice. Lawyer said it was restricted

TYPES OF RECORDING ACTS1. Race statute: btwn 2 purchasers, the first to record wins. Notice is not an issue at all..

a. Wild deed: outside the chain of title (your grantor doesn’t record and you go to record, this is invlaid bc your deed is invalid ad insufficient to provide notice): if a complete stranger to the record chain of title records a conveyance (wild deed), conveyance does not give constructive notice bc it is not within the chain of title. Board of Ed Minneapolis v Hughes

i. o to a. o to b. b to c. c records. A records. B 1. A versus c. A wins in a race or race/notice bc c’s deed is out of the chain

of title2. Notice Statute (FL): subsequent bona fide purchaser without notice of a prior unrecorded

transfer prevails over the prior purchaser who has failed to record. THIS IS TRUE EVEN IF THE SUBSEQUENT PURCHASER HAS NOT RECORDED.

a. FL statute: no conveyance…shall be good…against…subsequent purchasers for a valuable consideration and without notice unless the same be recorded according to the law

b. O to a. a fails to record. O to b for 100k. b has no idea of the o to a conveyance. i. B wins regardless of who records first

c. Critical Elements: 1) b lacked notice of prior conveyance to a 2) a had not recorded his deed 3) b paid valuable consideration

d. Shelter rule: a person who takes from a bona fide purchaser protected by the recording act has the same rights as his grantor (protection under recording act of subsequent purchaser extends to all takers from the bona fide purchaser) EVEN IF such takers knows of prior unrecorded conveyance.

i. Protects subsequent purchasers interest in his title by giving him the full value of his purchase in reliance on the records.

ii. O to a. o to b. a records. B records. B to c.1. A versus c. c is not innocent but wins in notice jurisdiction bc b was

subsequent purchaser w/o notice. We protect b’s ability to sell the property so c is protected under SHELTER RULE.

3. Race Notice Statute (CA, NY): subsequent purchaser prevails if he can show that 1) he took a deed WITHOUT NOTICE 2) the PRIOR DEED WAS NOT RECORDED and 3) he RECORDED FIRST

a. CA statute: every conveyance…is void as against any subsequent purchaser…in good faith and for valuable consideration whose conveyance is first duly recorded and as against any judgment affecting the title, unless the conveyance shall have been duly recoded prior to the record of notice of action.

i. A purchaser doesn’t purchase in good faith if he has notice

56

Page 57: Property Final Outline

b. O to a. a does not record. O to b for 100k. b has no clue about the o to a conveyance. A then records. Then b records.

i. A wins over b even though b lacked notice (good faith) bc A recorded before him.

c. Wild deed: outside the chain of title (your grantor doesn’t record and you go to record, this is invlaid bc your deed is invalid and insufficient to provide notice): if a complete stranger to the record chain of title records a conveyance (wild deed), conveyance does not give constructive notice bc it is not within the chain of title. Board of Ed Minneapolis v Hughes

i. o to a. o to b. b to c. c records. A records. B records. 1. A versus c. A wins in a race or race/notice bc c’s deed is out of the chain

of title

***if nobody has recorded, common law principle of FIRST IN TIME applies except in a notice jurisdiction when the subsequent purchaser lacks notice.

WHEN LOOKING AT A RECORDING ACT, What to do?1. Presumption of 1st in time has superior title

a. If nobody has recorded, common law principle of in time applies except in a notice jurisdiction when the sub purchaser lacks notice

2. Is subsequent purchaser protecting by a recording statute if not 1st in time?

TYPES OF NOTICE To be protected under a notice or race notice act, a purchaser must be without ACTUAL

OR CONSTRUCTIVE NOTICE of any prior unrecorded interests at the time the purchaser pays the consideration

1) actual notice: real, actual knowledge of the prior unrecorded transaction. 2) constructive notice:

o 1) record notice: If an instrument is validly recorded, every subsequent grantee has constructive notice of it and so cannot be a bona fide purchaser.

Wild deed: outside the chain of title: if a complete stranger to the record chain of title records a conveyance (wild deed), conveyance does not give constructive notice bc it is not within the chain of title.

o 2) inquiry notice: in most states, subsequent purchaser has an obligation to make reasonable inquires and is charged with knowledge of what those reasonable inquiries would reveal

Leslie Recording Act Hypos:O to a. o to b. b to c. a records. B records. C records. A versus c

notice: c race notice: a

o to a. o to b. b to c. c records. A records. B records. A versus c. race: a (wild deed) notice: c race/notice: a (wild deed)

o to a. o to b. b records. A to d. a records. D records. B versus d.

57

Page 58: Property Final Outline

notice: b race notice: b

o to a. o to b. a to d. a records. B records. D records. B versus d. Notice: d Race: d (bc a records 1st and d is in a’s chain of title) Race notice: d

O to a. a takes possession of property, lives in home. O to b. b records. Notice: a (b had inquiry notice) Race/notice: a (b had inquiry notice)

O to a. o to b with actual knowledge. B records. B to c. c records. A records. A versus c Notice: c Race notice: c Race: c

O to a. o to b. a records. B records. B to c. a versus c Notice: c (shelter rule) Race: a Race/notice: a

O to a. o to b. b records. A records. B to c. a versus c. Race: c (b recorded 1st) Race notice: c (record notice of A’s recording does not matter bc B recorded 1st) Notice: c (b recorded 1st)

Wild DeedBoard of Education of Minneapolis v. Hughes (1912)Hoerger owned a lot which was vacant. Hughes, grantee, bought the lot. Mr. Hoerger did not fill in grantees name on deed. Hughes filled in his name in the grantee slot before recoding the deed. Mrs. Hoerger quitclaimed deed to Duryea and Wilson which was recorded after Hughes deed. Duryea coveyed to Board of Ed who recorded before everybody. Minn has race notice statute.Ho Hu, with no name or date. Ho D&W; D&W BOE; BOE records. Hu inserts his name; Hu records. D&W records.Court holds 1) Hughes deed is effective at the time the name was inserted (hue was a subsequent purchaser) and 2) Board of Ed’s recording, although recorded 1st, gave Hughes in insufficient constructive notice of the prior conveyance by his grantor, bc the deed was a complete stranger to the chain of title. Therefore, Hughes was a subsequent purchaser w/o notice who recorded 1st.

Issue 1o When grantor receives and retains consideration and delivers deed in condition

described to purchaser, authority to insert his name as grantee is presumed on ground of implied authority

Issue 2: o When Hughes deed was recorded, there was no record deed Hoerger to Durya. Hughes

would look up Hoerger, would not have found a conveyance from Hoerger to D and W and wouldn’t have found the conveyance to the Board of Ed.

Thus the deed is wild bc its not inked up in chain of titleo When board recorded its deed w/o insisting Durya record his deed, the Deed was wild

and provides no notice to Hughes

Hypo: Minn repeals race notice and replaces with notice statute. Argue for Board of Ed

58

Page 59: Property Final Outline

1. Argue that Hughes was 1st and the board was a subsequent purchaser without notice and it doesn’t matter if the Board records

o Not required to record under notice act 2. Argue court should overrule the case to protect bona fide subsequent purchaser. 3. Argue don’t overrule court: Distinguish case

o Argue the problematic part of the holding is DICTA: when Hughes put his name as purchaser, it wasn’t outcome determinative

In Board of Ed, it was a race notice act. Court found Hughes was grantee when he inserts his name in deed, therefore, he is subsequent purchaser (day he purchases is irrelevant). Board is first in time.

We argue Hughes would have won either way. Even if Hughes was 1st purchaser and board was 2nd, board has no

notice and question is who records first. Hughes records 1st bc Board deed is wild and insufficient

****either way, Hughes wins if he is 1st in time or 2nd. Therefore, courts holding is wild deed is not sufficient to give notice its dicta****

o If we are in notice juris, now it matters where Hughes is. If Hughes is 1st, then board was purchaser w/o notice and wins. If Hughes was 2nd, board recording was wild and is no notice, therefore Hughes win

Thus, where Hughes is purchaser is outcome determinative in notice jurisdiction

Expanded Chain of Title; deed from common grantor and reciprocal implied covenantsGuillete v Daly Dry Wall Inc (1975)Gilmore, a developer and subdivider, conveyed a lot to Guillete under a recorded deed of the lot in a subdivision plan that restrictrf the lots use to single family dwellings only and the deed recited that “the same restrictions are hereby imposed on each of the lots now owned by the seller. This was effective to impose the single family residence restriction on all of Gilmore’s remaining lots. Later, Gilmore conveyed a restricted lot to Daly under a recorded deed that made no mention of the restrictions but did refer to the subdivision plan. Daly sought to build apt complex and Guillete sued to enjoin from violating use restriction. there was the restriction in Guillete’s deed and the restriction purports to apply to all other lots owned by the seller.Court holds Daly acquired the lot with constructive notice of the restriction even though the restriction was not in the chain of title from Gilmore to Daly. Daly’s deed referred to a recorded subdivision plan and deed to Guillete referred to the same plan. Guillete rule: requires a purchaser to expand his search of title to include all conveyances made by the grantor of other property w/in his subdivision in order to be certain that the grantor did not burden his remaining property with a use restriction contained in a deed to a third party

Who was the cheapest cost avoider?o Daly. Inquiry notice. Look around, see all the houses in the subdivision and inquire as

to whether these is a restriction. Buy title insurance

o Owners could not have protected themselves. *****Guillete does not apply when there is no express reciprocal negative

easement/restriction

59

Page 60: Property Final Outline

Daly argue: there were no restrictions in the chain of title. thus, they had no notice of the restriction when they did there title search. We are a subsequent purchaser for value without notice and thus are not bound by the restriction.

We want title search to be quick and efficient and this is a heavy burden on subsequent purchasers to do title searchers that are time consumer and expensive

This is inconsistent with objectives of an efficient recording systemGuillete argues: the restrictions were imposed on each lot now owned by seller. When they bought their lot, they accepted the restriction and developer was restricting all of the lots he still owned. Developer still owned that lot at the time, thus there exists a writing that says restriction is imposed on their lot as well.

Want to protect property value as residential neighborhood Public policy: We should follow guillete bc court recognized importance of rotecting other

properties in subdivision bc it wasn’t their fault that restrictions were not uniformally imposedo Property interests are worth protecting bc developer did wrong, not people livingo ****protect 3rd parties that cannot protect themselves

People Protected by Recording System

Actual Notice of Right of 1st Refusal; partial payment by later buyerDaniels v Anderson (1994)Daniels purchased land from Jacuala and k gave Daniels right of 1st refusal if Jacula ever sold adjacent parcel. The k of sale was not recorded and when Daniels received his deed, it did not include his right right of 1st refusal. Jac contracted with Zongrafos for the adjacent parcel for 60k under an installment k. Zog put downpayment of 10k and pay another 20k in installments. Daniels wife told Z about Dan’s right of 1st refusal and Z then paid the remaining 30k, received the deed and recorded it.Daniels sued for spec performance of his refusal option.Court holds that Zog was protected by his lack of notice only for payments made before he received notice and concluded than an award of property to Daniels w/ a requirement that Daniels reimburse Zog for the total of his payments to Jacula was best method to deal with the mischief cause by Jac.

Z has actual notice of right of refusal and ignores it. Even though Daniel is in the best position to avoid by recording his right of 1st refusal, he is still the real property owner and protected.

o Common law: first it time, first in right: Dan wins Recording Act: purpose is to protect sub purchaser of value w/o notice, but at the point they

receive notice, they are not protected anymore Pro Tanto Rule: Designed to protect the buyer’s payment prior to notice. Either by: (a) awarding the

land to the holder of an outstanding interest and award the buyer the payments that he made (b) award the buyer a fractional interest in the land proportional to the amount paid prior to notice, or (c) allow the buyer to complete the purchase, but to pay the remaining installment to the holder of the outstanding interest.

Daniels (prior purchaser) Argueso Best case scenario: get title to property and pay nothing

I was first in time, sub doesn’t become purchaser until he pays final consideration. By final considertion, I recorded and therefore there was notice to sub.

60

Page 61: Property Final Outline

o Pro Tanto Rule: pay sub 30k (amount he paid w/o notice). Even thouse this adds 30k to prior’s original price of 40k, he still wins bc property is worth 100k

Sub purchaser (zog):o Theory of equitable conversion: when person makes k for sale and puts down money,

equitable title is transferred to him. At this point, he is a bona fide purchaser Thus even though sub doesn’t have legal title, he has equitable title and sub

changed his position on reliance on the equitable conversiono Distinguish Daniels

GREAT ARGUMENT: in daniels, it was actual notice. Daniels is limited to actual notice of the downpayment. Here, this is constructive notice. Its very burdensome and expensive to require person who pays on an installment plan to keep do title search each time before pays another installment

Daniels isn’t controlling: court holds that trial court abuse didn’t abuse its discretion in applying the equitable standard. There are other ways to fix the problem by pro tanto rule

Alexander v AndrewTom and Charles owned a house as TIC. Tom conveyed his interest to Mary and then a week alter conveyed the same interest to Charles in return for 1k and Charles promised to care for him for remainder of his life. Charles, unaware of prior deed to Mary, recrded and a few months later, mary recorded. Charles cared for him until Tom died and Mary claimed her interest. Charles claim to be a prio purchaser for value w/o notice of Mary’s recorded interest.Court held Charles protected up to extent of 1k paid before Mary recorded. Value of his care of Tom after Mary recorded was expended w/ constructive notice of Mary’s recorded deed.

Lewis v. Superior Court (1994)Lewis contracted to buy house from Shipley for 2.3mm. A few days before they closing on feb 24, Fontana recorded (BUT IT WAS NOT INDEXED) a lis pendens (lawsuit affecting title in the property—once recorded, it’s a lien on title and prior purchaser takes subject to the clain) on the property). On feb 28, Lewis put 350k down with a note for 1.95mm, received the deed and recorded it. The next day, lis pendends was indexed and at this point, now served as constructive notice to all sub purchaser. In March, Lewis paid Shipley the blance due on their notice (1.95mm) and They spent 1mm on renovations. They were then served in Fontana’s lawsuit and received notice about the lis pendens. Court held Lewis were bona fide purchasers protected by the recording act. Court held where little of purchase price remain unpaid OR where substantial improvements had been made prior to notice, equity demanded Lewis retain property

Argument for Lewis:o ****in an installment k (or one which is not financed by 3rd party), if constructive

notice before each payment threatens him with loss of title, he will have to undertake a title search before each and every payment—360 title searches for a 30 yr note****such an obviously absurd result is fundamentally contrary to purpose of recording statutes

o traditional rule provides an anamlous and unwarranted benefit to cash buyers (or those who finance their purchase w/ 3rd party lender rather than by installment or seller financed mortgage)

o recording act protects installment payments when money is put down

Inquiry Notice

61

Page 62: Property Final Outline

Inquiry notice is based on the facts that would cause a reasonable person to make an inquiry into the possible

Duty to inquire: a subsequent purchaser has an obligation to make reasonable inquiries and is charged with knowledge of what those reasonable inquiries would reveal.

o Examples: Notation in a deed (Harper) or possession (Waldorff)

Duty to Inquire when recorded instrument references an unrecorded instrumentHarper v Paradise (1974)In 1922, Susan harper conveys warranty deed to daughter in law Maude as a life estate remainder to Maude’s children. Maude loses deed. Susan harper dies. In 1928, Maude records deed from grantor’s heirs who execute an instrument meant to re-deed the property to maude. This deed refers to the lost deed. Maude executes a security deed for Thornton. Loan goes in default and Thornton forecloses on property. actual possession of thornton chain of title. unbroken chain of title from thornton to paradise who aquires title in 1955. Remainderman of original deed finds original deed in old trunk and records in 1957. Maude dies in 1972 and her life estate ends and remainderman sue to recover possession.1922: Susan Maude for Life Maude’s Children. 1928: Susan dies. Heirs Maude (conveyed because the 1922 deed went missing). 30s: Maude Thornton Paradisesthe recitals in the 1928 deed put any subsequent purchaser on notice of existence of the earlier deed and this raises a duty in any subsequent purchaser to inquire of Maude to determine the details of the unrecorded deed referred to in the 1928 deed; therefore, the Paradise had constructive notice when they acquired title and are afforded no protection from the recording act.

The paradise had never made even the most modest inquiryo You had to at least ask Maude. However, even if Paradise asked Maude about the

contents of 1922 deed, she has incentive to say it was the same as the 1928 deed. Also, it was long ago, she might not know

Argue for Paradise:o We are subsequent bona fide purchaser for value w/o notice who recorded.

Court. NO. Paradise should have done a diligent inquiry into the lost deed bc recorded said its replicating another deed.

Adverse possession: no adverse possession here. o Paradise: we had long term possession. Holding against us undermines the objectives of

adverse possession doctrine.o Court: SOL didn’t begin running until life tenant dies

Majority Approach: Adverse possession SOL doesn’t toll until remainderman interest becomes possessory. Remainderman have property interest but its not possessory until life tenant dies. By the time Maude dies, the SOL for adverse possession has not been satisfied

Policy argument: SOL not re-setting best effectuates purpose of adverse possession doctrine

Duty to Inquire of Whoever is in Possession of Property at IssueWaldorff Insurance…Inc v Eglin National Bank (1984)Waldorff enters into purchase agreement for apt and pays Choctaw 1k (total purchase price is 23k) and Waldorf begins occupancy of the unit. Choc executes note and mortgage in favor of the bank which includes Waldorf’s until and then another mortgaged which included Wal’s apt. Choc never told the bank he sold to Wal but Wal remained in possession. Choc agreed to consider the purchase price of Wal’s apt paid in full in return of cancellation of debt owed by Choc to Wal. Choc gave him quitclaim deed and Wal recorded it. Bank then brings foreclosure against choc and Wal’s apt is included.

62

Page 63: Property Final Outline

Court held bc Wal was in possession, the bank was on inquiry notice that Wal might have a prior unrecorded claim. The purchase k btwn Choc and Wal vestsed equitable title in Wal; therefore, the interests acquired by the bank would be subordinate to Wal’s equitable interest if the bank had either actual or constructive notice.

The bank had constructive notice of Waldorf’s interest bc waldorf was in actual possession of the unit that was open, visible and exclusive. Possession puts the bank on inquiry notice.

Cheapest cost avoider: here, its Wal. o Why does court put burden on bank?

Wal who could have avoided the problem by recording BUT its unlikely the k of sale was notarized and capable of recording. Choch prob didn’t want to notarize and record the k to avoid any clouds on title until the purchase price is paid

Developer don’t want prospective buyers to notarize k bc it could prevent other buyers from looking at it due to cloud of title bc of buyer’s breach

What should bank have done? Obtain certificate of estoppel from every tenant stating they are not a party to any side agreements involving the land otherwise there will be litigable interest.

Argumentsa. Party 1st in time will prevail

i. Wal is 1st: equitable title: on signing the k, buyer gets equitable title thus Wal has equitable title

b. Bank is subsequent purchaser for value w/o notice i. Bank argues: k for sale wasn’t recorded. They did their title search and see no

evidence that anyone but Choc owns the apt1. Waldorf is cheapest cost avoider here: could have recorded k 2. Public polcy argument: we want to encourage parties to record the k of

sale of property—this is behavior that we want to encourageii. Wal argues: I took possession and was living there

1. Inquiry notice: you must go to ever single person living in every condo and ask whether they claim an ownership interest

PRIVATE USE LAND CONTROLS: THE LAW OF SERVITUDES

EASEMENTSAn interest in land that entitles the holder to use land owned or possessed by another person.

Classification of Easements Affirmative easement: right to enter or perform an act on the servient land. Permits a person

to use the servient estate in a specified manner. Negative easement: Only the right to prevent specified use of the servient estate but congers

no right to use the servient estate.

Types of Easements Easement appurtenant: easement for the benefit of a particular parcel of land and whoever

holds that land is the only one who can use that easement Easement in gross: not attached to piece of land. Designed to deliver a personal benefit rather

than to benefit a landowner on the land.

63

Page 64: Property Final Outline

When Ambigous, Construe Easement as Appurtenant1) appurtenants are easier to eliminate bc owner is easier to locate 2) are more likely to create value 3) not useful to people who move and 2) more likely to be intended than in gross

Creation of easements:

1. Expresse Easement (Easement by Grant) must be in writing Easement by Grant: most easements are created expressly by deed or other grant. Because an

easement is an interest in land, its created is subject to the SOF, which requires a writing signed by the grantor. ******if easement in writing

o By Reservation: grantors sometimes convey land and, in the same deed, purport to “reserve” an easement in favor if the grantor or 3rd party

1. In favor of grantor: Modern Courts allow say reservation of an easement in favor of the grantor is valid.

2. In favor of a 3rd party: majority of courts treat reserved easements in favor of 3rd party void. . In states that do not allow it, it is necessary to use 2 conveyances instead of one to create an easement in favor of a 3rd party

i.e. 2 conveyances: a owns blackacre wishes to convey the land to b but reserve an easement in the land for c. a must convey balckacre to c, who would then convey to B, reserving an easement in favor of himself.

In minority of jurisdictions like Cali, the reservation may be reserved in favor of 3rd party(Willard, RE)

****if easement is in writing, does burdened party have NOTICE OF IT

Willard v. First Church of Christ Scientist (1972)McGuigan owned 2 lots 19 and 20. She permitted the use of lot 20 for her church’s parking. She sold lot 19 to Peterson. Willard expressed interest in buying lot 19 and 20. Peterson signed a deposit receipt for the sale of the 2 lots even though he didn’t own lot 20. Peterson offered to buy lot 20 from McGuigan and she said she would sell it subject to church’s continued use of parking. Her lawyer drew up an easement of the deed. McGuigan sold the property to Peterson at a reduced price due to the easement and recorded the deed w/ easement. Willard then bought the lot from Peterson and got the deed and recorded it but his deed had no mention of easement. Peterson told Willard Church would want to use lot 20 for parking but didn’t tell him about the easement. Willard argued the Easement was invalid bc Church was 3rd party to the agreement btwn McGuian and Peterson and Mcguigan can reserve an easement for herself but not in favor of a 3rd party.The court holds the reservation may be reserved in favor of 3rd party bc the common law rule frustrates the clear expressions of grantor’s intent and delivers windfall profits to people like Peterson (who presumably pays less for prop w/ easement)

Court abandons majority rule

Church argues: the easement is in the chain of title, maybe not your deed, but still in the chain of title and your bound by the easement bc of record/inquiry notice

Also actual notice/inquiry bc Peteson says Church will want to park there What should have McGuigan’s lawyer done knowing the common law rule

1) Use 2 deeds. McGuigan transfers title to the church and church transfers title to Peterson retaining an easement for parking

2) defeasible fee to Peterson (so long as Peterson all Church to park on Sundays

64

Page 65: Property Final Outline

o if Peterson violates condition, he loses rights to propertyo why is easement more efficient?

if an easement, church could sue for an injunction but property is still Petersons buying easement is easier than buying defeasible fee bank less likely to provide mortgage for defeasible fee bc it will have no control

over mortgagor using property for non-church purposes

Argue for Baptist who is the nxt church who moves in : Presumption: when grant is ambiguous, presumption in favor of easement appurtenant

o easement appurtenant: attaches to the interest of land in dominant parcel and runs to subsequent purchasers

grantor’s intent was to give an easement for any church in that location and not any named church

condition in the easement: “only so long as used for chuch purposes” determinable easementArgue for Scientist church who was 1st church who moves out:

Easement in gross: personal to our entity Grantor’s intent: this is grantor’s church and she wanted the easement to benefit her church

only and willing to sell only if her church benefittedArgue for Willard who wants easement done

Argues for in gross: the condition doesn’t exist anymore Determinable easement which terminated when church left property Grantor’s intent: “only so long as property for who benefit easement is given

Grantor only meant so long as property is used for her church’s purposes

2. Easements by Implication (1) prior use and 2) necessity) 1) Easement Implied from prior use

o Elements for establishing an easement implied from prior use: these are the principal factors that tend to establish an inference that parties intended to create an easement

1) common owner: prior to division, the quasi servient estate and the quasi dominant estate must be owned by the same person

by grant or by reservation? if owner and grantor retains the quasi servient estate (burdened part), the

implied easement is by implied grant if the owner and grantor retain the quasi dominant estate (benefitted

part), the implied easement is by implied reservation 2) reasonable necessity: the prior use must be reasonably necessary for the use

and enjoyment of the quasi dominant estate. Van Sandt. courts will find reasonably necessity if it would be costly or difficult to

use the dominant estate w/o the easement or if the price paid for either the dominant or servient estate reflects the existence of the easement

**I wouldn’t not have bought land w/o easement. As part of purchase, I thought I was getting land

3) continuous use: prior use must be continuous, not sporadic 4) intended continuation: the parties must intend, at the time of division, to

continue the prior use price paid is helpful here bc parties who intend for quasi easement to

continue are likely to settle on a price that reflects the value of the easement

65

Page 66: Property Final Outline

5) existing use: the prior use must be existing use at the time of division. If there wasn’t, no no reason to think parties intended continuation of a non existing use.

6) Apparent use: does not meant it must be visible, prior use is apparent if it could be detected or even inferred from a reasonable inspection of the premises

Van Sandt v. Royster (1938)Bailey owned 3 adjoining lots. She built house on one of lots and ran an underground sewer line across the other 2 lots to a muni sewer. She later sold all 3 lots to separate owners under deeds that made no mention of sewer line. Van Sandt bought tract of land closest to main sewer which had the underground connecting drain running through his land. Another tract of land was sold to Royster which shared the sewer connection and then a 3rd tract which adjoined the 2nd tract was sold. All tracts shared sewer connection but none of the deeds mentioned underground pipes. Van Sandt’s home flooded due the sewage line which rain underneath all 3 homes. Royster refused to cease draining and discharging sewage and p brought suit for an injunction.Court found an easement implied by prior use and court rejects older view that an easement implied from prior use requires strict necessity.

When Bailey sold lot 19, an easement was created: it would be inconceivable that Bailey would sell lot 19 w/o getting an easement

original purchaser had knowledge of the sewer. Finally, court charges the subsequent purchaser (van Sandt) with inquiry notice bc prior purchaser would have inferred the existence of sewer by noting existence of plumbing fixtures and could have employed a plumber to detect the actual location of the sewer

To get an easement implied by prior use: 1) must find evidence of prior use or quasi easement on property

o here, grantors intent of implied easement bc subsequent purchasers plugged into sewage liens and know its there

2) parties intend upon severance of land that use of the easement would continue ***how necessary is the easement to the use and enjoyment of land

2) Easement implied by necessity (2nd of easement by implication)o an easement is implied from necessity ONLY when a common owner divides his

property in such a way that one of the resulting parcels is left without access to a public roadway.

easements by necessity are permitted only for right of way—ingress and egress--btwn land locked parcel and public road

o Required Elements 1) Common grantor: an easement by necessity can be created ONLY over

property owned by the person who also own the landlocked parcel and who divided the property to create the access problem. Othens v Rosier.

2) Necessity at time of severance—not prior use: necessity must exist at the moment the property is divided. No prior use is needed to establish an easement by necessity.

3) Road way is a necessity, not a mere convenience Degree of necessity required:

o 1) most court, like othen, require strict necessity: no reasonable person would purchase property w/o easement

66

Page 67: Property Final Outline

was there access by boat? If yes, but any reasonable person would expect access by land and not boat

why didn’t you bargain for easement when you bargained for land

***not necessity when you created the necessityo 2) some courts have granted an easement by necessity where

access to the land exists but it is claimed to be inadequate, costly, or difficult

o 3) few courts say necessity will not be implied if tract has access to navigable water

Duration: easement by necessity lasts as long as the necessity exists. If the necessity is removed, easement is terminated.

Othens v Rosier (1950)Hill owned large parcel of land, which he sold in pieces over time in 1890’s. In 1896, Hill sold 100 acres to Rosier’s predecessor possibly land locking the other parcels. Hill then sold 60 acres to Othen’s predecessor in 1897. Othens then acquires those 60 acres in 1904 and then acquires another 53 acres in 1913. In 1924, rosiers acquires 100 acres and then another 16 acres. Othens had no direct access from his land to the highway and for many years, Othen and his predecessors have been crossing over Rosier’s land. To avoid flooding, Rosier created levee and levee made the driveway so muddy depriving Othen of use of the roadway. Othen claims easement by necessity and prescription.Court holds no easement by the necessity bc there was insufficient proof of whether at time of severance of the parcels (Hill severing), Othen’s parcel was landlocked and Hill impliedly reserved an easement. Further, no prescriptive easement bc easement was permissive.

Court here requires strict necessity

Arguments for X who buys from common grantor and claims to have easement implied by nec: this is easement implied by necessity bc its reasonable for me to assume that o is going to let

me have access to his parcel when he sold it to me 2 major reasons for necessity:

o 1) parties intended easement by necessity (good inference that parties expectated than an easement would be created with the grant of the land)

o 2) bad policy to have land landlocked bc it used to its highest value Effectuate intentions of the parties justifies finding a right of easement for x

o X’s property worthless w/o easement o Not enabling parties to use land to its best use if no easement

X argues this is an easement by implication bc by granting me the land, he also implied to grant me an easement

Arguments for common grantor who sells parcel and claims to have easment implied by nec: O argues: implied by necessity, that he reserved the right of way when granting land to x X argues: no implied by necessity bc o is in the best position to know there is a landlocked

parcel and o is in better position to prevent this conflict

3. Easement by Estoppel: License: simply PERMISSION to enter the licensor’s land. Licenses may be oral or written

and are revocable at any time unless the licensor makes the license irrevocable, either expressly or by his conduct

Irrevocable License

67

Page 68: Property Final Outline

o Easement by estoppel: if a licensor grants a license on which the licensee reasonably relies to make substantial improvements to property or easement, equity requires that the licensor be estopped from revoking the license

Without Holbrook: licensee must get the easement in writing, which is easier than licensor stopping the improvements

Shepard(p.680): neighbors and close friends don’t need to reduce anything to writing.

Henry(p.680): there should always be a writing and no easements by estoppel bc SOF applies to bar creation of irrevocable oral license

o Duration: license is made irrevocable through estoppel and continues to exist until the reasonable expectations of the parties have been satisfied

Required Elementso 1) express permissiono 2) must show REASONABLE reliance (and investment) on the license

investment on the easement itself or the dominant estateHolbrook v Taylor (1976)Taylor buys adjacent land to Holbrook. Holbrook give permission to Taylor to use his road to move their stuff in and to get to his property. then with Holbrook’s knowledge and w/o any objection, Taylor improved on the access road by widening it and also used the access road to construct a substantial single family home. Holbrook then blocked the road w/ a steel cable.Court holds easement by estoppel:--holbrook is estopped from revoking the licensewhen landowner or previous landowner grants permission to adjoining landowners to use an license on land and watches while the party relies on the license and invests substantial money on reliance on this representation of use of the license, original landowner is estopped from revoking the license and it becomes an easement. Reliance on easement was: 25k construction of house, improvement of premises and maintenance of roadway easement

Argue for estoppel: o I never had a right, never negotiated the right, Holbrook was nice to me and allowed me

to drive thru my property, I invested money on relianceo Its not fair for licensor to stand by and watch licensee expend considerable money

and effort on reliance of the licene, then rescind the license Licensor is in best position to prevent these reliance expenditures

Even if I negotiate for easement, he has unfair bargaining power bc he knows I need it Its insulting for neighbors to gets something in writing; we just want to act cordial

Argue against estoppel: I didn’t have notice that there was reliance on the license and I didn’t reasonably

foresee reliance bc I was just trying to be a nice neighboro This penalizes good neighbor who fails to say no until the improvement is done

Taylor was in a better position to avoid this conflict bc he needed the easement to build the house so he should have negotiated for it

There was no permission or easement in writing:o Henry(p.680): there should always be a writing and no easements by estoppel

bc SOF applies to bar creation of irrevocable oral license

4. Easements by Prescription Prescription is analogous to adverse possession. Easements are NOT POSSESSORY

INTERESTS so an easement cannot be acquired by adverse possession, but adverse use for a sufficient period of time can ripen into an easement by prescription

68

Page 69: Property Final Outline

Elements of prescriptive useo 1): adverse use under a claim of right that is (go to 2)

use of another’s land must be ADVERSE and NOT with permission of the owner

o 2) open and notorious and adverse use must be conducted so that the use may be discovered by an

reasonable inspection It cannot be carried on in secret or carefully concealed

o 3) continuous for the prescriptive period if the adverse possessor continually asserts his claim of right by making

whatever use is consistent with the nature of the claimed easement, even if the use is sporadic

adverse use does not need to be used constantly o **note: exclusive use is not required here

PERMISSION=NO PRESCRIPTIVE EASEMENT o ACQUIESCENCE (such neglect to take legal proceedings for such along time as to

imply abandonment of a right)=PRESCRIPTIVE EASEMENT. OTHENSo In othens, there was a gate and that was considered permission. court says the

your character and actions can show permission

See Othens v Rosier (1950)Court holds no prescriptive easement by Othen’s use.

Court finds use permissive but only focuses on fact that there were gates on the servient parcel If the use had been exclusive, Othen might have had an adverse possession claim.

o However, requiring exclusive use for a prescriptive easement abolishes the different btwn it and adverse possession.

***based on these facts, we could say that Rosier acquiesced (prescriptive easement) to Othens use and did not give permission (no prescriptive easement)

SCOPE OF EASEMENTS***If easement is created, always ask whats the scope involves 2 questions

o 1) how extensively and intensively may the easement holder use the easemento 2) to what degree may the owner of the servient estate use or interfere with the

easement ***parties intentions control: the overriding principle in determining the scope of an

easement is to identify and uphold the parties intentionso 1) how was easement created (grant, implication, or prescription)

i.e. scope of necessity is congruent with the necessity i.e. scope of prescriptive easement: no reason to assume any greater or different

use was intended by partieso 2) how conditions have changed to affect the originally intended useo 3) what changes in use were reasonably foreseeable by the parties

if an easement specifies a specific location or one has been agreed to by parties conduct, the location is permanently fixed unless parties agree to change

this rule does not prevent modification to the easement that impose no additional burden on servient estate

69

Page 70: Property Final Outline

o 4) what changes are necessary to achieve the intended purpose of easement? Enlargement of dominant estate: easement cannot be used for the benefit of

land that is not the dominant estate. Brown v Vosso 5) whether a changed use imposes an unreasonable burden on servient estate?

GENERAL RULE: dominant estate may be divided and that each part of the divided estate is entitled to enjoy the easement, but this rule is subject to the important limit that the resulting increased burden on servient estate must be within the original contemplation of the parties

Whether division of the dominant estate is normal development, whether division is foreseeable and whether the burden on servient estate is substantially increased

Brown v Voss (1986)Voss acquired Parcel A in 1973. Brown bought parcel B in 1977 and parcel c in 1977 from 2 different owners. Parcel b has an appurtenant easement over parcel a. The previous owners of parcel c were not parties to the easement grant that parcel b had over the land of parcel a. brown began constructing a single family home on both parcel c and b with Voss’s knowledge. voss sought to bar brown’s sue of the easement in 1979 by which time Brown spent more than 11k in developing the property. As a general rule, an easement appurtenant cannot be extended to other parcels without consent of the servient owner, thus the use of the easement by Brown to benefit parcel was a misuse. However, here, the owner of dominant estate did abuse his rights to the easement because he didn’t materially increase the burden on the easement (no increase in volume or type of traffic or other burdned on Voss’s servient estate)

Court holds the trial court did not abuse in discretion in denying the servient owner an injunction (Broad discretion to the trial court – wasn’t abuse of discretion.

Estoppel: important to the court was that there was estoppel – seemed unfair to allow the owner of the servient tenement to allow the construction and then block it.

o court thought the owner of the servient tenement was acting unfairly. Also, court believed the parcel would be completely worthless – landlocked.

One way to read this case: extension of the easement to a parcel owned by the person who had the easement already.

Arguments easement should be extendedo Expansion of easement was reasonably foreseeable. Once you don’t object, the use

can change and your locked in,the servient owner is not injured financially by the dominants owner’s use, dominant owner acted reasonably, all dominant owner wanted to do was build a house than extended onto another parcel. Servient owner just sat and watched Brown build the house and after 11k, the servient owner tries to stop the building of the house, my house was landlocked, he was cheapest cost avoider, court should not issue a injuntion to stop building, more likely to require damages, if court holds for otherwise, I will pay a lot more for use of easement bc he is in position to extort more $, here trial court didn’t want to issue injunction bc felt servient owner stood by an did nothing while owner of dominant parcel spent great money thus it would be in equitable for him to lose his investment, policing this injunction is not possible

Arguments against Easement extensiono This is expanding the scope of the easement and there is a greater burden on my land,

court should find you cannot expand scope of easement Any use which is an

70

Page 71: Property Final Outline

unreasonable expansion is a misuse of the easement, I immediately objected to use of the easement before they expended money, their land was not landlocked, dominant owner refused to negotiate, he knew he needed this and was in best position to avoid this conflict

***Voss should have provided for automatic termination of easement of dominant estate uses it for benefit of non-dominant estate

More arguments against/for easements (being extended or given to another party who is not dominant estate)

Jerry (does not want easement extended) argues stranger has no express easement; doesn’t not make sense that trespass is ok when there is no damage to servient property.

o Jerry: Distinguishable from Brown – extension on the easement the use in Brown didn’t increase the burden on the easement.

o Jerry argues Brown is about: estoppel. Idea that an owner of a servient tenement can’t sit there and watch someone rely on this implicit permission to expand on the scope of this implicit easement and then pull the rug

Newman (wants easement extended) counter: court would you police an injunction like this. Courts don’t like to give injunctions that are tough to police. We like injunctions that solve the matter once and for all.

o cites Brown. No additional burden on the easement. No injunction. Jerry argues there’s a burden. I lose if he’s not materially increasing the burden on the

easement; but now it’s different. It’s increasing the burden on the land and benefiting the dominant parcel.

Jerry’s argues no estoppel, no reliance: I didn’t see it. Looks like Jerry has a strong case if the resort is built on the top;

Newman Counter: (use Brown’s lawyer tactic): makes the court feel sorry because of his investment.

Moving an Easement:

Common Law: Servient owner cannot relocate the easement unilaterally.Restatement: Servient owner can move it unilaterally if he pays for it himself and it does not significantly lessen the utility of the easement, increase the burdens on the owner of the easement in its use and enjoyment, or frustrate the process for which the easement was created.

Termination of Easements

Easements can be terminated in a number of ways:

(1) Release in writing.

(2) Expiration if the duration of the easement if it was limited.

3) defeasible easement that ends upon occurrence of event or condition

4) When the necessity that gave rise to an easement by necessity ends.

(4) Merger if the easement owner later becomes the owner of the servient estate.

(5) alternation of the dominant estate so the easement can no longer be used.

(6) Abandonment of easement by manifesting a clear and unequivocal intent to abandon the easement

71

Page 72: Property Final Outline

7) termination by condemnation if govt exercises eminent domain power to take title of servient estate for a purpose inconsistent with continued existence of the easement

(8) cessation of purpose: the purpose of an easement has completly ceased

9) termination of prescription: servient owner physically prevents easement from being used for the prescriptive period, easement is terminated

Easement Creation Checklist 1) when someone claim easement right, is there an easement in writing?

o 2) If there is an easement in writing, does the burden parted have notice of it (recording: constructive notice, actual notice)

o 3) even if there easement in writing, what’s scope (how far does it extend, what does it mean)

4) courts willing to find easement in situations with no writingo 5) if neighbor is nice to give license or implied reservation or express reservation

6) Does neighbor relies on the license and servient owner fails to stop, this revocable license evolves into easement by estoppel that runs w/ land to all who have notice of it

o 7) easement implied by necessity elements: doesn’t matter if necessary NOW; must show that originally, it was

1 parcel, parcel was severed into at least 2, and at that moment, it was necessary to have right of egress and ingress

parcel was landlocked and owner had no other way outo 8) easement implied by prior use

9) must go back in time when land owned as one piece: was there quasi easement: was owner giving use to benefit others

10) does it seem reasonably likely that contracting parties would ripen into real easement; use would continue to subsequent purchasers

here, you make intention argumentso necessity is an element here bc more necessary easement, more

likely parties intended easement would run with lando is easement claimed by grantor or grantee

is grantee claims they reserved right? its easier bc grantor knew of the need for the

easement; all the facts were within grantor’s possession; they were not taken by surprise

but when grantor gives servient property and later claims this right of way, its in derogation of grant the grantor gives

grantor was in best position to know the need and it might be surprise to assert this right to the new owner

courts are more hostile with these claims ****final question: always ask what is the scope of the easement?

72

Page 73: Property Final Outline

COVENANTS RUNNING WITH THE LAND these promises concerning land use consist of REAL COVENANTS (enforceable at law to

recover damages) and EQUITABLE SERVITDUES (enforceable in equity by an injunction)

REAL COVENATS: Covenants enforceable at Law A real covenant is a promise about land usage that runs with an estate in land, meaning that

it binds or benefits subsequent owners of the estate A real covenant may be affirmative or negative

o Affirmative: promise to use land in a specified fashion (maintain a box hedge)o Negative: a promise not use land in a specified fashion (no industrial use)

Benefit and burdeno Benefit: a promise about land useage that benefits the lando Burden: a promise about land usage that burdens the land

73

Page 74: Property Final Outline

Runs with the land: property law becomes involved when a covenant about land use is sought to be enforced either by a suceesor to the estate in the land benefitted or burdened by the covenant

Remedy: money damages

What is difference btwn easement and covenants?o Affirmative easements confer rights to use another’s land; real covenants are promises

that land will be used or not used in a specific wayo NEGATIVE EASEMENT: RIGHT to an unobstructed view of your neighbor’s land

REAL COVENANT: PROMISE by your neighbor not to obstruct you view. Covenant is an interest in land.

CREATION of real covenants: can only be created by a written instrument. Cannot be created by implication or prescription

o Necessary elements for burden of a real covenant or its benefit or both to run with the estate

1) intent: original parties must have intended the burden (or benefit, or both) to run with the land to successors

this is satsifed by an explicit statement in covenant that’s binds “successors, heirs, assigns

if silent, court looks to surrounding circumstances to decide whether partie intended to create a running covenant (covenants purpose, conduct of parties)

2) horizontal privity (required for burden to run)**this has been discarded by most courts and RE, but not all: privity of estate btwn original parties

required only for burden to run, not benefit ways for horizontal privity

o 1) ll/tenant relationship (not recognized in US)o 2) conveyance: any connection of interest btwn the original

contracting parties; all you need is that instaneous moment when title passed from grantor to grantee and that’s the conection of interest btwn their estates

3) vertical privity: privity of estate btwn orignal promisor and successor to the burdened estate

exists when successor has aquird the burdened or benefited estate in land held by the original party to the covnenant

1) burden of covenant: necessary to prove the successor aquired the EXACT SAME ESTATE in land owned by the original contracting property

o if something less than the original promisor’s estate is conveyed the burden does not run

o i.e. a owns in fee simple absolute subject to a covenant. Leases property to b. ***covenant does not run bc this is not an identical estate

2) Benefit of Covenant: the benefit of a covenant will run to a successor of SOME INTEREST in the benefitted estate

o i.e. a owns in fee simple with covenant. Leases land to b. covenant does run b acquired some interest in a’s land.

74

Page 75: Property Final Outline

4) touch and concern: substance of promise must touch and concern the burdened or benefitted land

Comonn law: whether the substance of the burden and benefit involves the tangible use and enjoyment of the land

o Effect of covenant: does it depress value of burdened land or increase value of benefitted land

Negative covenants almost always touch and concern the burden and benefitted land

o Courts more generally reluctant ot enforce affirmative covenants RE 3.1 purports to discard touch and conern: holds that servitude is valid

unless it is illegal, uncon or violates public policy o i.e 1) arbitrary, spiteful, capricious 2) unreasonably burden a

fundamental con right 3) impose unreasonable restraint on alienation 4) are unconscionable

Neponsit Property Owners Assoc. v Emigrant Industrial (1938): Touch and ConcernBank purchases land at judicial sale. The deed to the d and every deed in d’s chain of title contains an affirmative covenant to pay an annual charge to the property owner’s association. The association seeks to impose a lien from the covenant.Court holds the affirmative covenant touches and concerns the land, runs with the land and there is privity. Thus the covenant is enforceable and Assoc has standing to enforce the covenant.

Touch and concern: if it affects the legal relations of the parties to the covenantso if the covenantor or covenantee’s interest in their land is substantially altered (less or

more valuable) by the covenant, it touches and concerns the land. ***Negative covenants always touch and concern land

o Affirmative covenants is a harder issue TODAY: Assessment covenants in common interest communities are generally enforceable

(covenants to pay money in other situations usually are found not to touch and concern land), and homeowner associations have standing to enforce them.

EQUITABLE SERVITUDES Covenant about land use that be enforced in equity by an injunction against a successor of

burdened estate who aquired it WITH NOTICE of the covenant Difference btwn real covenant and equitable servitude

o 1) Remedy: damages verusus injunctiono 2) No privity needed: neither horizontal nor vertical privity is needed for either the

benefit or burdeno 3) Can be created by implication in many jurisdictions

Origins of Equitable ServitdueTulk v. Moxhay (1848)Tulk sold to Elms a portion of Leiceister Square with a covenant to maintain it as a garden. The land was then conveyed to Moxhay, whose deed did not contain the covenant but who know of the covenant in the original deed. Moxhay prob said he had notice bc he expected that lack of privity would have protected him bc under the traditional English rule there was no privity btwn Tulk and Elms bc there was no LL/Tenant relationship. Tulk sought to enjoin Moxhay from altering the character of garden.

75

Page 76: Property Final Outline

Court held the covenant was enforceable in equity against a successor even though the promise could not have been enforced as a real covenant (due to lack of privity) bc covenant intended to bind successors, its substance touched and concerned the land and Moxhay had notice of it.

Equity requires: 1) the intent for promise to run 2) subsequent purchaser has actual or constructive notice of the covenant 3) covenant touch and concern the land.

Argue for YES, enforce servitude: Elms paid a cheap price for the land bc it was restricted and when he sells to Moxhay, he sells it for a higher price w/o the restriction.

It would be inequitable to permit Moxhay to violate the restriction bc Elms, the original purchaser, could buy the land at a price reduced due to the restriction and then re-sell for a greater price to a successor who could freely ignore the promise.

Also Moxhay has actual notice of the restriction

CREATION OF EQUITABLE SERVITUDES : 1) In Writing: They are interests in land so the SOF required they that be created in writing

and signed by the promisoro ***Major exception: many states permit negative equitable servitudes to be

created by implication when there is a common scheme of residential development (referre to in Sanborn court as impled reciprocal negative easment. Sanborn

2) Intent for covenant to run to successor owners (see below) 3) notice of the covenant (see below) 4) touch and conern the land (see below)

IMPLICATION FROM A COMMON DEVELOPMENT SCHEMEo Many states will imply a negative equitable servitude where developer sells lots in a

subdivision on the promise that all lots will be burdened with the same use resriction and later fails to carry through on the promise to burden all lots

o REQUIRED ELEMENTS 1) common scheme of development:

development must be of uniform character and recognizable as such by purchasers

Existence of a common scheme determined:o 1) advertisements 2) recorded subdivision map 3) representations

to buyers that it’s a subdivision 4) sale of a number of lots in subdivision with obvious common use restriction

**if developer conveys land in the subdivision w/o use covenants BEFORE THE SUBDIVISION begins, no reciprocal covenants will be implied

2) negative covenant Courts will imply reciprocal covenants ONLY when substance of

covenant is negative-limiting use of property rather than requiring a positive act on part of the owner

3) Enforceability by or against successors requirments must be met for the benefit or burden to run with the land to

successors

76

Page 77: Property Final Outline

a) INTENT: if parties expressly or impliedly intended for covenant to run to benefit or burden successors, the equitable servitude will run

b) NO PRIVATY REQUIRED: no horizontal or vertical privity c) ****NOTICE: subsequent purchaser for value W/O notice of the

servitude at the time is NOT BOUND by the servitudeo 1) actual notice: actual knowledge of the servitude (seller tells

buyer in person or by letter or in sales k or neighbor tells the buyer)

o 2) record notice: if the servitude is ANYWHERE in the chain of title, buyer has constructive notice of servitude

o what if deed from common grantor is not in the chain of title? (Guillete v Daly Dry Wall): required all purchases of

property in subdivision to search records of all grants of other property in subdivision for possibility that servitude was create in a deed outside the buyer’s chain of title

many jurisdictions reject this: on grounds it imposes untolerable burdens on buyers

****does not apply unless there is an express reciprocal negative easement/servitude/restriction

o 3) inquiry notice: purchaser should have inquiry notice bout the existence of servitudes if the neighborhood exhibts a common character. Thus buyer is charged with inquiry notice or the duty to inquire. Sanborn

D. TOUCH AND CONCERN: o In order for the benefit or burden to run to successors in interest,

the substance of the covenant must touch and concern the benefitted or burdened land

o See Neponsit

4) Identifying the Benefitted Land (standing to sue) What if not expressed, court must infer? 1) Land retained by the promisee: if party imposing the covenant owns

land to burdened land, courts presume that the covenant was intended to benefit the retained land

o If promisee later covneys his retained and benefitted land, benefit runs to the assignee

2) Enforceability by 3rd parties: sometimes the benefit of covenant is sought to be enforced by person who is neither a successor the covenant nor successor land benefitted by the covenant

o 2 types of situationso 1) Prior purchaser: intended beneficiary is prior purchaser of a

lot in subdivision developed by common grantor. How to enforce the benefit of the covenant?

1) thru the title chain to the person imposing the covenant (in the subdivision cases, this is easily met)

2) 3rd party beneficiary theory: intended 3rd party beneficiary can enforce the benefit of the covenant

77

Page 78: Property Final Outline

3) ****by implication from a common scheme: Burden by implication: uses subdivision to identify the property benefitted by covenants that are expressly created

o 2) Stranger to chain of title: intended beneficiary is stranger to chain of title

a landowner who is not part of a subdivision and who did not aquire the land from person imposing the servitude, MAY ONLY ENFORCE the benefit IF

1) landowner is an intended beneficiary and 2) juris permits 3rd party beneficiary to enforce the benefit of a covenant

Sanborn v. Mclean (1925)Common owner intended to create a 91 lot residential subdivision but only 53 out of the 91 deeds expressly restricted the lots to residential use. Mclean purchased an unrestricted lot 20 years after the houses were built and wanted to build a gas station. There were no restrictions that appeared in Mclean’s chain of title. Sanborn argued that the subdivision covenant expressly states that the subdivision could be only used for residential purposes only. Sanborns sought an injunction to prevent Mclean from building a gas station as a violation of an implied negative equitable servitude.Court holds this is an implied reciprocal negative servitude. If the owner of two or more lots, so situated as to bear the relation, sells one with restrictions to benefit the land retained, the servitude becomes mutual and during the period of restraint, the owner of the lots or lots retained can do nothing forbidden to the owner of the lot sold. ***Thus, when the developer sells 1 lot with a restriction, all the other lots are restricted by implication.****

Reasoning: the subsequent purchaser, seeing the character of use of the lots in the subdivision and the residential appearance of the neighborhood, was charged with inquiry notice and should have known that there was an implied negative easement on all the lots

Scope of Sanborn: when to use this caseo 1) can be used to imply a restriction a lot that doesn’t have one in writingo 2) can be used to give standing to a party in a subdivision to enforce a restriction when

they aren’t the benefitted parcel in writing How could this problem have been avoided?

o 1) Developer could have expressly restricted every deedo 2) Developer could have use a strawman

As developer buys subdivision and record. As a prospective purchaser, if I want to make sure every lot will have restriction,

I take all the deeds in subdivision and transfer title to a strawman. Strawman creates a deed for every lot except for one and every deed contains a restriction (you hang on to one deed bc there must be a burdened and benefitted parcel). The one lot is the benefitted lot and now there is 99 restricted lots.

Finally, sell the 1 lot benefitted with a restriction to the purchaser Argue against implied reciprocal negative easement

o This is bad case law with serious notice issues, serious ramifications if the restrictions are not in writing and serious burdens

Burden of checking every deed outside of my chain of title looking for restrictions

o Argue brown v voss: sitting around, waiting to sue, thus estopped from suing (why didn’t you stop me earlier, why did you wait to sue)

Argue for the doctrine

78

Page 79: Property Final Outline

o All you must do is look around to realize there must be a restriction. You can see its all lots are residential houses and these people obviously, in the middle of the subdivision, do not want a gas station bc it will devalue their property.

o Distinguish san born: san born says when you go to buy property, look around and what do you see (pleasentiville—al homes—thus you can’t build gas station)

What if the court doesn’t follow Sanborn?o Argue 3rd aprty beneficiary theory. See below on question 6.

NY: REJECTS ANY RESTRICTIONS THAT ARE NOT IN WRITING

WHO CAN ENFORCE COVENANTS: not everyone in the world can enforce a covenant against you. Must have standing to enforce a covenant (3rd party beneficiary)

Sanborn can be used for 2 things to enforce restrictiono 1) to imply restriction on a lot that doesn’t have one in writing o 2) give party standing in a subdivision to enforce a restriction when they aren’t

benefitted parcel in writing Enforcing a Restriction: Rules of Construction

o 1) Presumption: if language of the covenant is unclear or ambiguous, court will resolve the covenant in favor of free enjoyment of the land and against restrictions.

o 2) Court will not read restrictions on the use and enjoyment of the land into the covenant by implication

o 3) court must interpret reasonably, but strictly, so as not to create an illogical, unnatural or strained construction

o 4) court must give words in the covenant their ordinary and intended meaning

when developer sells lot with an a express reciprocal restriction 1. the owner of Murray’s Bagels (located one block away) enforce the restriction?

NO. Murray does not having standing bc he doesn’t have the benefit of the restriction. Must show you have the benefit Person seeking enforcement of the restriction must establish he has legal standing to

enforce it. ***so what can murray do: 1) make a deal with people who have standing to enforce

restriction bc it would devalue all property 2) try under 3rd party beneficiary theory 2. Can the owner of lot 5 restrict lot 1’s building.

Argues for guy who wants to restrict: o YES. Lot 5 has right to enforce the restriction bc when O sold lot 1 with the express

reciprocal restriction, the restriction was to benefit o’s retained land. Lot 5 was was one of the lots retained by o, thus lot 5 is a benefitted parcel.

o Further, the benefit runs with the retained land to subsequent purchasers, thus owner of lot 5 has standing to enforce the restriction.

Argues for guy who says NO restriction: I am a subsequent purchaser w/o notice. o counters: the restriction was in lot 5’s deed when o restricted lot. Thus no lack of

notice here.3. Suppose the owner of lot 5 seeks to build a starbucks. Can 1 obtain an injunction?

YES. When o expressly imposed the same restrictions on his retained land when he sold to lot 1, lot 5 became restricted at that time as well.

79

Page 80: Property Final Outline

o Lot 1 must argue the burden created in lot 5 rain with the land to subsequent purchasers AND argue that lot 5 had notice of the restriction

1 argues for notice: the restriction is in 5’s chain of title and its in lot 5’s deed.

*****Suppose lot 5's deed did not have a restriction in it. So lot 1-4 are sold with reciprocal restrictions and lots 5 and 6 have no restrictions?

Must ask 3 questions?o 1) is there a restriction on lot 5?o 2) did the owner of lot 5 purchase the lot with notice of the restrictiono 3) are we in a Guillete juris? If No, 5 Wins.

argues for implying a restiction: DryWall. Lot 5 has no restriction, but o restricted his retained lots in the deed to lot 1 so the restriction is placed outside of 5’s chain of title. He is charged with inquiry notice.

Argues for no implied restriction: there is no restriction bc when o sold lots 1-4, there was restrictions in writing

o Even if there was a restriction, I had no notice bc it was not in my deed or chain of title. this is not daly dry wall.

Distinguish Daly Dry Wall: in Dry wall, there was 200 house subdivision. Here, only 6 houses on a block. All I see is 6 parcels that used to be owed by 1 person. This doesn’t tell me I should do a more extensive title search

o Court should not extend Dry Wall here: this rule only makes sense if it’s a clear subdivision with a plan, map and its recorded and there is a lot of similar houses thus requiring a subsequent purchaser to search outside of his chain of title

If Dry Wall is extended here, anyone everywhere will have to look at all the deeds outside of their chain of title of previous owners to see if there is a restriction.

o Bad policy: makes titled searches time consuming and expensive. Daly should be confined when there is a clear subdivision wth a map and a lot of houses.

developer expressly restricted development of each lot sold, but did not include a reciprocal restriction on his remaining property. does owner of lot 5, not restricted, having standing to sue owner of lot 1, restricted?

When lot 1 was restricted, there was a burden on 1 and a benefit that attaches to lots 2-6. When 5 purchases from 0, the benefit runs and 5 has standing.

developer expressly restricted development of each lot sold, but did not include a reciprocal restriction on his remaining property. does owner of lot 1, restricted, having standing to sue owner of lot 5, not restricted?

Problem for lot 1 bc no standing: o lot 1 is restricted but 2-6 are not restricted, they are benefitted and not burdened. When

o sells lot 5 with a restriction, it only benefits o’s retained land which is only lot 6. Thus only 6 has standing to enforce.

Lot 1 argues for standing under Sanborn: o when o sold lot 1 with a restriction, Sanborn holds that there can be an implied

reciprocal negative servitude. o Thus when Lot 1 sold with a restriction, lot 5 is restricted by implication and the benefit

80

Page 81: Property Final Outline

runs lot 1. If court doesn’t follow Sanborn, 1 argues 3rd party beneficiary theory:

o when o sold lots 2/3/4/5, he restricts each lot, he is doing this not only to benefit the land he retains, but also to benefit all the other lots in the subdivision he has already sold. We are all intended beneficiaries of the restriction.

o 1 must show there was a restriction and it was intended to benefit 1: must be clear intent to create a subdivision and developer intended to benefit

other parties in subdivision. 5 argues that the burden did not run

o 1) there is no restriction here. Even if you apply San Born, I had no notice of the restriction bc when I looked around, all I saw was 6 parcels in a semi urban area

o This doesn’t tell me I am in a subdivision. No NOTICE. In Sanborn, it was 100 lots and it was absolutely clear that it was a subdivision

for residential use.

INTERPRETATION/SCOPE OF COVENANTSGENERAL RULE: courts try to implement the intentions of the parties in creating the covenant but the terms used in a covenant are often ambiguous

1. Building restriction or use restriction: covenants may restrict uses or they may restrict construction of certain types of buildings.

**may be an important difference 2. Residential purposes: many covenants limit use to residential purposes. The content of this

term is not self evident bc people commonly use their residences for various purposes****if a covenant restricts residential property, building a condo is not restricted bc its residential property (building condo is restricted with single family home restriction******

o 1) Combined business and residential use: some courts take the view that ANY commercial enterprise no matter how domestic, is not residential

o 2) Group Homes: most courts conclude that group homes for handicaps, juveniles delinquients, or other groups do not constitue violations of residential only covenants bc people do in fact live there

Hill v. Community of Damien Molokai (1996) o 3) what constitutes family? Many residential covenants are phrased to limit use to

single family residential use or single family residence see Hill holding group of 4 unrelated AIDS patients living together under the

supervision of the Community of Molokaie was a family for purposes of the covenant

3.Racial Restrictions: Racial bias manifested by covenants restricting use and occupany of property to a paricular group of people (like whites people) are NOT ENFORCEABLE bc they are unconstitutional.

See Hill holding covenant violates the FHA and Shelly v. Kramer

Hill v. Community of Damien Molokai (1996)Molokai leased a single family residence in a subdivision for use as a group home for 4 people with AIDS. All were unrelated. Restrictive covenant said no lot in the subdivision can be used for no purpose other than single family residence. Residents of the subdivision sued to enforce the restriction.Court held use of the lot as a group home for AIDS patients did not violate the covenant. Even if it did violate the covenant, forcing them out would be illegal under the Fair Housing Act.

Enforcing a Restriction: Rules of Construction

81

Page 82: Property Final Outline

o 1) Presumption: if language of the covenant is unclear or ambiguous, court will resolve the covenant in favor of free enjoyment of the land and against restrictions.

o 2) Court will not read restrictions on the use and enjoyment of the land into the covenant by implication

o 3) court must interpret reasonably, but strictly, so as not to create an illogical, unnatural or strained construction

o 4) court must give words in the covenant their ordinary and intended meaning 1) Molokai did not violate covenant

o Group home had a traditional family structureo ordinance definition of family: any 5 or less unrelated people lving in the same dwellingo Strong public policy favoring removal of barriers preventing people with disabilities

from living in group homes in residential settings (Urged by fed and state govt) 2) Even if they do violate covenant, the Covenant violates Fair Housing Act

o 1. NO discriminatory intent: whether d has treated handicap people differently from other similarity situated people

evidence by community is lacking here to support this claimo 2. YES disparate treatment: p need to prove that d’s conduct actually and predictably

results in discrimination or has discriminatory effect 1) covenant, which attempts to limit group homes, has discriminatory effect of

denying housing to handicapped bc they usually live in group homes p’s interests in avoiding traffic DOES NOT outweigh

2) strong public policy favoring disabled people living in residential settingso 3. YES reasonable accommodations: p failed to make reasonable accommodations

under 3604 (f)(3)(b): discriminations includes refusal to make accommodation in rules, etc as may be necessary to handicap

court says if neighbors denied access or refusal to accommodate to neighborhood through its covenant, it should be forced to accommodate

Racial Restrictions: covenants restricting use and occupancy to property to white people are not enforceable: their enforcement is not con permittedShelley v KramerFitzy conveyed property to Shelley who was black. Kramer sought to enjoin Shelley from occupying their new home bc prop burdened by covenant resrtricting use to whites.US Sup Court overturned Missouri court’s enjoining Shelley from occuping property and held state’s refusal to permit a willing buyer and seller to transfer prop bc of race is violation of equal protection clause.

TERMINATION OF REAL COVENANTS AND EQUITABLE SERVITUDES 1) Merger 2) Eminent Domain 3) Express Waiver or Release: if all the holders of the benefit of a covenant expressly release

the covenant, its extinguished 4) Expiration of the Covenant: Covenants sometimes have a defined life span and when its

defined life expires, covenant is extinguished. 5) DOCTRINES TERMINATING EQUITABLE SERVITUDES:

o a. Changed Conditions Within the Affected area (subdivision): a covenant will no logner be enforced in equity if conditions have so radically and thoroughly

82

Page 83: Property Final Outline

changed withint the area ffected by a covenant that the covenant can no longer achieve its purpose (usually occurs in a subdivision)

o b. Changed Conditions in the Surrounding Area: nature and character of the surrounding area has so changed that it would no be inequitable to enforce the servitude

NECESSARY TO ESTABLISH that the extrinsic changes in the neigborhood have been so pervasive THAT ALL OF THE BENEFITTED LOTS have lost the benefit of covenant

put another way, there is no point to enforcing the covenant any longer bc its purpose can no longer be achieved. burden is on people who want restrictions released

see Western Land v Truskolaski o c. Abandonment: servitude can be abandoned by widespread violation of the

covenant w/o enoforcement. If violations are sufficiently numerous and there has been no enforcement action taken, covnenat may be seen as abandoned

o d. Balance of Hardships: even when injunctive relief is appropriate, a court may deny an injunction if the hardship imposed by the injunction is very large in relation to the benefits produced.

See Rick v West

Western Land Co. v. Truskolaski (1972)Developer subdivided his development and imposed restrictive covenants that the homes be used only for residential purposes. Developer now wants to build a shopping center on a parcel he retained claiming the purpose of the restriction has been thwarted by increase in city population, value of the land (circumstances have changed) and homeowners continued disregard for the covenant (abondonment). Homeowner sought to enforce covenant against the developer.Court grants injunction and holds that its not that there isn’t a disparity btwn residential and commercial use, but homeowners still deriving a substantial benefit and value from the covenant.

Rule: Covenants are still enforceable if the single-family resdiental character of the neighborhood has not been adversely affected and purpose of the restriction not thwarted.

Argue for injunction: Here, although city is changing, within the subdivision, its not changing. Subdivision is still residential and thus the homeowners are still deriving value from the covenant

o Bad precedant: to allow developer to remove covenants at their will, it will set a bad precedant and people wont but on reliance of the covenant and value of the covenant will be eroded

o Damages are inadequate here: if court awards damages instead of an injunction, court will award the difference of the mv of land w/ and w/o covenant, this is problematic bc it doesn’t factor in subjective attachment of homeowner

Injunction allows homeowners to realize the value of the covenant

Argue for no injunction: it is unreasonable to hold me to these restrictions bc the circumstances of the city has changed and this is a restraint on alienation. The best use of the land is for commercial purposes

o Awarding injunction is problematic bc strategic/actor holdout problem

Argue for who homeowner wants to restriction lifted so they can sell: Here it’s no longer quiet for anyone; the market value has dropped for single family homes.

83

Page 84: Property Final Outline

o In Western Land the existence of commercial development didn’t devalue the land. Everyone’s being hurt by this unforeseen development argument. The covenant there was to keep the value; here’s it’s the opposite – causing devaluation.

o No forseeability as to what development would happen down the line***couldn’t predict this and its not fair to hold people to something they had no idea about

o Problem of the holdout: this is not attaching subjective valueo Restriction on alienation without any benefit

Argue for holdouts who want to keep restriction and don’t want to sello Rick v west: reliance idea: the very reason she bought was for the home in a

neighborhoodo This is a prop right and no one could be forced to give up their prop righto If homeowners didn’t want to be bound by this restriction, why did they buy in

Abandonment: people are violating home restriction and developer wants it removed Landowners:

o This is a specific kind of commercial use – unlikely to change the makeup of the neighborhood.

o They’ve been operating this business; nothing has changed. It’s one thing to have small commercial use; another thing to lift it and have everyone do what they want. If you lift the restrictions for everyone going forward there will be a radical change.

o Its not fair to infer strategic behavior/power of holdout. There violations were minor. Developer: allowing these pople to slide is that they don’t find real value in the covenant

o to the extent that they’re showing that the covenant isn’t that valuable…further evidence that the covenant isn’t doing what we want.

o While there’s debate about what the covenant’s generating; here they had a right to enforce it and they didn’t. The covenant should be waived.

Waiver argument would be strong if we had one homeowner who broke the covenant; you didn’t move against that person; unfair to selectively enforce against me.

o Leslie argument: if the other homeowners failed to enforce it on other occasions, that this is additional evidence that they’re behaving strategically rather than out of a sincere desire to enforce the covenant.

Here they’re just exploiting their roles as holdouts. Just trying to profit from being paid off to release the covenant and then pay at the higher rate now that a payoff is more likely.

Rick v West (1962)Developer sold land with a single family home restriction. Now, the developer wants to sell his land to a hospital. One homeowner refuses to sell her land. Developer sued claiming the covenant was no longer enforceable due to changed conditions.Court holds there was no evidence of any substantial change in the general neighborhood and no change at all within the developer’s tract, so the homeowner should be allowed to refuse.

Even when injunctive relief is appropriate, court may deny an injunction if hardship imposed by injunction is large in relation to benefits.

o **Balance of equities are in favor of landowner

84

Page 85: Property Final Outline

When developer subdivided, he included restrictions and landowner relied on those covenants and has a right to continue to rely on them. Landowner deserves to be protected by the covenant.

Difference btwn here and Western Land: there is no holdout problem

COMMON INTEREST COMMUNITIES 1) Covenants Recorded in a Master Deed: any purchaser of a condo has constructive notice

of such covenantso courts are divided in the degree of DEFERENCE GIVEN TO THESE COVENANTS:

1) some states hold these restrictions are “clothed with a very strong presumption of validity” and invalidated only when they are arbitrary, violate public policy or interfere with exercise of fundamental con right

2) other states these covenants should be enforceable unless UNREASONABLE see Nahrstedt

2) Covenants Imposed by Homeowners Assoc: some states that clothe original recorded covenants with strong presumption of validty are not so deferential to covenants imposed by homeowners assoc AFTER THE OWNERS HAVE AQUIRED TITLE

o there is no presumption of validity and less deference to board and only valid if not unreasonable

o Counter argument: I bought my condo on reliance that I could have cats 3) Coooperative Apts: each owner-lessee is obligated for a portion of entire cost of owning the

building and 1 owner’s failure imposes burdens on other, financial fate of co-ops is more independent that that of a condo owner

o as result court are willing to defer to business judgment rule: board permitted to deny ownership to anyone for any reason except violation of civil rights law

o Business Judgement Rule: lowest level of review, great deference Court will not disturb a judgment made in good faith by the board

Nahrstedt v. Lakeside Villago Condo Assoc (1994)Nahrstedt bought a condo unit that was subject to a negative covenant in the master deed against pets. She had three cats. Assoc found out about the cats and asked her to remove the cats and assessed fines each month the cats were there. She sued the assoc on grounds the restriction was unreasonable since her cats remainded indoors, didn’t bother anyone and that she didn’t know about the restriction when she bought the condo.Court gives presumption of validity if the covenant is in the master deed unless the covenant is unreasonable. The Restriction was not unreasonable bc it wasn’t arbitrary, against public policy or burden of the covenans did not outweigh the benefits.

The homeowner had constructive notice of he restriction bc it was in the master deed. Efficency argument: if court allows as applied challenges, a lot of litigation will ensue in all

co-op board and the homeowners will share the cost of the litigation Court gives deference to restrictions: to protect general expectations of condo owners that

the restrictions in place when they buy the condo will be enforceableo Protects buyers who paid premium for the condo and who have relied on the restriction

Presumption of validty in master deed: rebutted ifo 1) covenant violates public policyo 2) covenant is arbitrary and bears no rational relatoionship to the protection,

preservation, operation or purpose of the affected lando 3) burden of covenant outweights the benefits of it

85

Page 86: Property Final Outline

JUDICIAL LAND USE CONTROLS; THE LAW OF NUISSANCEGeneral principle: one must use one’s property so as not to injure another’s property

A person may not use his own land in an unreasonable manner that substantially lessen’s another person’ use and enjoyment of his land

Private nuisance: substantial interference with private rights to use and enjoy land either: Intentional and unreasonable conduct OR Unintentional conduct that is either negligent, reckless or so inherently dangerous that strict

liability is imposed

86

Page 87: Property Final Outline

A. Intentional and Unreasonable Conduct: most common form of nuisance. Focus here is whether conduct is an unreasonable interference with another’s land. 3 views:

a. Balancing: Harm and social utlityi. If the gravity of the harm inflicted by the conduct outweighs its social

utility, the conduct is unreasonable. ii. To measure the gravity of harm, consider the extent of the harm, its character,

social value of its use, suitability of use to location, and burden of avoiding harmiii. To measure the social value of the conduct, consider its suitabliltiy to location

and practicle difficulty of preventing the harmb. Balancing: uncompensated harm and ruinous liability

i. The intentional activity is unreasonable if it causes serious harm and the actor could compensate for that and similar harm w/o going out of business

c. Substantial Harmi. Some courts ignore the balancing test if substantial harm is inflicted

ii. Here the nuisance exists if the injury it inflicts is severe enough to be above the max level of interference that person can be expected to endure w/o redress.

Jost v. Dairyland Power Coop: ***applies the substantial harm test****Dairyland’s electric power plant spewed sulfur dioxide into air causing minor property damage to jos and his farm. Court held it was a nuisance and stated “where invasion involves physical damage to property, gravity of harm is regarded as great even though extent of harm is small. To permit a socially useful pulbic utility to deprive others of full use of their property w/o compensation…would constitute taking of property w/o due process of law.

Morgan v High Penn Oil Co (1953)****seems to apply substantial harm testHigh Penn operated oil refinery that emitted noxious odoars several times each week, polluting the air for about 2 mile radius. Along with many other people who owned land located w/in radius, Morgan sued to enjoin refinery’s operations alleging noxious odors made him sick and deprived him of use and enjoyment of his property.Court applies rule that use is nuisance if is both intentional and unreasonable or unintentionally produced by negligence, recklessness or extremely dangerous activity. Court held High Penn intended to operate the refinery and knew or should have known that its operation would produce the noxious odors.

*****Their use was intentional and unreasonably caused noxious gases and odors to escape into p’s land to such as degree so as to impair in a substantial manner the p’s use and enjoyment of the land*******

Substantial interference: alleged nuisance, whether intentional or not, must be a substantial impediment to use and enjoyment of land. The average person is the standard measurement for substantial interference. See Morgan v High Penn

Who is Cheapest Cost Avoider: Penn knew what their operations do, they came into neighborhood 2nd, could have seen this neighborhood was residential, could have setup elsewhere or bought out neighbors, but didn’t and they should bear the cost

d. They could have foreseen this conflict and avoided it

Unintentional Conduct: When actor uses his land is way that unintentionally injures another’s use or enjoyment of land, the action is a nuisance if either the conduct is below the standard of care commonly required (reckless or negligent) OR risk of harm is great that conduct ought not be tolerated

87

Page 88: Property Final Outline

(i.e. inherently dangerous). Here the focus is on the actor’s conduct—does it pose an unreasonable risk of harm either bc it is careless or inherently dangerous.

Public nuisance: affects rights held in common by everybody—the public—rather than just private rights of land use held by landowners

Enforcement: normally abated by suits brought by public officials but a private citizen can bring suit to abate a public nuisance if he has been specially injured by the nuisance

o This means the private p has suffered some particular and personal injury

COASE THEOREM: in a perfect world free of transaction costs, it doesn’t matter which of Adam and Eve are entitled to continue their use or which is the most efficient use.

o When negociation is possible and there are no transactions costs, rational actors will bargain to the most efficient outcome bc the use rights will end up in the hands of person whose use is the most valuable.

Under Coase, corts should not worry about most efficient use bc prties will work it out bc whoever values their land more will buy other party out. No matter what, the most efficient use is perpetuated.

ONLY ISSUE for judge is who should absorb the cost. Coase assumptions- when Coase breaks down

o 1) Does not take into account subjective attachmento 2) Does not take into account people acting in irrational self interest due to

hatred for other party (i.e. unless it companies bc there is no emotion involed: all cost benefit analysis)

o 3) Assumes the most efficient use will have funds to buy other party out (i.e. party who has high subjective value will have enough to buy other guy out)

o 4) transaction costs. The cost of moving the right from Adam to Eve or Eve to Adam is not zero, its not even insignificant. WHY? 3 reasons.

1) Bilateral monopoly: where there are only 2 people involved in the transaction, there is an inherent bilateral monopoly problem. There is only one seller and 1 buyerdueling monopolies

They will haggle; they will bluster; they will hire lawyers to threaten more litigation and thus spend gains before acquiring them. In short, they will play negotiation games with each other, expending money and time as they do, thus making it harder to reach a deal and diminishing its value even if reached.

2) Free Riders: when there are numerous parties to the negotiation, different problems emerge. One of them is caused by human impulse to get a free ride at someone elses expense.

88

Page 89: Property Final Outline

The rational response of landowners is to contribute and purchase the right, but some landowners will not contribute because they hope to receive the benefit of cessation of use w/out paying for it. The knowledge that this may happen will inhibit other landowners from making contributions because they dislike giving free ride to someone else. Thr right may not be purchased, even though it is clearly economically efficient to do so.

3) Holdouts: The last holdout will realize that Eve will rationally pay as much as as

their max value to obtain the holdout’s right. This simple fact is likely to spur holdouts. Of course, Eve can make her purchases conditional upon obtaining all rights but that condition does not eliminate the incentive to hold out. Because Eve cannot compel everyone to sell on reasonable terms, she may never be able to complete the transaction and the right will stay with Adam and his cohorts, the inefficient outcome.

Who Should Bear the Cost of Nuissance (who gets initial entitlement)? Answer: who is the cheapest cost avoider. He who could have avoided

conflict should bear cost. He who could have forseen the conflict and avoided it should bear the cost.

o Who gets the initial entitlement? 1) More valuable use: this is the outcome that (but for transaction costs)

would ultimately result. 2) First user: some would give initial entitlement to 1st user on theory the later

users should adapt themselves to existing conditions. Those who knowingly acquire and use land in a manner incompatible with exising uses have voluntarily assumed burden of what might be nuisance were it to have been the later arrival.

3) Disfavored uses: some identify uses that, however efficient or utile, are dangerous to public health and give initial entitlement to competing use

o CHEAPEST COST AVOIDER, who could have avoided the conflict: 1st in time

The person who comes 2nd could have scoped the area, looked around, not build or build carefully.

While 1st in time must either negociate for a servitude, must find all neighbors, risk holdout or freerider.

Here, the cheapest cost avoider is 2nd in time party. They could have avoided this conflict and should bear the cost.

Not always 1st in time is cheapest cost avoider (idiosyncratic preference should bear the cost sometimes)

i.e. homeowner is first. bakery opens up. Homeowner has hypersensitive reaction to smell of wheat. Wants to shut down bakery bc I was here first

distinction btwn cheapest cost avoider and 1st in timeo not clear that baker is best person to prevent harm. Baker could

not have reasonably conceived smell of wheat would be a nuisance

89

Page 90: Property Final Outline

here, person with idiosuncratic preference should bear cost

Four possible remedieso Remedy 1) No nuisance: use continues w/o restraint.

If a challenged activity is found not to be a nuisance, use right is allocated to the nuisance and is protected by a property rule. The nuisance man canot be forced to stop the use w/o his consent.

Use will continue unless the nuisance man is the less valuable one and transactions costs do not inhibit the transfer.

o Remedy 2) Enjoin the nuisance to stop it: if activity is found to be a nuisance, challenged activity will be enjoined and it will stop and use right will be transferred to challenger

challenger can continue use at his pleasure. If the enjoined activity is more valuable, the use right will likely be shifted to the enjoined user unless transaction cost prevent the transfer

Estancias Dallas Corp v Schulz:Estancias constructed an apt complex adjacent to Schutlz residence. To save 40k, Estancas located it central AC unit about 5 feet from Schult’s house. The AC was so noisy that it prevented Schultz from entertaining outdoors and interfered with indoor convo and sleep. To change the location of unit would cost Estancias 150k to 200k. the apts could not be rented w/o AC in Dallas, TX. Value of Schultz house was 25k but house loss half its value with the nuisance.Court held AC was nuisance and affirmed trial courts judgment of past damages and enjoined it from further operation.

Why a nuisance? Court applied substantial harm test: o Gravity of harm (loss of half the market value of the hosue down to 10-12,500k) was

outweighed by the social utility of AC (cost of avoiding harm: 40k at outset, now 150k to 200k after injunction)

Although economic theory says this result should have resulted in shift of use right from Schulz to Estancias upon payment of some amount of 25k to 150k, this did NOT happen

o either Schultz was irrational or transactions costs consumed surplus or Schultz’s subjective value was higher than 200k

Apply Coaseo Estancias will lose 150k to 200k if forced to mitigate while Schultz would have lost 15k

in diminution of market valueo No matter what, Estancias will buy out owner bc they have incentive up to 200k to get

injunction lifted Who is cheapest cost avoider?

o Estacias. 2nd in time, could have avoided/mitigated this. Conflict was foreseeable bc it’s a huge AC unit

Could have avoided problem by installing individual AC units for 40k less. ARGUE AGAINST INJUNCTION

o Estancias argues that court did not err in finding nuisance, court erred in issue an injunction and not damages

By awarding injunction, you allow other party to extort money bc they have all the bargaining power

90

Page 91: Property Final Outline

o With injunction, there is holdout problem, freerider issues, and cant truly tell if homeowner attaches subjective value or just extracting profits (strategic actor)

Argue for injunctiono Without an injunction, it will materially affect the use of my propertyo Conflict was foreseeable, you could have avoided this bc you were cheapest cost

avoider.o Incentive structure: Damage award would set wrong precedant. This would encourage

developers and builders to factor in nuissance damages as cost benefit analysis. Developer says I might have to pay damages and I will factor this into my cost benefit.

Wth injunction, incentives develpers to minimize nuisance.o With injunction, I can factor in my subjective value of land as opposed to damages, it

doesn’t factor in my subjective value, factors diminution in mv

Remedy 3): Award Damages to landowners affected by nuisance, but permit use to continues

a. large numbers of landowners affected by a more valuable use that is, on balance, a nuisance, the presence of holdout transaction costs may prompt a court to protect the use right of numerous landowners by a liability rule instead of property rule.

b. Coourt awards damages to the affected landowners instead of enjoining nuisance. Damages are PERMANENT DAMAGES

i. Permanent damages is an amount sufficient to compensate now for all past and future injury that may be inflicted by continuation of nuisance

Boomer v. Atlantic Cement Co: Atlantic factory produced dirt, smoke, nouse and vibration that substantially interfered w/ use an denjoyment of land owned by a large group of neighbors.Court upheld trial courts finding that factory was a nuisance and an award of damages instead of an injunction.

Case was remanded for determination of the amount of PERMANENT DAMAGES to be awarded for “servitude” thus created over the affected land.

Rational for Damages over injunction: o 1) technological impossibility of abatemento 2) recognition that the factory was the more valuable use (provided jobs and economic

benefits to region) o 3) holdout possibility: might frustrate market transfer of right if factory was enjoined o 4) Court applies balancing formula and asks whether factory could compensate for all

serous harm it causes w/o ceasing business and concluded YES, it could do so. Why no injunction: Courts assumptiom:

o assumes factory will shut down if theres an injunction Thus, Court overrules long established rule of granting an injunction where a

nuisance results in substantial injury and grants damageso Even if factory doesn’t close, court is nevous about regulating a public health prob from

the bench which could have massive external effects Legislature is best suited to do this

91

Page 92: Property Final Outline

Argue for injunction 1) allows owner to realize subjective value of land versus just the diminution in mv 2) we don’t force property owners to sell their property interest 2) incentive structure

o strong incentive to minimize nuisances by locating in area where use will be no problem or take all necessary steps to reduce the nuisance

if I know I will be enjoined, company realizes it will be very expensive to buy the injunction and will factor this is

if there is damages awards in jurisdiction, when I do cost benefit analysis, if I am sued, I will only pay for diminution in mv and that’s all

3) past precedant: in Ny, if nuisance, then inunction

Argue against injunction 2 main arguments: 1) fear parties couldn’t bargain to efficient result 2) impact on larger societal

problem damages are better bc we employ lots of people and they lose their job with injunction

o counter: no they wont, you just will have to pay the homeowners for their subjective value bc disparity is huge (45mm verus 185k), thus those people will not lose their jobs***PARTIES WILL BARGAIN TO EFFICIENT SOLUTION AND NO JOBS WILL BE LOST

FACTUAL DISTINCTION BTWN ESTANCIAS is HOLDOUT PROBLEM. in perfect bargaining world, they are better off with 50k in damages; but they look at deep pockets of company and take advantage.)***all it takes is 1 of these people.

to the extent that injunction forces people to adopt safest measures, in this case there is no technology yet developed

o counter: I still have a property right. Why should I subsidize the fact that cement is a polluter and why should court be in the position to decide the cement is reckless or negligent. Stick with injunction and MAKE SURE THE CEMENT IS OPERATING IN SAFEST WAY POSSIBLE.

worried about precedent this will set. If injunction granted, cement will have to pay millions in damages.****THERE IS NO BETTER WAY TO OPERATE A CEMENT COMPANY IN ANY LESS POLLUTING WAY. IF THEY GRANT INJUNCTION, EVERY HOMEOWNER IN VICINITY OF CEMENT PLANT WILL HAVE ABILITY TO SHUTDOWN THE CEMENT AND EVEN IF NOT SHUT DOWN, THOSE CRIPPLING DAMAGES COULD HAVE BAD EFFECT ON THE INDUSTRY (LEGISLATURE IS BETTER ABLE TO DEAL WITH THIS)

o THIS HOLDING COULD HAVE MASSIVE IMPACT ON THE INDUSTRY

Remedy 4) Enjoin the use and award damages to the enjoined userc. courts may enjoin an activity but require that the benefitted landowner compensate the

enjoined actor for lost use

Spur Industries v. Del E. Webb DevelopmentSpur operated a cattle feed lot. Feed lot generated enourmas quantites of manure, attracted clouds of insects and created noxious odors. Del began building a housing complex and at the time, did not consider the odors and problems of Spur so they continued to develop until Sales resistence got to bad

92

Page 93: Property Final Outline

that parcels were impossible to sell. Del argues Spur’s operation was a public nuisance bc of flies and odor drifted to housing complex and created an unhealthy and annoying situation.Court enjoined Spur from further operation of the feedlot but required Del to pay Spur a reasonable amount of the cost of moving or shutting down

Equity required Del to compensate Spur bc Del came to the nuisance and Dell must pay as the cheapest cost avoider.

This allowso 1) most efficient use to prevailo 2) puts cost on cheapes cost avoider who did nothing to prevent this

Who is cheapest cost avoider? Dell: they came 2nd. They saw land was cheap bc it wa next to Cattle lot. They shoul have

foreeen this conflict and avoided itIf court grants an injunction?

Unsatisfatory result bc it fundamentally unfair. Spur did nothing wrong, could not foresee this problem bc they were first in time

If court grants damages Nope, bc this decision doesn’t factor in broader problem.

o feed lot constitutes a public nuisance and presents heath issueso also, spur would only compensate dell. What about all the other owners of properties

within dell. They get nothing

*when city is bringing claim, it will be zoning, eminent domain or public nuisance (always keep in mind spot zoning)

93

Page 94: Property Final Outline

LEGISLATIVE LAND USE CONTROLS; LAW OF ZONING****when making arguments regarding zoning, use takings cases: hadychuck for something harmful like pornship, no economic viable use (lucas), physical occupation*****

Zoning is use of public power to impose uniform results that might otherwise be accomplished in more piecemeal and selective fashion by private bargains (servitudes) and nuisance law

zoning objectiveso 1) to prevent incompatible uses from occurring (thus reducing need for nuisance)o 2) increase prop values by minimizing use conflicts and aesthically more pleaseo 3) channel development into patterns that may serve largers social goals

Why we need Zoning and Govt Intervention (minimizes litigtion costs and puts people on notice ina dvance of what they are restricted from doing)

o 1) Servitudes are limited solutions peoples preferences might not align and its unlikely to get uniform scheme

high transaction costs to get everyone to agree people cant get adequate representation to enforce their rights can restrict you neighborhood but what about the houses on next block

o 2) Nuissance nuisance arrives after the fact, thus its expensive sometimes only get damages and not injunction and use continues high litigation costs

A. Constitutional Validity

Village of Euclid v Amber Realty Co (1926)Amber owned 68 acres in the village of Euclid, right outside of Cleveland, when Euclid adopted a comprehensive zoning ordinance that restricted the permissible uses of property, limited height of structures, and imposed mimimum lot size requirements for certain structures. Euclid wanted to use their land for indstrial purposes but a reasonable amount of the land had been zoned for non industrial use. The zoning reducded Amber’s land by 75%. Amber claimed the zoning ordinance deprived him of his property w/o due process of law.The Supreme Court upheld the validity of the law as constitutional against due process bc the law’s objective—minimizing land use conflicts to prevent nuisances from ever occurring—was a legitimate exercise of state’s inherent police power bc its content was neither UNREASONABLE NOR ARBITRARY.

Court takes a deferential standard o As long as there is some rational basis for muni actor to believe this ordiance promotes

the general welfare, health and safety of its citizens, courts defer Danger of Deferential Standard to Muni

o Danger in allowing legislature to enact ordiance bc it may suppresses minority interests and they cant vote the legislature out as the minority (lacks resources and the vote)

o legislature has no forseeabiltiy as to future conflict and will often be over or under inclusive and it will work unfairness on a particicular group of people

B. Authorization for Zoning

94

Page 95: Property Final Outline

in general, a local zoning law is void unless it is in conformity to the State’s enabling act the law authorizing localities to engage in zoning

o Standard State Zoning Enabling Act (in effect in every state): 1) requires zoning decisions to be made in accordance with a comprehensive

plan which is intended to be a general guide for overall development of locality 2) requires establishment of a zoning commission/board and appeal

mechanism for affected landowners ***also need Board of Adjustment independent form the commission to

settle ordiance disputes

C. Statutory Discretion and Restraint and FLEXIBILITY IN ZONING Zoning responds to this by 1) tolerating the continued exisence of land uses existing prior to

adopting of zoning law 2) providing for amendment of zoning law 3) conferring discretion of administrators in application of the zoning law

1. Non-Conforming Useo non-conforming uses are permitted to continue to exist bc their immediate abatement

would amount either to a taking of property w/o just compenation or an unreasonable exercise fo zoning power

o A. Forced Phased Out (amortization period): the zoning law may specify a period after which non-conforming use must cease

1. Majority Ammortization Rule: Valid if reasonable period Forced phased out are valid as long as amortization period is reasonable

as to the affected nonconforming user and user has time to recoup his investment

How to determine reasonable amortization: Balaning Testo Balancing test involves a process of weighing the public gain to

be derived from a speedy removal of the nonconforming use against the private loss which removal of the use would entail

o Factors courts use to make this determination include nature of use, character of location, portion of the user’s total business affected, salvage value, extent of depreciation of use

2. Minority Amortization Rule: Invalid per se Forced phase outs are invalid bc either localities lack statutory authority

to impose forced phase outs or the phase out constitutes an uncompensated taking of property no matter what lengh the amortization period is

PA Northwestern Distributors Inc v Zoning Hearing Board (1991)P opened an adult bookstore in a town. 4 days later, the town published a public notice that it will amend its zoning ordiance to regulate adult commercial enterprises. 2 weeks later, Board adopted an ordiance imposing restrictions on the location and operation of adult commercial enterprises and contained an ammortiztion provision giving 90 days for nonconforming pre-exisitng uses to come in compliance. P did not meet the restriction in the ordiance.Court held the amortization and discontinuance of a lawful pre-existing nonconforming use is per se confiscatory and violates the Penn Constitution. All nonconforming pre-exiting uses must be grandfathered in.

95

Page 96: Property Final Outline

Court says this is an unlawful taking of persons property bc all investment costs of book store are lost, store has limited bargaining position bc people know they need to get rid of store and property will be cheap

What rule is better? Majority balancing test or minority rule of per see invalido Minority Bright Line rule Arguments

1) affords certainty: person can make decisions and plans and know what the law provides whereas with the balancing test, people prob willing to invest less due risk of losing land (reliance on standard)

2) Less litigations and costs whereas standard require case by case analysis, goes to court, litigation costs.

3) this runs with land bc as amortization period runs out, alienation is harder bc people wont buy subject to uncertainty

4) Amortization DOES NOT take into account subjective valueo Balancing Test Arguments

1) Legislatures have no forsight thus they are overinclusive or underinclsuive and this allows them to adopt to changed circumstances

Counter: legislature should pay for the privilage to take the land 2) with bright line rule, muni is stuck with rule and must deal with it

then muni will try to enact all rules it can even if unsure bc they are stuck with the rules

B. Administrative Discretion: Variances, exceptions, amendments, spot zoning

o 1. Variances every zoning law establishes a zoning appeals board or a board of

adjustments who are authorized to grant variances from the zoning law in the interest of alleviating practical difficulties or unnecessary hardships

variances can be area variances (alleviates siting problems like set back requirements or minimum yard area) or use variances (permit an otherwise probihited use, like a multi home family in a single family district)

o General Standard for granting variance Variance should be granted upon a showing that compliance with the

zoning law would impose undue hardship on the applicant The hardship must not be created by or be peculiar to the owner

Commons v Westwood Zoning Board of Adjustment (1980)Zoning ordinance required that all single family homes be located on lots with at least frontage of 75 ft and be on a lot that is 7000 square feet. P’s 3000 square feet lot was located in residential area which had 7/32 nonconforming lots with frontages under 75 feet. Board of Adjustment denied variance for home with 30 ft of frontage. Board found p failed to demonstrate evience of hardship even though p offered evidence to show it was difficult to sell the lot to neighbors. Board also found granting variance would substantially impair the intent of purpose of zoning plan.Court reversed the denial of a variance and remands reasoning that UNDUE HARDHSIP means that, absent a variance, the property may not effectively be used. This was a condition that Commons may have established by his efforts to either sell the land to his neighbors or acquire additional adjacent land to conform to ordinance area requirements.

96

Page 97: Property Final Outline

Why allow variance?o Over and under inclusive of the zoning ordiance. The ordinance may be unduly

burdensome to some people without the exceptions of a variance. P must establish ordiance imposes undue hardship if self imposed undue hardship, this

doesn’t satisfy the requirement. If personal hardship (i.e. parents dying, this wont suffice)o On remand, how to establish undue hardship

1) show a plan or blueprint of house and show how the house will be consistent with the objectives of the comprehesive plan and will not undermine the purpose of the ordiance

2) show with the ordinance, property will be zoned into innullity 3) show you tried negociating with neighbors to buy land, tried selling, but

nothing is working Court gives creative options: neighbors buy land at fmv as though the variance would be

granted (so higher than current fmv)o Court gives message to all munis: you must be transparent and accountable and also

shape future negotiations btwn parties facilitate private negociations

2. Zoning Amendments and Spot zoningo Spot zoning: a zoning amendment that delivers special private

benefits (and no public benefits) to a small, discrete parcel and land and which is not in conformity with the comprehensive plan

o Presumptive deference: traditional approach is to presume (as with other legislators) that the zoning amendment is valid until the challenger has proven otherwise

State v City of RochesterNeighbors sought a declaratory judgment challenging the validity of an amendment to an ordinance enacted by Rochester City Council which re-zoned a 1.18 acre lan from single family and low density residential use to high density residential use to permit the building of a 49-unit condo apt complex. The owners of land entered into agreement with developer contigent upon re-zoning of the property. The lot was bounded on 2 sides by multi unit apt buildings and on other side was residential uses. Although the planning commission recommended the re-zoning be denied bc it was inconsistent with city’s land use plan, City Council passed the ordiance and re-zoned property. City Council gave no reasons or findings in support for re-zoning although there was evidence that more high density housing was needed in Rochester.The court denied relief bc: 1) amendments to zoning acts are in form legislative and “a legislative body can best determine which zoning classifications best serve the public interest 2) the fact that the amendment was not consistent with the existing plan was NOT, by itself reason to void the amendment or to shift burden of proof to the city

***Bc there was ample evidence to show the new use was compatible with existing uses and served the public need for high density housing, the amendment was reasonably related to THE PULBIC HEALTH, SAFETY, MORALS AND GENERAL WELFARE AND NOT ARBITRARY, CAPRICOUS OR IRRATIONAL

3) No proof that re-zoned property was “an island of nonconforming use,” and no showing of any “substantial diminution” is values due to rezoning, there was no basis for characterizing the amendment as spot zoning

****Presumption of Validity is default position, but if SPOT ZONING?

97

Page 98: Property Final Outline

o zoning amendment that delivers special private benefits to a small, discrete parcel, produces lititle or no public benefits and which is inconsistent with the plan is presumed invalid as spot zoning. Spot zoning signifies the abuse of power and oppression of minorities

o burden shifts to govt to prove amendment bears a substantial relationship to the general welfare of the affected community

EMINENT DOMAIN AN THE PROBLEM WITH REGULATORY TAKINGAll govts in the US have the power to take private property for public purposes, but eminent domain power limited by 5th Amm: “private property shall not be taken for public use w/o just compensation

98

Page 99: Property Final Outline

Takings clause serves 2 important purposeso 1) prevents the forcible redistribution of property: prevents this by stipuating,

through the just compensation requirement, that when govt power is used to take private property, the public pays the property owner the market value of the property

o 2) Public use requirment was designed to prevent any taking, whether or not compensated, that forces a transfer of property from one private party to another w/o any public benefit in the forced transfer

govt power to take property may only be exercised for the public benefit

Principal issues under the Takings Clauseo 1) Public use: is the govt taking of prop for public use? The govt sometimes takes

private prop and conveys it to another private person to reap some collateral public benefit

o 2) Regulatory takings: at what point does govt reg of prop (restricting its use, possession or disposition) becomes so burdensome that it is a de facto taking which triggers the con requirement of just compensation

o 3) Compensation: private prop owner is entited to the fmv of taken property. This fmv includes any reasonable expecations that buyer may have about possible future uses. But the owner is not entitled to any subjective value.

***Biggest danger of Eminent domain: OPPRESSION OF MINORITY INTERESTSo anytime govt power is used to minimize the rights of a few to benefit the majority, those

people don’t have the capital or resources or democratic voice to resort to the democratic process

1. Public use Requirement public use limitation has been eliminated by the Court’s deference to legislative judgments

about what constitutes public use. o As long as the taking is rationally related to any conceivable public purpose, the public

use requirement is satisfied. In essence, public use is whatever the legislature thinks is conducive to the public welfare.

Kelo v City of New London (2005)New London decided to condemn private residential properties in order to assemble a 90 acre tract or an integrated re-development plan that would create more than 1000 jobs, increase taxes and revitalize what had become a blighted city. Portions of the prop were to be conveyed to private developers to construct 1) a small urban village 2) 80 news residences 3) office and retail space 4) marina parking 5) space of a 300mm Pfizer facility. The holdout landowners sued the city arguing that the land grab did not qualify as public use within the meaning of the Takings Clause of the 5th Amendment.The Sup Court held the taking was rationally related to a conceivable public purpose: the plan unquestionably served a public purpose and satisfied the public use requirement of the 5th amm

So long as the condemnation was part of a comprehensive development plan that subjected to thorough deliberation, the court will gove complete deference to the legislature

Dissent: eminent domain should be upheld when there is a social harm to be cured, but not when the beneficiaries are likely to be those citizens w/ disproportionate influence and power in the politicl process

Argue for Landowner

99

Page 100: Property Final Outline

This is just a wealth transfer, from one private party to another private party***under kelo, you cannot do this. It exceeds the govt’s eminent domain power

In Kelo, there was a more direct action btwn the actions the city took and the benefits conferred to the public

If you hold for the govt here, govt will be able to exercise eminent domain easily. The only justification the govt must give in the future for transferring a private to private transfer without a public benefit is that increases tax revenues and tourism

Argue for Govt Kelo: this is a taking for public use to invigorate the economy Legislature is in the best position to improve the public welfare of the community bc the

legislature has identified a harm, although not a blight, there is an economic harm

2. Regulatory Takings: when does govt regulation becomes so extensive that it amounts to a de facto taking even though the govt denies its taking property?

Per se Ruleso Rule 1) Permanent Physical Occupation Is a Taking: a taking has occurred if

a regulation produces a permanent physical occupation of all or part of the property. Temporary occupations are not a per se taking

Loretto v Teleprompter Manhattan (1982) NY law provides that a landlord must permit a cable television company to install its cable facilities upon his property and that the State Commission on Cable Television will determine reasonable compensation, which was set at 1. The cable installation occupied minor portions of Loretto’s five-story apartment building. The US Supreme Court held a permanent physical occupation by government (as opposed by temporary invasion) is always a taking a requires compensation, without regard to whether the action achieves an important public benefit or has only minimal economic impact on the owner.

The right to exclude is huge consideration for Marshall:“The power to exclude has traditionally been considered one of most treasured strands in owner’s bundle of property rights.”

Argue for Landowner this is authorizing a permanent physical occupation **an attorney for the landowner here would probably advise that suit is not worth the expense

or time of litigating bc this is def a taking but based on Loretta, just compensation will be lowGovt

yes, it’s a taking, thus just compensation similar to Loretta, is $1o the wire cables in Loretta diminished the value of the property and the court decided the

diminution in mv was $1; the permanent physical occupation here increase the value of the property

o Rule 2) Nuissance Abatment: if govt regulates property to abate activities that are common law nuisances, there is no taking, even though the regulations might bar all economically viable uses of the property

Hadacheck v Sebastian (1915)P violated an ordinance which prohibited the establishment and operation of a brickyard and burning of bricks. P is the owner of a tract of land where there’s a valuable bed of clay and where he manufacturers bricks (a business worth to him not less than $800,000; $60,000 for residential.

100

Page 101: Property Final Outline

The US Supreme Court held that if the municipality or state can justify an ordiance as true harm prevention legislation used to promote the publc welfare, then it’s a legitimate use of the govt’s police police power, its not a taking and the court will defer to the legislature’s judgment.

Landowner must argue: o Conferring a benefit upon a particular group of people at the expense of others is NOT

harm prevention: BENEFIT CONFERRING ON THE MINORITY IS A TAKINGArgue for govt:

City would argue Hadycheck: preventing the harmo Here, the historic tapes are valuable bc it preserves our culture history and it would be

harmful to destroy the tapes to the extent we can learn from them for the benefit of the public

This is not a lucas case: bc there is not a 100% wipeout of the value. It still has value

Rule 3. Balancing Public benefits and Private Costs (if per se rules don’t resolve issue)

101

Page 102: Property Final Outline

If the per se rules don’t resolve the issue of whether a regulation is a taking, courts weigh the public benefits achieve by the regulation against the private costs imposed

A reg is not a taking if it substantially advances a legitimate state objectiveo To determine this, following conditions must exist

1) public benefits from the regs must outweigh the private costs of the reg 2) regulation must not be arbitrary 3) prop owner must be permitted to earn a reasonable return on investment on

the prop

Penn Central Transportation Co v NY (1978)In 1968, Penn Central entered into a lease w/ Union General Properties, which was to build a multistory office building above the Terminal. Both Penn Central and UGP applied to the Commission for permission to construct the office building on top of the terminal (they had 2 plans), and were denied. Instead of coming up with new plans or appealing, they filed suit in NY Supreme Court, claiming, among other things, that the application of the Landmarks Preservation Law had taken their property w/out just compensation in violation of the 5th and 14th Amendments and arbitrarily deprived them of their property w/out due process in violation of the 14th. The court upheld the law by using a balancing test that turned on number of factors:

o 1) the nature and importance of the govt regulation and the govt’s interests (the more akin to a physical invasion the more likely a taking; the more it merely adjusts the benefits and burdens of economic life to the promote the common good the less likely it is);

o 2) the reasonable expectations of the property owner (the stronger the “investment-backed expectations” and the more thoroughly frustrated they are by the regulation the more likely there is a taking);

Court held: The Landmarks Law posed no threat of physical invasion, left Penn Central with the ability to

earn a reasonable return on its investment-backed expectations, and did not raise issues of govt use. The restrictions imposed were substantially related to the promotion of the general welfare.

P could argue that 1) it’s a complete deprivation of their air rights 2) it seems arbitrary; it gives a group of citizens of whose property should and should not be protected 3) reverse spot zoning problem; it’s a potential discrimination issue where minority rights are being infringed, when they don’t have power through the democratic process; 4) this doesn’t fall into the ordinary scope of the police power (it’s not a health or safety issue)Govt Argues:

Distinguish Penn Central. The ends here is to preserve our cultural history Landlord counter: even if the regulation is important, the method or the means

do not ensure the goals or end Landlord: there is no greater good here by keeping the property in its original

form like in penn central Govt: investment backed expectations

o When he bought the tapes he prob didn’t expect to splice the tapes and as the court is penn central said, the owner can still maintain the use of the tapes when he bought them thus there is no diminution is mv

102

Page 103: Property Final Outline

Rule 4: Loss of all Economically Viable Use is per se taking: If a govt reg leaves the owner with no econmicaly viable use of his property, it’s a taking

Lucas v South Carolina Coastal CouncilSC’s beachfront mgmt act, as applied to Lucas’ lots forbade the construction of any permanent habitable structures. Trial court concluded the regulation rendered the property valueless.Court held that a reg that deprives a landowner of all economically viable uses is a per se taking unless the loss of all economically viable use results from abatment of a common law nuisance

Exception: if it’s a nuisance, it would make the reg a per se non-taking The court justified the rule bc

o 1) total deprivation of beneficial use is, from the landowners pov, the equivalent of a physical appropriation AND

o 2) these regulations carry with them the heightened risk that private prop is being pressed into some form of public service under the guise of mitigating public harm

Palazzolo v. Rhode Island (2001)RI corp owned a 20 acre undeveloped parcel that was mostly a salt marsh wetland. During that period, various new regs were adopted that effectively barred development of the wetland. The title of the property passed from the corp to the p, who sought approval to develop the parcel. Upon denial of the plans because the land required significant filling. p brought suit contending RI had taken his property. The RI sup court ruled that 1) the use restrictions in place in 1978, when p acquired title, were part of the background title he had acquired and thus he could not assert they constituted a taking and 2) p was not deprived of all economically viable use of the property.Sup court held that: 1) RI court erred in holding the p did not have standing to sue bc he acquired the property after the enactment of the regulations and2) affirmed the RI court’s holding that the use regs did not deprive the p of all economically viable use bc he could still build a large residence on the upland portion of the property****Court remands for determination of whether, under Penn Central Test, the regs constituted a taking

P’s Big Mistake: p argued that the upland portion on which he was permitted to build, was a separate parcel and he was stripped of all economically viable use

Bc this argument was not made in lower court, p cannot argue it on appeal

After Lucas, The Test Is: 1) if it’s a permanent physical occupation, this is a taking and just compensation is required 2) if the govt can justify the ordinance as harm prevention and thus a legitimate interest under

the govt’s police power, its NO TAKING UNLESS the property owner suffers a total 100% wipeout and is deprived of all economic use of his property

3) the prop owner still might lose even if it’s a total wipeout, if the govt can prove they can prohibit the property owner’s use of the land thru nuisance law

4) if the govt cannot prove it’s a legitimate interest justified as harm prevention under the police power and they cannot prove they could prohibit the use via nuisance law then move to Penn Central Balancing Test

o Balance: Govt’s legitimate interests versus the owner’s reasonable investment backed expectations

103